olimpiade2009

157
Olimpiadele de matematica 2009 Posa Bogdan-Petre,Dragoi Marius-Valentin,Stoean Bogdan,Voinescu Sorin "Nu te ingrijora de dicultatile tale la matematica;te asigur ca ale mele sunt mult mai mari." Dedicam aceasta carte profesorilor nostri de matematica Paicu Gheorghe,Drula Valeriu si Paralescu Iustin , precum si familiei si prietenilor.

Upload: bogdanlyex

Post on 25-Jun-2015

738 views

Category:

Documents


2 download

TRANSCRIPT

Page 1: olimpiade2009

Olimpiadele de matematica 2009

Posa Bogdan-Petre,Dragoi Marius-Valentin,Stoean Bogdan,Voinescu Sorin

”Nu te ingrijora de dificultatile tale la matematica;te asigur ca alemele sunt mult mai mari.”

Dedicam aceasta carte profesorilor nostri de matematica PaicuGheorghe,Drula Valeriu si Paralescu Iustin , precum si familiei si

prietenilor.

Page 2: olimpiade2009

2

Cuvant inainte

Prezentul volum reprezinta debutul editorial al unor tineri entuziasti,interesatide lumea matematicii,dar care,in relatie cu domeniul pasiunii lor,nu mai suntde mult timp niste debutanti.

Marius-Valentin Dragoi,Bogdan-Petre Posa,Bogdan Stoean,Sorin Voinescu suntfosti elevi ai Colegiului National ”George Cosbuc” din Motru,participanti lazeci de concursuri de matematica,castigatori a numeroase premii in cadrul unorcompetitii organizate la nivel interjudetean si national,rezultate concludente fi-ind obtinerea medaliilor de argint si de bronz la faza natioanla a olimpiadei dematematica,timp de 5 ani consecutivi,si selectarea in lotul largit al Romanieipentru pregatirea Olimpiadei Balcanice de Matematica.

Cartea de fata ofera suport pentru pregatire scolara celor aflati acum pe ban-cile liceului si invita in acelasi timp la colaborare pe taramul matematicii,tineriiautorii fiind oricand deschisi dialogului cu cititorii.

Structurandu-si materialul in trei capitole -”Enunturi” ,”Solutii”,”Problemepropuse”-,autorii dovedesc interes si pentru zona cercetarii matematice,ultimaparte o volumului cuprinzand,alaturi de probleme care apartin unor matemati-cieni cunoscuti,probleme originale,personale,rod al preocuparilor pentru studiuconsecvent,al placerii de a raspunde la provocari,de a crea si de a traversa ob-stacole.

Ca truditor alaturi de acesti matematicieni in devenire,am participat la pre-gatirea lor stiintifica,le-am fost alaturi in momentele de cumpana,le-am impar-tasit bucuriile si am trait propriile satisfactii urmarindu-le traiectoria promi-tatoare, de la statutul de rezolvitor de probleme pana la insusirea unui modpersonal de a gandi lumea,logic,rational,activ si creativ.

Prof.Valeriu Drula

Page 3: olimpiade2009

Cuprins

3

Page 4: olimpiade2009

Partea I

Enunturi

4

Page 5: olimpiade2009

Capitolul 1

Clasa IX

1.1 Etapa locala

1.1.1 Arges

1. Sa se rezolve in multimea numerelor reale ecuatia: 27d2(x) − 2[x] + 3 = 0(unde [x] reprezinta partea reala a numarului real x, iar d(x) = min∣x − k∣,k ∈ Z)

Marian Teler, profesor CostestiMarin Ionescu, profesor Pitesti

2. Sa se arate ca daca numarul natural x ≥ 2 si patratul sau sunt termeniiunei progresii aritmetice infinite crescatoare de numere reale, atunci aceastaprogresie contine toate numerele xn, pentru orice n ∈ N .

I.S.J Arges

3. Fie functia f : R → R avand proprietatea f(x + 2) + f(x − 2) = f(x),x ∈ R.

5

Page 6: olimpiade2009

CAPITOLUL 1. CLASA IX 6

a) Sa se arate ca functia este periodica.

b) Aratati ca functia f : R→ R , f(x) =x

12− [

x

12]

I.S.J Arges

4. Se dau vectorii nenuli u, v astfel incat ∣u + v∣ = ∣u∣ + ∣v∣. Aratati ca ceidoi vectori sunt coliniari si au acelasi sens.

Marian Teler, profesor Costesti

1.1.2 Braila

1. Fie triunghiul ABC. Paralela prin A la BC taie paralela prin B la AC in P siparalela prin C la AB in M. Paralela prin B la AC taie paralela prin C la AB inN. Fie G1,G2,G3 centrele de greutate ale triunghiurilor ACM, ABP si respectivBCN. Sa se arate ca triunghiurile ABC, MNP si G1G2G3 au acelasi centru degreutate.

prof. Carmen si Viorel Botea

2. Fie S = n+ (n+ 2) + (n+ 4) + ...+ 3n, n ∈ N∗ .a)Determinati valorile lui n stiind ca numarul S are 3 cifre.b)Pentru n=2009, aflati cate cifre are numarul de cifre ale lui S2009.

prof. Valentin Damian

3. Sa se arate ca (1 + x2008)2009 ≥ (1 + x2009)2008.

prof. Valentin Damian

Page 7: olimpiade2009

CAPITOLUL 1. CLASA IX 7

4. Sa se determine m,n,p ∈ N cu n ∕= 0 astfel incat [x+1

n]+[x+

m√

2

n] = [px],

∀x ∈ R, unde [x] reprezinta partea intreaga a numarului real x.

prof. Dan Negulescu

1.1.3 Caras-Severin

1. Sa se demonstreze ca daca x,y,z, ∈ (0,∞) si xyz=1 atunci:

1

x+ y +√z

+1

y + z +√x

+1

z + x+√y<

3√

2

4.

Lucian Dragomir, Otelu-Rosu

2. a)Sa se arate ca nu exista numare reale x pentru care [x

2] = x2 + 1

b)Sa se determine numerele reale y pentru care [y

2] = y2.

Lucian Dragomir, Otelu- Rosu, articol RMCS 25/2008

3. Sa se arate ca numarul an = 3n+1 + 2n+ 1 este divizibil cu 4, ∀n ∈ N .

Marin Chirciu, Pitesti, RMT 4/2008

4. Pentru orice triunghi ABC se noteaza AB=c, BC=a, CA=b, iar I estecentrul cercului inscris, iar G este centrul de greutate. Sa se arate ca:

a)−→GI =

1

a+ b+ c(a−→GA+ b

−−→GB + c

−−→GC).

b)daca c=3,b=4,a=5, atunci IG este paralela cu AC.

Page 8: olimpiade2009

CAPITOLUL 1. CLASA IX 8

Gabriel Popa, Iasi, GM 3/2008

1.1.4 Calarasi

1.a) Daca n ∈ N , n ≥ 2 atunci sa se calculeze produsul23 − 1

23 + 1

33 − 1

33 + 1...n3 − 1

n3 + 1.

b) Daca ABC si A1B1C1 sunt triunghiuri dreptunghice (A si A1 unghiuri drepte)aratati ca BCB1C1 ≥ ABA1B1 +ACA1C1.

Cristina Bornea, Calarasi

2. Aratati ca propozitia ” ∀a, b ∈ (0,∞), a ≤ b si ∀x, y ∈ [a, b], 2 ≤ x

y+y

x≤

a

b+b

a” este adevarata.

Gheorghe Fianu , Stefan cel Mare

3. Fie A1A2...A7 un poligon cu 7 laturi. Sa se arate ca daca G este centrulde greutate al poligonului,G1 este centrul de greutate al triunghiului A1A3A5

iar G2 este centrul de greutate al patrulaterului A2A4A6A7 ,atunci G,G1, G2

sunt coliniare.

Dan Marinache, Calarasi

4. Se considera progresia geometrica (bn)n≥1. Sa se arate ca:n∑k=1

(−1)k−1b2k ≥4n∑

k=2n+1

(−1)k−1b2k.

G.M. 4/2008

Page 9: olimpiade2009

CAPITOLUL 1. CLASA IX 9

1.1.5 Cluj

1. Sa se rezolve ecuatia: [√

16− x2] =x2 + 1

5, unde [x] reprezinta partea

intreaga a numarului real x.

prof. Violin Gorcea, Lic.T. Avram Iancu

2. Sa se rezolve in R ecuatia: [x− 2

3] = [

x− 3

2], unde [x] reprezinta partea

intreaga a numarului real x.

prof. Alb Nicolae, Lic.T. O.Goga Huedin

3. Se considera progresia geometrica (bn)n≥1. Sa se arate ca:b21 − b22 + b23 − b24 + ...+ b22n−1 − b22n ≥ b22n+1 − b22n+2 + ...+ b24n−1 − b24n, oricarear fi n ≥ 1

prof. Ilie Diaconu, Lic.T. Avram Iancu

4. Demonstrati ca daca a,b,c,x,y,z sunt numere reale strict pozitive, atunci:

a)x

a+y

b+z

c≥ (x+ y + z)2

ax+ by + cz;

b)a+ 2b

a+ 2c+b+ 2c

b+ 2a+c+ 2a

c+ 2b≥ 3.

prof. Ilie Diaconu, Lic.T. Avram Iancu

5. Fie ABCD un patrulater inscriptibil. Notam Ha ortocentrul triunghiuluiBCD si cu Ma mijlocul lui AHa. In mod similar definim punctele Mb,Mc,Md.Demonstrati ca punctele Ma,Mb,Mc si Md coincid.

prof. Magdas Camelia, prof. Jecan Eugen, Col.Nat. A.Muresanu, Dej

Page 10: olimpiade2009

CAPITOLUL 1. CLASA IX 10

1.1.6 Dolj

1. Sa se arate ca pentru orice numar natural n ≥ 2, avem identitatea:[√

1] + [√

2] + ...+ [√n2 − 1] + 12 + 22 + ...+n2 = n3, unde [a] reprezinta partea

intreaga a numarului a.

GM nr.4/2004, E:24957

2. Fie functia f : R → R cu proprietatea ca f(x) + 2f(1

x) = x2 + 3 +

2

x2,

∀x ∈ R∗. Sa se calculeze suma

2007∑k=2

1

f(k)− 2.

GM nr.2/2008, E:25840

3. Fie a1, a2, a3, ..., a12 o progresie geometrica crescatoare cu termeni pozi-tivi. Demonstrati ca a12 − a1 ≥ 11 ⋅ (a7 − a6).

***

4. Fie ABCDE un pentagon inscris intr-un cerc. Notam cu H1, H2, H3, H4

ortocentrele triunghiurilor ABC, BCD, CDE, respectiv ACE. Sa se demonstrezeca H1H2H3H4 este paralelogram.

Olimpiada faza local, Bucuresti, 2008.

1.1.7 Galati

1. Sa se rezolve ecuatia [x− 2

2] + [

2x− 1

4] =

3

2x− 2, unde [x] reprezinta partea

intreaga a numarului real x.

Page 11: olimpiade2009

CAPITOLUL 1. CLASA IX 11

Conf. Dr. Ion Mirica

2. Sa se calculeze expresia:

E =sin 53∘ − 2 cos 40∘ sin 13∘

cos 63∘.

prof. Veronica Grigore

3. Fie an > 0, n ∈ N∗, pentru care an2 ≤ an − an+1, (∀)n ∈ N∗.

Sa se arate ca [n⋅ an] = 1, (∀)n ∈ N∗.

prof. Iuliana Duma

4. Fie unghiul ABC si A′ ∈ (BC), B′ ∈ (AC), C ′ ∈ (AB) astfel incat AA’,BB’, CC’ sunt concurente in M. Demonstrati ca:

a)MA

MA′=C ′A

C ′B+B′A

B′C

b)MA

MA′+MB

MB′+MC

MC ′+ 2 =

MA

MA′⋅ MB

MB′⋅ MC

MC ′.

c)MA

MA′+MB

MB′+MC

MC ′≥ 6.

prof. Vasile Dumbrava

1.1.8 Gorj

Subiectul 1Daca a, b ∈ R∗ si a+ 2008

√ab = 2009b, sa se calculeze

a

b.

Subiectul 2

Page 12: olimpiade2009

CAPITOLUL 1. CLASA IX 12

a) Daca a si b sunt numere reale, aratati ca: a2 − ab+ b2 ≥(a+ b

2

)2

.

b) Demonstrati ca:√x2

1 − x1x2 + x22+√x2

2 − x2x3 + x23+⋅ ⋅ ⋅+

√x2n − xnx1 + x2

1 ≥ x1+x2+⋅ ⋅ ⋅+xn∀x1, x2, ⋅ ⋅ ⋅ , xn ∈ R si n ∈ N,n ≥ 2. In ce caz avem egalitate?Subiectul 3Se considera punctele A,B,C,M,N, P necoliniare doua cate doua astfel ca−−→MA+

−−→MB+

−−→NA+

−−→NC+

−−→PB+

−−→PC =

−→0 . Daca H si O sunt respectiv ortocentrul

si centrul cercului circumscris triunghiului ABC , sa se arate ca:−−→OM+

−→On−−→OP =−−→

OH.Subiectul 4Sa se determine prima zecimala a numarului a =

√n2 + n, ∀ninN∗.

1.1.9 Hunedoara

1. a) Fie a1, a2, a3, ..., an, ... termenii pozitivi ai unei progresii aritmetice. Sa sedemonstreze inegalitatea:

a1⋅ a2⋅ a3⋅ ...⋅ a2n+1

a2⋅ a4⋅ ...⋅ a2n≤√a1⋅ an

b) Determinati x,y,z ∈ R−{�2 , k ∈ Z

}pentru care

2 (tg x+ tg y + tg z)−(tg2x+ tg2y + tg2z

)≥ 3

2. Rezolvati in N ecuatia:

1

x+

1

y=

1

41.

3. a) Demonstrati ca: [xy] ≥ [x]⋅ [y], (∀) x, y ≥ 0

b) Fie S(n) =

n∑k=1

1

[√k4 + 4(k3) + 5(k2) + 4k + 1]

, n ∈ N∗, unde [k] reprez-

inta partea intreaga a lui k. Demonstrati ca: S(n) < 1, (∀)n ∈ N∗.

4. Fie triunghiul ΔABC, punctele A′ ∈ (BC), B′ ∈ (AC), iar P = AA′ ∩BB′.a) Demonstrati ca oricare ar fi punctul M in planul triunghiului ΔABC, exista�, �, ∈ (0, 1) astfel incat

Page 13: olimpiade2009

CAPITOLUL 1. CLASA IX 13

−−→MP = �⋅ −−→MA+ �⋅ −−→MB + ⋅ −−→MC.

b) Aratati ca dacaA′C

A′B-B′C

B′A= 1, atunci punctul P este mijlocul segmentului

(BB’).

1.1.10 Iasi

1. Sa se rezolve ecuatia [x+ k] {x+ k} = xk, unde k ∈ Z.

2. Sa se arate ca pentru orice numere reale pozitive a,b si c, are loc inegali-tatea:

a+ b

a2 + b2+

b+ c

b2 + c2+

c+ a

c2 + a2≤ 1

a+

1

b+

1

c.

3. Fie ABCD, un patrulater convex.

a) Construiti punctele N pe (AD) si Q pe (BC) astfel incat−−→AN =

5

7

−−→AD si

−−→BQ =

4

7

−−→BC.

b) Determinati locul geometric al punctelor M din planul patrulaterului ABCD

pentru care modulul vectorului 2−−→MA + 3

−−→MB + 4

−−→MC +5

−−→MD are valoare con-

stanta.

4. Se considera punctele D si M in planul triunghiului ABC astfel incat34−−→MA + 36

−−→MB + 5

−−→MC = 0 si

−−→AD= 18

35

−−→AB. Fie Q punctul de intersectie

dintre dreptele AM si BC.a) Demonstrati ca punctele C, M si D sunt coliniare.

b) AflatiMA

MQ.

Subiect elaborat de Ioana Galan

Page 14: olimpiade2009

CAPITOLUL 1. CLASA IX 14

1.1.11 Prahova

1. a) Demonstrati ca daca produsul a doua numere pozitive este constant, atuncisuma lor este minima cand numerele sunt egale.b) Formati o progresie aritmetica cu n termeni pozitivi stiind ca produsul dintreprimul termen si ratie este a, iar suma celor n termeni ai progresiei este minima.

***

2. Sa se rezolve sistemul:

a[a] + c {c} - [b] {b} = 0,164b[b] + 4a {a} - 4[c] {c} = 1c[c] + b {b} - [a] {a} = 0,49

unde [x], x reprezinta partea intreaga respectiv partea fractionara a numaruluireal x.

Prof Gabriel Necula, Plopeni

3. Fie triunghiul ABC si punctele D-piciorul bisectoarei din A, I1 si I2-centrele cercurilor inscrise in triunghiurile ABD respectiv ACD. Se stie ca BC=12,AD=8 si I1I2∣∣BC.

a) Demonstrati ca AB=AC;

b) Daca−−→AI1 = u si

−−→AI2 = v, exprimati

−−→AB si

−−→AB in functie de u si v.

Prof Claudiu Militaru, Ploiesti

4. In triunghiul ABC se considera punctele D, E, F - mijloacele laturilorAB, BC, AC si M ∈ (BE), N ∈ (CE).

Page 15: olimpiade2009

CAPITOLUL 1. CLASA IX 15

Aratati ca AE, DM, FN sunt concurente daca si numai dacaBM

ME=CN

NE∕=

1.

Prof. Ion Nedelcu, Ploiesti

1.1.12 Sibiu

1. Determinati sirul (xn)n∈N∗ cu termeni strict pozitivi, astfel incat:

x13 + x2

3 + ...+ xn3 = (x1 + x2 + ...+ xn)2,∀n ∈ N∗.

***

2. Fie Sn = 5n + 7n, n ∈ N . Aratati ca:

a) Sn+2 = 12Sn+1 − 35Sn, ∀n ∈ N ;b) Sn∣Sn+1 ⇔ n = 0.

Gazeta Matematica

3. Se considera o progresie aritmetica (an)n≥1 cu ratia r ∈ [0, 1). Demon-strati ca exista n ∈ N∗, astfel incat [an]=[an+1]. (Prin [x] s-a notat parteaintreaga a numarului real x).

Prof. Alin Pop

4. Se considera triunghiurile ABC si A’, B’, C’ inscrise intr-un cerc de centruO. Daca H1, H2, H3, H4, H5, H6 sunt ortocentrele triunghiurilor BAA’, CAA’,CBB’, ABB’, ACC’ si respectiv BCC’. Demonstrati relatiile:

Page 16: olimpiade2009

CAPITOLUL 1. CLASA IX 16

a)−−→OH1=

−→OA+

−−→OB +

−−→OA′ (Relatia lui Sylvester);

b)−−−→H1H2+

−−−→H3H4+

−−−→H5H6=0.

Prof. Petru Vlad

1.1.13 Timis

1. Sa se rezolve ecuatia [-2x]+[-x]+[x]+[2x]+1=0.

Prof. Andrei Eckstein

2. Fie [AA’ bisectoarea interioara a unghiului A al triunghiului ABC cuA′ ∈ (BC) si A” simetricul cu A’ fata de mijlocul laturii (BC).

a) Sa se exprime−−→AA” in functie de

−−→AB si

−→AC;

b) Sa se arate ca (b+ c)−−→AA”+(c+ a)

−−−→BB”+(a+ b)

−−→CC”.

Prof. Aurel Dobosan

3. Daca a,b,c sunt numere reale a caror suma este 0 sau 1, sa se demonstrezeinegalitatea:

ab+ bc+ ca <3

2.

***

4. Fie x,y,z trei numere reale pozitive, cu x+y+z=1, iar ΔABC un triunghioarecare. Pe laturile acestuia se considera punctele A1 ∈ (BC), B1 ∈ (CA),C1 ∈ (AB), astfel incat:

Page 17: olimpiade2009

CAPITOLUL 1. CLASA IX 17

A1B

A1C=z

y, B1CB1A

= xz , C1A

C1B= y

x .

Aratati ca:a) dreptele AA1, BB1, CC1 sunt concurente intr-un punct S.

b) pentru orice punct P din plan are loc egalitatea:−→PS = x

−→PA + y

−−→PB + z

−−→PC.

Prof. Mihai Chis

1.1.14 Vaslui

1. Sa se rezolve ecuatia: [x− 2

2]− (x− 2)=x

2 - 2x−14 .

Prof. Teclici Daniela

2. Fie numerele x,y,z ∈ R+ si x+y+z=1. Sa se demonstreze ca au loc ine-galitatile:

0 ≤ xy + yz + zx− xyz ≤ 8

27.

Cand au loc egalitatile?

Prof. Mihalache Dumitru

3. Fie A1, B1, C1 puncte pe laturile BC, CA, AB ale triunghiului ABC.notam cu G1, G2, G3, G,G

′, G” centrele de greutate ale triunghiurilor AB1C1,BC1A1, CA1B1, ABC, G1G2G3 respectiv A1B1C1. Sa se arate ca G, G’, G”sunt coliniare si G”G = 3G”G’.

G.M. 7-8/2008

Page 18: olimpiade2009

CAPITOLUL 1. CLASA IX 18

4. Fie f : N → N definita prin f(0)=0 si f(n+1)=2(n+1+f(n)), ∀n ∈ N .a) Demonstrati ca f(n) < 2n+2, pentru orice n ∈ N .b) Aflati n pentru care f(n) este putere (cu exponent intreg) a lui 2.

***

1.1.15 Valcea

1. Fie p,q doua numere reale care verifica relatia (p2 + q)(p+ q− 1)+q-1=0. Sase arate ca daca p este numar natural nenul, atunci q este irational.

Prof. Vasile Pop, Universitatea Tehnica Cluj-Napoca

2. a) Sa se arate ca:

√n− 1

n<

2n− 1

2n<

√2n− 1

2n+ 1, ∀n ∈ N∗.

b) Sa se arate ca:1

2√n<

1⋅ 3⋅ 5⋅ ...⋅ (2n− 1)

2⋅ 4⋅ 6...⋅ 2n<

1√2n+ 1

, ∀n ∈ N,n ≥ 2.

***

3. Se considera 2009 puncte in plan, oricare trei necoliniare, astfel incat ariaoricarui triunghi determinat de acestea sa fie mai mica decat 1. Sa se arate catoate punctele considerate sunt in interiorul unui triunghi de arie egala cu 4.

***

4. In patrulaterul convex ABCD se noteaza cu G centrul de greutate al tri-unghiului BCD si cu H ortocentrul triunghiului ACD. Sa se arate ca patrulaterul

Page 19: olimpiade2009

CAPITOLUL 1. CLASA IX 19

ABGH este paralelogram daca si numai daca G este centrul cercului circumscristriunghiului ACD.

***

1.2 Olimpiada judeteana

Subiectul 1.Pe laturile AB si AC ale triunghiului ABC se considera puncteleD si respectiv E,astfel incat DA+DB+EA+EC = 0.Fie T intersectia dreptelorDC si BE.Sa se determine � real astfel incat TB + TC = �TA.

Gazeta Matematica

Subiectul 2.Elementele multimiiM = 1, 2, 3, ..., 99, 100 se aseaza intr-un tabloucu 10 linii si 10 coloane,astfel:

1 2 ... 1011 12 .. 20... ... ... ...91 92 ... 100

Sa se arate ca oricum am sterge 10 elemente ale tabloului , printre cele 90 denumere ramase exista cel putin 10 numere in progresie aritmetica.

Bogdan Enescu,Buzau

Subiectul 3.a)Fie a, b ≥ 0 si x, y > 0.Sa se arate ca a3

x2 + b3

y2 ≥(a+b)3

(x+y)3 .

b)Fie a, b, c ≥ 0 si x, y, z > 0 astfel incat a+ b+ c = x+ y+ z.Sa se arate caa3

x2 + b3

y2 + c3

z2 ≥ a+ b+ c.

Dan Marinescu,Viorel Cornea,Hunedoara

Page 20: olimpiade2009

CAPITOLUL 1. CLASA IX 20

Subiectul 4.Sa se determine functiile f : N∗ → N∗ pentru care f(x+y)+f(x)2x+f(y) =

2y+f(x)f(x+y)+f(y) pentru orice x, y ∈ N∗.

Nelu Chichirim, Constanta

1.3 Olimpiada nationala

Subiectul 1.Fie ABC un triunghi oarecare si k un numar real nenul.Pe laturileAB si AC ale triunghiului consideram punctele variabile M ,respectiv N ,astfelincat MB

MA −NCNA = k.Aratati ca dreapta MN trece printr-un punct fix.

Dan Nedeianu

Subiectul 2.Fiind date numerele reale a, b, c, d > 0 si e, f, g, ℎ < 0, demonstratica inegalitatile ae + bc > 0, ef + cg > 0, fd + gℎ > 0, da + ℎb > 0, nu pot fisimultan indeplinite.

Ion Nedelcu

Subiectul 3.Fiind date numerele reale pozitive distincte a1, a2, ..., an, n ∈ N ,sio rearanjare b1, b2, ..., bn a lor ,demonstrati inegalitatea (a2

1 +b1)(a22 +b2)...(a2

n+bn) ≥ (a2

1 + a1)(a22 + a2)...(a2

n + an).

Dan Nedeianu

Subiectul 4.Fiind date secventele ordonate de numere reale distincte a1 < a2 <... < an si b1 < b2 < .. < bm, n,m ∈ N,n,m ≥ 2, consideram multimea {ai +bj ; 1 ≤ i ≤ n, 1 ≤ j ≤ m}. Demonstrati ca aceasta multime are exact n+m− 1elemente daca si numai daca ambele secvente sunt in progresie aritmetica deaceeasi ratie .

Vasile Pop

Page 21: olimpiade2009

CAPITOLUL 1. CLASA IX 21

1.4 Concursuri interjudetene

1.4.1 Concursul ”Alexandru Myller”

1. Determinati n ∈ N∗ pentru care exista o multime A ⊂ R cu n elemente,avand proprietatea a(b3 + 6) ≤ b(a3 + 6), ∀a, b ∈ A.

Gheorghe Iurea

2. Care este numarul minim de elemente care trebuie eliminate din multimea{1, 2, 3, ..., 100}, astfel incat in multimea ramasa sa nu existe trei elemente x,y,zastfel incat xy=z?

***

3. Fie triunghiul ABC si punctele M ∈ (AB), N ∈ (AC) astfel incat BM +CN = MN + BC. Notam p raza cercului inscris in triunghiul AMN. Aratati ca:

p(√bc+

√(p− b)(p− c)) ≤ r(

√bc−

√(p− b)(p− c)).

Dan Branzei

4. Numarul intreg m are proprietatea ca, pentru orice numar natural n,exista an ∈ Z astfel incat ∣nm− 80an + 1∣ < 20. Aratati ca 80 divide m.

Dinu Serbanescu

Page 22: olimpiade2009

CAPITOLUL 1. CLASA IX 22

1.4.2 Concursul ”UNIREA”

1. Rezolvati ecuatia:[x

2

]+ {4x} = 4

√5− 7.

2. Se considera patrulaterul ABCD, fara unghiuri opuse congruente si punc-tul E = [AC] ∩ [BD]. Aratati ca patrulaterul este inscriptibil daca si numaidaca BC⋅CD⋅AE = AB⋅AD⋅EC.

3. Fie (an)n≥1, (bn)n≥1 doua progresii aritmetice. Aratati ca sirul (un)n≥1,dat de:

un = an+2bn+2 − 2an+1bn+1 + anbn, ∀n ≥ 1

este constant.

4. Fie I centrul cercului inscris in triunghiul isoscel ABC (AB = AC) siIL ∥ AB, L ∈ (AC). Demonstrati ca LO ⊥ CI, unde O este centrul cerculuicircumscris triunghiului.

1.4.3 Concursul ”Petru Morosan-Trident”

1. Sa se determine f : R → R cu 2f(x) − 3f(1

x) = 2x2 − 3

x2+ 12x − 18

x− 9,

(∀)x ∈ R∗.

Calculati suma: S =

2009∑k=1

1

f(k)− 4.

Prof. Daniela Covaci, Braila

2. Aratati ca ∀n ≥ 2, n ∈ N , [√

1]2+[√

2]2+...+[√n2 − 1]2 ≥ n2(n− 1)(4n+ 1)2

36(n+ 1),

unde [x] este partea intreaga a numarului x.

Prof. Carmen Botea, Braila

Page 23: olimpiade2009

CAPITOLUL 1. CLASA IX 23

3.a) Sa se arate ca in orice patrulater conveg EFGH avem relatia:

EF 2 + FG2 +GH2 +HE2 = EG2 + FH2 + 4MN2,

unde M este mijlocul segmentului EG si N mijlocul segmentului HF.b) Consideram triunghiul ABC. Atunci oricare ar fi punctul D ∈ (BC), avem:

(AB

AD +BD

)2

+

(AC

AD +DC

)2

≥ 1

Prof. Gheorghe Stoica, Petrosani

1.4.4 Concursul ”Vasile Dumitrache”

1. Sa se rezolve sistemul cu coeficienti reali:

⎧⎨⎩ x2 + yz = a2

y2 + xz = a2

z2 + xy = a2, a ∈ R.

2.Daca a1, a2, ..., an ∈ [0, 1], n ∈ N∗n ∕= 1 sa se demonstreze inegalitatea:a1

a1a2+n−1 + a2a2a3+n−1 + ...+ aa

ana1+n−1 ≤ 1

3.Fie P un punct in interiorul unui triunghiu echilateral ABC.Notam P1, P2, P3

proiectiile punctului P le laturile triunghiului.Sa se arata ca−−→PP1 +

−−→PP2 +

−−→PP3 =

32

−−→PO

1.4.5 Concursul ”Victor Valcovici”

1.Fie numarele a1, a2...an ∈ (0,∞),astfel incat a1 +a2 + ...+an = 1. Demon-strati urmatoarea inegalitatea:

Page 24: olimpiade2009

CAPITOLUL 1. CLASA IX 24

(a2

√a2

1 + a24 +a3

√a2

2 + a25 + ...+a1

√a2n + a2

3)(a1

a2 + a3+

a2

a3 + a4+ ...

ana1 + a2

≥√

2

2

2.In triunghiul ABC,[AD] mediana,D ∈ (BC), I1, I2 centrele cercurilor in-scrise in triunghiurile ABD si ACD,G centrul de greutatea al triunghiului ABCsi 2a > b + c,unde a,b,c sunt laturile triunghiului.Daca I1, G, I2 sunt coliniareatunci 2AD = 2a− b− c

Alexe Gheorghe, Braila

3.Sa se arate ca orice progresie aritmetica formata din numere naturale con-tine termenii unei progresii geometrice.

***

1.4.6 Concursul ”Grigore Moisil”

1.Fie a, b, c, x, y, z ∈ (0,∞) astfel incat a2 + b2 = c2 si x2 + y2 = z2.Aratatica :

* * *

(a+ x)2 + (b+ y)2 ≤ (c+ z)2

2.Scrieti numarul

√√5 + 2 +

√√5− 2√√

5 + 1sub forma a+b

√c

d cu a,b,c,d numere

intregi.

* * *

3.Determinati a ∈ N∗ si a, b, c, d ∈ R astfel incat : ∣ax3 + bx2 + cx+ d∣ ≤ 2oricare ar fi x ∈ [−2, 2] .

Cristinel Mortici

4.Pe laturile AB si AC ale triunghiului se considere punctele D si respectivE,astfel incat

−−→DA +

−−→DB +

−→EA +

−−→EC =

−→0 .Fie T intersectia dreptelor DC si

BE.Determinati � ∈ R cu proprietatea ca−→TB +

−→TC = �

−→TA

Page 25: olimpiade2009

CAPITOLUL 1. CLASA IX 25

Gazeta Matematica

1.4.7 Concursul ”Laurentiu Duican”

1.Fie f : R → R o functie de gradul al doilea.Sa se arate ca numarul ele-mentelor multimii A = {x ∈ R∣f(f(x)) = x} este diferit de 3.

Octavian Purcaru

2.Sa se determine toate functiile monotone f : R→ Z care satisfac conditiile:i)f(x) = x, ∀x ∈ Zii)f(x+ y) ≥ f(x) + f(y),∀x ∈ R

Aurel Barsan

3.a)Fie numerele reale x,y,z si t cu proprietatile x < y ≤ z < t si x + t =y + z > 0.Sa se arate ca x3 + t3 = y3 + z3

b)Sa se demonstreze ca pentru oricare a, b, c, d > 0 are loc inegalitatea:3(a+ b+ c+ d)4 +

∑(a+ b)4 > 2

∑(a+ b+ c)4,

unde∑

(a+ b)4 = (a+ b)4 + (a+ c)4 + (a+ d)4 + (b+ c)4 + (b+ d)4 + (c+ d)4

iar∑

(a+ b+ c)4 = (a+ b+ c)4 + (a+ b+ d)4 + (a+ c+ d)4 + (b+ c+ d)4

Eugen Paltanea

4.Fie triunghiul ABC,cu unghiul B ascutit,bisectoarea unghiului B inter-secteaza inaltimea si mediana corespunzatoare varfului A in punctele P si re-

spectiv Q.Sa se demonstreze ca daca AP=AQ,atunciSAPQSABC

≥ (√

2−1)2

2 .Sa sedetermine masura unghiului B in cazul in care se realizeaza egalitatea in relatiade mai sus.

1.4.8 Concursul ”Gheorghe Lazar”

1.Notam cu F multimea functiilor f : R → R care verifica relatia : 2f(2 −3x) + f( 3−x

2 ) = x, ∀x ∈ Ra)Sa se arate ca F contine cel putin o functie bijectiva.a)Sa se arate ca multimea F contine o infinitate de elemente.

Page 26: olimpiade2009

CAPITOLUL 1. CLASA IX 26

2.Fie (ak)k≥1 o progresie aritmetica cu ratia r > 0 si primul termen a1 ≥ 12 .

Sa se calculeze partea intreaga a numarului

sn =

√1 +

r

a1a2+

√1 +

r

a2a3+ ...+

√1 +

r

an−1an+

√1 +

1

an

3.Sa se demonstreze egalitatea:1

cosa+ 2

cosa

cos2a+ ...+ 16

cos15a

cos16a= 0 unde a = �

31

4.Daca ABC este un triunghiu de laturi a,b,c iar 0 < x ≤ y < �2 , atunci:

b2 + c2tg2y > a2sin2x

Page 27: olimpiade2009

Capitolul 2

Clasa X

2.1 Etapal locala

2.1.1 Arges

1.Fie z1, z2 ∈ C∗ astfel incat ∣z1 + z2∣ = ∣z1∣+ ∣z2∣. Sa se arate ca exista � > 0astfel inca z1 = �z2.

2.Sa se rezolve ecuatiile :a)9x − 5x − 4x = 2

√20x

b)√

1 + x+√

1− x = 2x + 2−x.

3.Fie f : N → Z astfel incat (f(n+1)−f(n))(f(n+1)+f(n)+4) ≤ 0,∀n ∈N .Sa se arata ca f nu este injectiva .

Marin Ionescu, profesor Pitesti, Marian Teler, profesor Costesti

4.Fie a, b, c > 1.Sa se demonstreze inegalitatea :(logab− 1 + logcb)(logbc− 1 + logac)(logca− 1 + logba) ≤ 1.

Marin Ionescu, profesor Pitesti

27

Page 28: olimpiade2009

CAPITOLUL 2. CLASA X 28

2.1.2 Braila

1.a) Sa se demonstreze ca numarul log32 este irational.b) Sa se arate ca exista numere irationale x si y astfel incat xy sa fie rational.

2.Determinati z ∈ C stiind ca ∣z − i∣ ≤< 1 si ∣z − 2− i∣ ≤< 1 .

Gazeta matematica

3. Rezolvati in multimea numerelor reale ecuatia:[√x+ 1+[

√2− x] = [2− lnx],unde [A] reprezinta partea reala a numarului real

A.

prof. Valentin Damian

4. Sa se rezolve ecuatia: 149 + 5x+2 = 25x2 + log2(x100)

prof. Gabriel Daniilescu

2.1.3 Caras-Severin

1. Sa se determine numerele naturale n pentru care exista z ∈ C cu ∣z∣ = 1 sizn + z + 1 = 0 .

GM 4/2008

2. Sa se determine x ∈ N pentru care4x + 4x+ 1

2x + x= 2x − 1.

GM 2/2008

3.Sa se determine functiile injective f : R→ R cu proprietatea ca:f(x)f(y) = xf(y) + yf(x)− f(xy), ∀x, y ∈ R.

Iacob Didraga, RMCS 23

Page 29: olimpiade2009

CAPITOLUL 2. CLASA X 29

4.Sa se determine numerele naturale m si n pentru care egalitatea :xn+2⋅x3m+1 = x2mn este adevarata pentru orice x > 0 , x ∕= 1.

RMCS ,enunt modificat

2.1.4 Calarasi

1.Fie z ∈ C∗ fixat.Sa se arate ca numarul t = z∣z∣ + ∣z∣

z ∈ R si apoi sa se demon-

streze ca exista numerele reale m si n astfel incat z2 +mz + n = 0.

2.Fie a, b > 1.Se considera f : (1,∞)→ (1,∞), f(x) = alogxb

a)Sa se studieze monotonia functiei f .b)Sa se calculeze f(f(x)) si sa se arate ca f este bijectiva.c)Sa se rezolve in multimea (1,∞) ecuatia2logx2009 + 3logx2009 = x− 2004.

Dan Marinache ,Calarasi

3.Fie n ∈ N,n ≥ 2 si f : D → R,D ⊂ R cu f(x) = n√

1− n+1√x+ 1 +

n+1√

n√x− 1.Sa se determine domeniul maxim de definitie al functiei f.

4.Fie a, b ∈ C, a ∕= 0 si functia f : C → C, f(z) = az + b. Sa se arateca daca punctele de afixe z1, z2, z3 sunt coliniare atunci si punctele de afixef(z1), f(z2), f(z3)

G.M. 10 /2008

2.1.5 Cluj

1.Fie a, b, c ∈ (0, 1), 0 < a < b < c < 1. Sa se compare numerele :A = logba+ logcb+ logac si A = logab+ logbc+ logca

Prof. Ilie Diaconu

2.Fie � o radacina nereala de ordinul 3 a unitatii si z = (1 + �)(1 + �2)...(1 +�2008).

Page 30: olimpiade2009

CAPITOLUL 2. CLASA X 30

Determinati ∣z∣ si argz .

3.a)Calculati suma : (1− 14 ) + (2− 1

4 ) + (n− 14 ) .

b)Rezolvati n multimea numerelor reale ecuatia :√x1 − 1 +

√x2 − 2 +

√xn − n+ n(2n+1)

4 = x1 + x2 + ...xn

Prof. Ilie Diaconu

4.Se considera functia f : (0,∞)→ R cu proprietatea ca :f(xy) = f(x) + f(y),∀x, y ∈ (0,∞) .a)Aratati ca f(1) = 0b)Aratati ca daca 1 este singura solutie a ecuatiei f(x) = 0,atunci f este injectiva.

2.1.6 Dolj

1.Fie n ≥ 1 un numar natural dat.Sa se rezolve in multimea numerelor intregiecuatia : xn + 25 = x2 + 5n

2.Fie z si z’ doua numere complexe iar u una dintre valorile lui√zz′.Aratati

ca : ∣z∣+ ∣z′∣ = ∣ z+z′

2 − u∣+ ∣ z+z′

2 + u∣.

3.Sa se rezolve sistemul:i)2logsin2xx < 1ii)y2 + y⋅

√5− x− x

2(1+ 4√

(log2cos2�x)= 0

4.Fie functia f : I → R ( I interval ) , astfel incat:

f(a+b2 ) ≤ f(a)+f(b)

2 , pentru orice a, b ∈ I.a)Sa se arate ca pentru orice a, b ∈ I si orice p, q ∈ N∗ are loc relatia :

f(pa+qpp+q ) ≤ pf(a)+qf(b)

p+q

b)Sa se arate ca functia arcsin verifica relatia data pe (0, 1) .

2.1.7 Galati

1.Sa se demonstreze inegalitatea:(sin2a+ sina⋅ cosa)sina(cos2a+ cosa⋅ cosa)cosa ≤ 1,∀a ∈ (0, �2 )

Page 31: olimpiade2009

CAPITOLUL 2. CLASA X 31

2.Sa se rezolve in R ecuatia :log�

2(arcsin(x− [x]) + log�

2(arccos(x− [x]) = 2

log�4

(arctgex+arcctge[x])

Vasile Duma

3.Sa se demonstreze inegalitatea :x3

y3 + y3

z3 + z3

x3 ≥ 12 (x

2

yz + y2

zx + z2

xy ) + 32 ,∀x, y, z ∈ (0,∞) .

Dumitru si Rodica Balan

4.Sa se rezolve in C ecuatia :∣z∣+ ∣z − 1∣+ ∣z − 2∣+ ...∣z − 2009∣ = 10052

Prof. Totolici Mihai

2.1.8 Gorj

Subiectul 1.Fie n ≥ 2, n ∈ N si z1, z2, ..., zn ∈ C. Sa se demonstreze ca∣z1∣2 + ∣z2∣2 + ...+ ∣zn∣2 ≥ Im(z1z2 + z2z2 + ...+ zn−1zn + znz1).

Subiectul 2.a) Fie � = cos 2�7 +i sin 2�

7 . Sa se arate ca 1+�+�2 + ...+�6 = 0si ca pentru orice z ∈ C avem ∣z − 1∣+ ∣z − 2�∣+ ...+ ∣z − 7�6∣ ≥ 28.b) Sa se afle o relatie intre � si a astfel incat locul geometric al imaginilor

numerelor complexe z care verifica relatia az+bz + az+b

z = �, a, b, � ∈ R∗ este oparabola sau o pereche de drepte paralele.

Subiectul 3.a) Sa se rezolve ecuatia : 3√

41− 2x+√x− 6 = 4.

b) Sa se rezolve sistemele:⎧⎨⎩27−x + 16−y =

5

6−16y + 27x ≥ 1

log27 x− log16 y ≤1

6

si {aln x = bln y

(bx)ln b = (ay)ln a

Page 32: olimpiade2009

CAPITOLUL 2. CLASA X 32

unde a, b > 1 .

Subiectul 4.Se considera un pentagon ABCDE inscris intr-un cerc . Sase arate ca ortocentrele tringhiurilor ABC,ACD,ADE si ABE sunt varfurileunui paralelogram.

2.1.9 Hunedoara

1.a)Sa se arate ca :x3 + y3 + z3 − 3xyz = (x+ y + z)(x2 + x2 + x2 − xy − yz − zx).b)Fie a, b, c ∈ (0,∞).Aratati ca a,b,c sunt in progresie geometrica daca si numaidacaln3(a

2

bc ) + ln3( b2

ac ) + ln3( c2

ab ) = 0.

2.Fie punctul P situat in planul patrulaterului convex ABCD, Q simetricullui P fata de mijlocul laturii [AB], R simetricul lui Q fatta de mijlocul laturii[BC], S simetricul lui R fata de mijlocul laturii [CD], T simetricul lui S fata demijlocul laturii [DA]. Sa se arate ca T = P.

3.Sa se determine z ∈ C∗,stiind ca r ∈ (2,∞) si ∣z +1

z∣ = ∣z3 +

1

z3∣ = r.

4.Fie punctele A si B diferite , M ∈ (AB) si punctele C, D, E in acelasi plancu A si B, astfel incat perechile (C,E) , (D,E) sunt de o parte si de alta a drepteiAB, iar triunghiurile CAM, DMB si EBA sunt asemenea.a) Aratati ca centrul de greutate, G, al triunghiului CDE se afla pe (AB).b) Sa afle locul geometric al lui G cand M este variabil pe (AB).

2.1.10 Iasi

1.Fie z1, z2 ∈ C.Aratati ca daca ∣z1∣ = ∣z1∣ si z1 + z2 > 0 atunci z1⋅ z2 > 0

2.Determinati a ∈ R astfel incat inegalitatea log a−1a+2

(x2−2x+3) ≥ 1 sa aiba

loc pentru orice x real.

3.Rezolvati in R ecuatia [ 3√x− 3] + [ 3

√x− 2] = 1.

Page 33: olimpiade2009

CAPITOLUL 2. CLASA X 33

4.Intr-un turneu de sah, fiecare dintre jucatori disputa cate o partida cufiecare dintre ceilalti.Stiind ca nicio partida nu se termina remiza si ca toti par-ticipantii obtin cel putin cate o victorie, demonstrati ca exista un grup de treisahisti A, B, C astfel inct A il invinge pe B, B il nvinge pe C si C il nvinge pe A .

2.1.11 Prahova

1.Sa se rezolve in R+ ×R+sistemul:i)x2009 − y2009 = log2009

xy

ii)x2 − xy + y2 = 2009.

Coman Vasile

2.a)Fie x, y, z, t ∈ (1,∞) astfel incat x3 > yzt.Sa se demonstreze ca :lg3x > lg(y)lg(z)lg(t)b)Sa se generalizeze punctul a) .

Petre Nachila

3.Fie m,n ∈ N∗,m > n si functia f : R → R, f(x) = n2mx −m2nx.Aratatica:a)f(x) ≤ f(0),∀x ∈ Rb)Functia f este strict descrescatoare pe (−∞, 0] si strict crescatoare pe [0,∞).

Cezar Apostolescu

4.Fie z ∈ C∗ astfel incat [log2∣z∣] + [log2∣z + 1∣] = 0, unde [x] reprezintapartea intreaga a numarului real x.a)Determinati o solutie a ecuatiei.b)Determinati valorile posibile pentru[∣z∣].c)Sa se arate ca ∣Im(z)∣ ≤

√152

Emil Vasile

Page 34: olimpiade2009

CAPITOLUL 2. CLASA X 34

2.1.12 Sibiu

1.a)Rezolvati in R ecuatia:√lgx+

√lgy − 1 +

√lgz − 2 = lg

√xyz

Alina Tintea

b)Rezolvati in R×R sistemul:

i)2132 +2y2 + 2

2x2 = 2x+ 4

y

ii)2132 +2x2

+ 22y2 = 2y+ 4

x

Doriana Dorca

2.Determinati functiil f : R→ [−1, 1] cu proprietatea ca :3√

1 + f(x) + 3√

1− f(x) ≤ 3√

2,∀x ∈ R

Mioara Macrea,Ioana Aleman

3.a) Demonstrati ca ∣z1∣ − ∣z2∣ ≤ (z1 + z2).b) Daca z ∈ C∗ astfel incat ∣z + 1

z ∣ = 1,aratati ca:

2∣z∣4 − 4∣z∣2 + 8Re2(z) ≤ 1 +√

5 .

Ileana Otoiu

4.Daca a, b, c ∈ (0,∞) si a+ b+ c = s , demonstrati ca :a+b√

(s−a)(s−b)+ b+c√

(s−b)(s−c)+ a+c√

(s−a)(s−c)≥ 3

Gazeta Matematica

2.1.13 Timis

1.Determinati 2 ≤ n ∈ N pentru care [lg2 + lg3 + ...+ lgn] = n2

Prof. Budai Stefan ,Prof. Aurel Dobosan

2.Fie 2 ≤ n ∈ N . Daca z1, z2, ..., zn sunt n numere complexe cu proprietateaca: ∣z1∣ = 4∣z2∣ = ... = n2∣zn∣,aratati ca z1+z2+...+zn = 0⇔ z1 = z2 = zn = 0.

Page 35: olimpiade2009

CAPITOLUL 2. CLASA X 35

Mihai Chis

3.Fie u,v,w,z patru numere complexe avand acelasi modul, astfel incat z=u+w+v.Sase calculeze E = (u+ v)(v + w)(w + u).

Prof. Valerica Negru

4.Fie a,b,c,d patru numere naturale astfel incat c+ d ∕= 0.Aratati ca functiaf : N∗ → N∗,definita prin f(n) = [an+b

cn+d ] ∀n ∈ N∗ este bijectiva daca si numaidaca a = d > b > c = 0.

Mihai Chis

2.1.14 Vaslui

1.Sa se determine numerele complexe z1, z2, z3 de acelasi modul, cu proprietateaca z1 + z2 + z3 = z1z2z3 = 1

G.M. 12/2008

2.Sa se rezolve ecuatia: 5x√

12 + x+ 7x

√12 − x =

√25x + 49x

Gvan Traian

3.Fie a, b ∈ (0,∞) diferite de 1.Sa se arate ca:

(ln2a+ ln2b)( 1ln2a + 1

ln2b ) ≥ln2(ab)lna⋅lnb

Brasoveanu Diana

4.Fie z1, z2, z3 ∈ C∗ distincte astfel incat:∣z3 − z1∣+ ∣z2 − z3∣ = ∣z1 − z2∣.Sase arate ca:∣z1∣2∣z2 − z3∣+ ∣z2∣2∣z3 − z1∣ = ∣z3∣2∣z1 − z2∣+ ∣z3 − z1∣∣z2 − z3∣∣z1 − z2∣

Cozma Constantin

Page 36: olimpiade2009

CAPITOLUL 2. CLASA X 36

2.1.15 Valcea

1.Sa se rezolve sistemul :i)xy = yx

ii)xx = yy

2.Fie functia f : (0,∞)→ R astfel incat :a)f(xy)=f(x)+f(y)b)singura solutie a ecuatie f(x)=0 este 1.Sa se arate ca f este injectiva .

3.Fie numerele complexe z1, z2, z3, ..., zn ∈ C∗ de modul 1.

Sa se arate ca (z1+z2)(z2+z3)...(z1+zn)z1z2z3zn

este numar real .

4.Sa se rezolve in R×R ecuatia sin(2x) + tg2y + ctg2y = 1 .

G.M. Nr. 1 / 2007

2.2 Olimpiada judeteana

Subiectul 1.Fie f, g : R→ R doua functii cu proprietatea f(g(x)) = g(f(x)) =−x pentru orice x ∈ R.

a) Sa se arate ca f si g sunt functii impare.

b) Dati un exemplu de functii cu proprietatea din enunt.

Bogdan Enescu,Buzau

Subiectul 2.Sa se determine numerele complexe z1, z2, z3 de acelasi modul,cuproprietatea ca z1 + z2 + z3 = z1z2z3 = 1.

Gazeta Matematica

Subiectul 3.Se considera multimile A = {x ∈ R∣3x = x + 2} si B = {x ∈R∣log3(x+ 2) + log2(3x − x) = 3x − 1}.Sa se arate ca:

Page 37: olimpiade2009

CAPITOLUL 2. CLASA X 37

a)A ⊂ B

b) B ∕⊂ Q si B ∕⊂ R∖Q

Cristinel Mortici,Targoviste

Subiectul 4.a) Fie z1, z2, z3 numere complexe nenule de acelasi modul astfelincat z1+z2+z3 = 0.Sa se arate ca punctele A1(z1), A2(z2), A3(z3) sunt varfurileunui triunghi echilateral.

b) Fie n ≥ 3 un numar natural si fie Un = {z ∈ C∣zn = 1}multimearadacinilor de ordin n ale unitatii.Sa se determine numarul maxim de elementeale unei multimi A ⊂ Un cu proprietatea ca z1 + z2 + z3 ∕= 0 pentru oricez1, z2, z3 ∈ A.

Vasile Pop,Cluj

2.3 Olimpiada nationala

Subiectul 1.a) Aratati ca, daca x, y ∈ (1,∞) si xy = yx atunci x = y sau

exista m ∈ (0,∞)∖{1} astfel incat x = m1

m−1 , y = mmm−1 .

b) Rezolvati in multimea (1,∞) ecuatia cu doua necunoscute xy + xxy−1

=

yx + yyx−1

.

Cristinel Mortici

Subiectul 2.Fie a ∈ R, a ∈ [2+√

2, 4].Determinati minimul expresiei ∣z2−az+a∣, cand z ∈ C si ∣z∣ ≤ 1.

Wladimir Boskoff si Viorel Vajiitu

Subiectul 3.Determinati toate functiile f : R→ R care verifica relatia f(x3 +y3) = xf(x2) + yf(y2),∨x, y ∈ R.

Page 38: olimpiade2009

CAPITOLUL 2. CLASA X 38

Vasile Pop

Subiectul 4.Vom spune ca un numar natural n ≥ 4 este neobisnuit daca sepoate aseza cate un numar real in fiecare din cele n2 patrate unitate ale unuipatrat P de latura n , astfel incat suma celor 9 numere din orice patrat de 3×3continut de P sa fie strict negativa, iar suma celor 16 numere din orice patrat4×4 continut de P sa fie strict pozitiva.Determinati toate numerele neobisnuite.

Mihai Baluna

2.4 Concursuri interjudetene

2.4.1 Concursul ”UNIREA”

1.Rezolvati ecuatia:[5x3] +

{2x2}

= 4−√

5

2.Se considera m ∈ R,m ≥ 1 si functia :f : [m,∞)→ (1,∞), f(x) = x2 − 2mx+m2 + 1.a)Aratati ca functia este o bijectieb)Rezolvati ecuatia :x2 − 2mx+m2 + 1 = m+

√x+ 1

3.Fie f : R→ R o functie cu proprietatea :f(x2 − y2) = (x+ y)(f(x)− f(y),∀x, y ∈ R .Demonstrati ca f(2009x) = 2009f(x).

4.Fie a, b > 0,astfel incat√a = 1 +

√b. Demonstrati ca:

a4+b4

2 < a2b2 + (a+ b)3.

2.4.2 Concursul ”Alexandru Myller”

1.Determinati valorile lui n ∈ N pentru care 41 are un multiplu de formaa 00...0︸ ︷︷ ︸n cifre

b, unde a,b sunt cifre zecimale nenule.

Page 39: olimpiade2009

CAPITOLUL 2. CLASA X 39

Mihai Baluna

2.Fie ABC un triunghi si k ∈ R, k ∕= 1 si punctele M ∈ BC,N ∈ CA,P ∈AB astfel incat MB

MC = NBNA = PA

MB = k.Se stie ca AM = BN=CP. Demonstratica triunghiul ABC este echilateral.

I. V. Maftei

3.Pentru x, y ∈ R definim f(x,y)=distanta de la ∣x − y∣ la cel mai apropiatintreg, iar pentru o multime finita M ⊂ [0, 1] definims(M) =x,y∈M,x<y

∑f(x, y) .Determinati valoarea maxima pentru s(M) cand

M parcurge familia multimilor de 4 elemente.

Dinu Serbanescu

4.Fie a un numar real. Aratati ca sirul cu termen general an = (−1)[na], n ≥0 etse periodic daca si numai daca e este rational .

Gheorghe Iurea

2.4.3 Concursul ”Victor Valcovici”

1.Sa se rezolve ecuatia:16x + 81x + 256x + 625x = 24x + 108x + 240x + 320x.

Marius Damian, Braila

2.Fie � o radacina nereala de ordinul 3 a unitatii.Sa se arate ca daca z esteun numar complex cu proprietatile ∣z − 1∣ ≤ 1 si ∣z − �∣ ≤ 1,atunci ∣z∣ ≤ 1.

***

3.Sa se arate ca ecuatia (x−4)log45 = x−1 + (2x−5)log64, x > 4, are solutieunica.

Carmen si Viorel Botea, Braila

Page 40: olimpiade2009

CAPITOLUL 2. CLASA X 40

2.4.4 Concursul ”Grigore Moisil”

Editia a V-a,30 ianuarie - 1 februarie 2009

1.Rezolvati in numere complexe sistemul:i)z + w = 4ii)(z2 + w2)(z3 + w3) = 280.

***

2.Se considera functia f : [−1, 1]→ R definita prin f(x) =√x+ 1+

√x− 1.

a)Demonstrati ca f este crescatoare pe [−1, 0] si descrescatoare pe [0, 1]b)Rezolvati ecuatiaf(x) =

√2 + log2x

Cristinel Mortici

3.Fie a, b ∈ (0,∞) cu proprietatea ca ab = 10√

2.Aratati ca lg2a + lg2b ≥1.Cand are loc egalitatea?

Revista Minus

4.Fie a, b ∈ C, a ∕= 0 si functia f : C → C, f(z) = az + b. Sa se arateca daca punctele de afixe z1, z2, z3 sunt coliniare atunci si punctele de afixef(z1), f(z2), f(z3)

2.4.5 Concursul ”Laurentiu Duican”

1.Sa se arate ca pentru orice a, b, c ∈ C are loc inegalitatea:∣a∣+ ∣b∣+ ∣c∣ ≥ Re(ab) +Re(bc) +Re(ca) + 1

6 (∣a− b∣+ ∣b− c∣+ ∣c− a∣)2

2.Sa se demonstreze:√ab ≤

n√

(a+nb)(2a+(n−1)b)...(na+b)

n+1 ≤ a+b2 ,∀a, b > 0,unde n ∈ N,n ≥ 2.

3.Fie a un numar irational si b un numar real astfel incat :b ≥ a2−2a

4 .Demonstrati ca functia f : Q→ R definita prin:f(x) = (x2 + ax+ b)2 + a(ax+ b),∀x ∈ Q, este injectiva.

Page 41: olimpiade2009

CAPITOLUL 2. CLASA X 41

4.Fie [ABCD] un tetraedru echifacial, iar M,N,P,Q picioarele inaltimilorsale.Sa se arate ca tetraedrele [ABCD] si [MNPQ] au acelasi centru de greu-tate.

2.4.6 Concursul ”Gheorghe Lazar”

1.Sa se rezolve ecuatia: ( 13 )x+1 + ( 1

12 )x − 3(√

312 )x = 3.

G. M., 12/2008

2.Fie M ={z ∈ C −R∣z2 − z + 1 ∕= 0, z4 + 5z2 + 1 ∕= 0

}si functia

f : M → C data de relatia f(z) = (z2+z+1)2+(z2−z+1)2+z4+z2+1(z2+z+1)2+(z2−z+1)2−z4−z2−1 . Sa se arate ca

daca f(z) ∈ R atunci Re(z) = 0 sau ∣z∣ = 1.

Emil C. Popa, Sibiu

3.Se da triunghiul ABC in care are loc relatia:2 + sinA+ sin2B − 2

√2(�4 +B)2

sinA2 ≤ 0.

Sa se arate ca triunghiul ABC este dreptunghic.

Liana Agnola, Sibiu

4.Fie n numar natural , n ≤ 2. Sa se arate ca n! se divide la (1 + 22 + ...n2)daca si numai daca n+ 1 si 2n+ 1 sunt numere compuse.

Dumitru Acu, Sibiu

2.4.7 Concursul ”Trident”

1.Fie ABC un triunghi inscris intr-un cerc de raza 1 cu A(zA), B(zB), C(zC) si∑ciclic

(b + c + ka)zbzc = k.Daca k=0,sau k=1,sau k=-2 atunci triunghiul ABC

este echilateral.

Gheorghe Alexe

Page 42: olimpiade2009

CAPITOLUL 2. CLASA X 42

2.Fie a,b,c numere reale pozitive.Demonstrati ca daca a2 + b2 + c2 + abc = 4atunci a+ b+ c ≤ 3.

Olimpiada, Iran 2003

3.Sa se arate ca 39 este singurul numar natural n pentru care logn−7(n +2009) ∈ Q .

Dan Negulescu

Page 43: olimpiade2009

Capitolul 3

Clasa XI

3.1 Etapa locala

3.1.1 Arges

Subiectul 1

Se da functia f : (0,∞) → R , f(x) =1

x. Pe graficul functiei f se iau punctele

An(n, f(n)) , n ∈ N∗. Se noteaza cu sn aria triunghiului AnAn+1An+2 , n ∈ N∗.

Sa se demonstreze ca sirul (Sn) , Sn =

n∑k=1

sk este convergent.

Marian Teler, profesor Costesti

Subiecul 2a) Se considera numerele naturale prime p si q si A,B ∈ Mn(R) doua matricicare verifica relatia: pqAB = pA+ qBAratati ca AB = BA.b) Fie functiile f, g : (o, e2)→ R, f(x) =

√x pentru x ∈ (0, 1) si f(x) = 1 +

√x

pentru x ∈ [1, e2) , iar g(x) =f(ln(1 + x2))

ln(1 + f(x2)).

Sa se determine multimea punctelor de discontinuitate a lui g.

* * *

43

Page 44: olimpiade2009

CAPITOLUL 3. CLASA XI 44

Subiectul 3

Sa se rezolve ecuatia matriciala: Xn =

⎛⎝ 1 1 10 1 10 0 1

⎞⎠, X ∈ M3(R) , n ∈ N ,

impar.

* * *

Subiectul 4Fie A ∈ M2(R) cu proprietatile det(A− 3I2) = 4 si det(A+ 2I2) = 9. Sa sedetermine (A− I2)

npentru n ∈ N si n ≥ 2.

ISJ Arges

3.1.2 Braila

Subiectul 1

Fie matricea A =

(a 11 a

), a ∈ R−{1}. Calculati det (

n∑k=1

Ak) , n ∈ N∗.

* * *

Subiectul 2Fie matricile A,B,C ∈Mn(C) astfel incat ABC = In.a) Sa se demonstreze ca A,B,C sunt inversabile.b) Daca matricea U = In+A+AB este inversabila , sa se denostreze ca matricileV = In + B + BC si W = In + C + CA sunt inversabile si suma inverselor loreste In.

Emilian Runceanu

Subiectul 3

Fie sirul (xn)n≥1 , x1 = a , x2 = b , a > b > 0 si

(xnxn+1

)2

− 1 =xn+2+(−1)n+1

xn+2,

Page 45: olimpiade2009

CAPITOLUL 3. CLASA XI 45

∀n ≥ 1. Sa se studieze convergenta sirului.

Nicolae Stanica

Subiectul 4Fie q > 1 si functia f : (0,∞)→ (0,∞) , cu proprietateaf(x+ 1)− f(x) = r ∈ R,∀x ∈ R.

Demonstrati ca sirul (an)n∈N∗ , an =

n∑k=1

f(k)

qk, n ∈ N∗ este convergent si

calculati limita lui.

Marius Damian

3.1.3 Caras-Severin

Subiectul 1a) Sa se dea exemplu de matrice A ∈M2(R), A ∕= O2, A ∕= I2 pentru care existap ∈ N, p ≥ 2 astfel incat Ap = A.b) Sa se arate ca daca n, p ∈ N,n, p ≥ 2 si A ∈ Mn(R) astfel incat Ap+1 = Aatunci rang(A) + rang(In −Ap) = n.

RMCS 24 , articol

Subiectul 2Un determinant D de ordinul 3 are elementele de pe diagonala principala egale

cu1

2, iar suma elementelor de pe fiecare linie si fiecare coloana este egala cu 1.

Sa se arate ca: D > 0.

* * *

Subiectul 3Fie x1 ∈ (0, 1) si sirul (xn)x∈N∗ definit prin xn+1 = xn

3 − xn2 + 1,∀n ≥ 1. Sase arate ca sirul (xn)n∈N∗ este convergent si sa se calculeze lim

n→∞(x1 ⋅x2 ⋅ ... ⋅xn).

Page 46: olimpiade2009

CAPITOLUL 3. CLASA XI 46

* * *

Subiectul 4

Sa se calculeze limx→0

(3x + 5x

2

)ctgx.

GM 10/2007

3.1.4 Cluj

Subiectul 1Se considera matricele A,B ∈ Mn(R) cu proprietatea ca B − A = In. Ce cpn-ditie trebuie sa indeplineasca matricea A astfel incat , oricare ar fi k ∈ N∗ , sa

aibe loc egalitatea: B +B2 +B3 + ...+Bk = kIn +k(k + 1)

2A ?

Subiectul 2Fie a, b, c, d ∈ (0,∞) astfel incat a2 + b2 = c2 + d2 si matricea

A =

⎛⎜⎜⎝a 0 0 b0 c d 00 −d c 0−b 0 0 a

⎞⎟⎟⎠.

a) Daca suma elementelor matricei A2 este 0 atunci b = c.b) Sa se calculeze An unde n ∈ N∗.

Ilie Diaconu

Subiectul 3Se considera functia f : R → R cu proprietatea ca ∣f(x) − x ⋅ sinx∣ ≤ ∣x3∣ ,

∀x ∈ R. Calculati: limx→0

f(x)

x2.

Subiectul 4Sa se calculeze limita sirului:

Page 47: olimpiade2009

CAPITOLUL 3. CLASA XI 47

(xn)n≥1 definit prin xn =

n∑k=1

1

n+3

√(k + 1)

2(k2 + 1)

2, ∀n ≥ 1.

Liviu Tivadar

3.1.5 Constanta

Subiectul 1Fie A o matrice de ordin n cu elemente reale , avand proprietatea ca A2007 +A2008 +A2009 = On. Notam cu B = In+A+A2. Sa se demonstreze ca matriceaIn −AB este inversabila.

Subiectul 2i) Fie A,B ∈M2n+1(C) cu A2 −B2 = I2n+1. Aratati ca det(AB −BA) = 0.ii) Gasiti A,B ∈M2(C) cu A2 −B2 = I2 , dar det(AB −BA) ∕= 0.

GMB

Subiectul 3Sa se calculeze lim

n→∞n(

k√nk + ank−1 +

k√nk − ank−1−2n) , unde k ∈ N∗, k ≥ 2

si a ∈ R+∗.

Dorin Arventiev

Subiectul 4

Fie sirul (an)n≥1 cu a1 = 1 si an+1 =an

2 + 2

n+ 1,∀n ∈ N .

Sa se calculeze: limn→∞

a1 + a2 + ...+ an

1 +1

2+

1

3+ ...+

1

n

.

GMB

Page 48: olimpiade2009

CAPITOLUL 3. CLASA XI 48

3.1.6 Dolj

Subiectul 1Fie a, b, c ∈ R care verifica a2 + b2 + c2 = 1 si matricea

A =

⎛⎝ a b cc a bb c a

⎞⎠ .

Aratati ca ∣detA∣ ≤ 1.

* * *

Subiectul 2Definim sirul (un)n∈N prin u0 = u1 = u2 = 1 si apoi prin conditia

det

(un un+1

un+2 un+3

)= n!,∀n ≥ 0.

Aratati ca un ∈ Z , pentru orice numar natural n.

* * *

Subiectul 3Fie sirul (xn)n≥1 astfel incat x1 = 1 si (n+ 1)(xn+1 − xn) ≥ 1 + xn , ∀n ∈ N∗.

Aratati ca (xn)n≥1 nu este marginit.

* * *

Subiectul 4Sa se calculeze:

limn→∞

(

∑1≤i ∕=j ∕=k ∕=i≤n

1

i

1

j

1

k

(lnn)2 − lnn).

Page 49: olimpiade2009

CAPITOLUL 3. CLASA XI 49

Tatiana Ignat , Craiova

3.1.7 Galati

Subiectul 1Se considera matricile A,B ∈M2(R) care indeplinesc conditiile:(i). det(A+ 2008B) = det(2008A+B) ;(ii). det(A+ 2009B) = det(2009A+B).Sa se demonstreze ca detA = detB.

Totolici Mihai

Subiectul 2Fie a, b, c ∈ R, a ∕= 0. Sa se arate ca det(aX2 + bX + cI2) ≥ 0,∀X ∈ M2(R)daca si numai daca b2 − 4ac ≤ 0.

Conf. dr. Jenica Cringanu

Subiectul 3Fie � ∈ R. Sa se arate ca sirul cu termenul general xn = {n ⋅ �}, n ∈ N , esteconvergent daca si numai daca � ∈ Z. (S-a notat cu {a} partea fractionara anumarului a).

Conf. dr. Jenica Cringanu

Subiectul 4

Sa se calculeze limx→0

tg2008(x)− arctg2008(x)

x2009.

Dumitru si Rodica Balan

Page 50: olimpiade2009

CAPITOLUL 3. CLASA XI 50

3.1.8 Gorj

Subiectul 1.Fie a, b, c ∈ R∗ , t o radacina a ecuatiei x2 + x + 1 = 0 functia

f : R → R, f(x) = a + bx + cx2 si matricele A =

⎛⎝ 1 1 11 t t2

1 t2 t4

⎞⎠ , B =⎛⎝ a b cb c ac a b

⎞⎠. Sa se arate ca :

a) detA = t(t+ 1)(t− 1)3.b) detB = −f(1)f(t)f(t2).

Subiectul 2.Fie sirul (an)n≥1 definit astfel a1 = 1 sin

an+1− n+ 1

an=

n(n+ 1),∀n ≥ 1 . Sa se calculeze limn→∞

1

n3

n∑k=1

1

ln(1 + ak).

Subiectul 3.a) Se consideraA ∈M3(C) avand proprietateaAX = XA, ∀X ∈M3(C). Sa se arate ca A = aI3 , unde a ∈ C.b) Daca A,B ∈Mn(R) astfel ca BA = −A si AB = −B sa se arate ca matriceaA+B + In este inversabila.

Subiectul 4.Fie (An)n≥1 un sir de numere reale cu proprietatea ca sirul(bn)n≥1 este marginit , unde bn = n2(an− sinn)2.Aratati ca sirul an este diver-gent.

3.1.9 Hunedoara

Subiectul 1a) Se considera o matriceA ∈M2(R) nenula pentru care Tr(A2008) = (Tr(A1004))2

. Sa se demonstreze ca rang(A) = 1.b) Sa se dea exemplu de 2 matriciA1, A2 ∈M2(R) nenule pentru care Tr(Ai

2008) =(Tr(Ai

1004))2, i = 1, 2 si rang(A1 +A2) = 2.

Subiectul 2Fie n ∈ N,n ≥ 2 si o permutare � ∈ Sn. Se considera multimea A� ={∣i− �(i)∣∣i ∈ {1, 2, 3, ..., n}}.a) Sa se arate ca

∪�∈Sn A� = {0, 1, 2, 3, ..., n− 1}.

b) Daca n = 4k+2 sau n = 4k+3, k ∈ N , sa se arate ca n−1 ≥ card(A�),∀� ∈

Page 51: olimpiade2009

CAPITOLUL 3. CLASA XI 51

Sn.

Subiectul 3Pentru n ∈ N∗ , se considera sirul de numere pozitive (xn)n≥1 si sirul (yn)n≥1

definit astfel yn =

√x1 +

√x2 +

√...+

√xn. Se cere:

a) Daca xn = 1,∀n ∈ N∗ , atunci (yn)n≥1 este convergent.b) Daca xn ≤ e2n ,∀n ∈ N∗ atunci (yn)n≥1 este convergent.c) Daca exsita k > 2 pentru care xn ≥ ek

n

,∀n ∈ N∗ , atunci (yn)n≥1 estedivergent.

Subiectul 4

Fie sirurile (xn)n≥1 si (yn)n≥1 definite prin x0, y0 > 0 , xn+1 = yn +1

ynsi

yn+1 = xn +1

xn,∀n ∈ N .

a) Sa se arate ca limn→∞

xn = limn→∞

yn =∞.

b) Sa se arate ca limn→∞

xn√2n

= limn→∞

yn√2n

= 1.

3.1.10 Ilfov

Subiectul 1Sa se determine numerele reale a, b, c astfel incat:

limx→∞

x(√ax2 + bx+ c− 2x− 3) = 2009.

Subiectul 2

Fie a ∈ C si matricea A ∈ M2(C), A =

(a 10 a

). Daca se noteaza cu

B = A+A2 +A3 + ...+An, n ∈ N∗ , sa se determine matricea B.

Subiectul 3

Sa se arate ca sirul (an)n≥1 , dat de relatia an =6an+1 + an−1

5cu a1 =

1

2si

a2 =1

3este convergent si sa se calculeze limita sa.

Page 52: olimpiade2009

CAPITOLUL 3. CLASA XI 52

Subiectul 4a) Fie A ∈ M2(C) cu proprietatea ca Tr(A) = Tr(A2) = 0. Aratati cadet(A2) = 0.b) Fie A ∈ M2(R) cu proprietatea ca Tr(A) ∕= 0. Sa se arate ca matriceaB ∈M2(R) comuta cu matricea A daca si numai daca comuta cu A2.

3.1.11 Prahova

Subiectul 1

Se considera sirul (an)n≥1 cu a1 =√

10, a2 =√

20 si a[an] =3an + 2[an] + 11

5,∀n ≥

1 unde [x] = partea intreaga a numarului real x.a) Calculati a3 si a4.b) Sa se studieze convergenta sirului (an)n≥1.

c) Sa se determine limn→∞

ann

.

Militaru Claudiu

Subiectul 2Fie f1, f2 : R⇒ R functi periodice de perioade T1 , respectiv T2 astfel incat:

limx→0

f1(x)

x= l1 ∈ R si lim

x→0

f2(x)

x= l2 ∈ R∗.

a) Sa se arate ca (3 +√

7)n + (3−√

7)n = k ∈ Z.

b) Sa se calculeze limn→∞

f1((3 +√

7)n ⋅ T1)

f2((2 +√

2)n ⋅ T2).

Popa Dumitru

Subiectul 3Fie n ∈ N∗ si A,B ∈Mn(C) inversabile. Sa se arate ca:a) Daca AB−1 = In +BA−1 , atunci det(A−B) ∕= 0.b) Daca BA−1 = In +AB−1 , atunci det(A+B) ∕= 0.

Ion Nedelcu

Subiectul 4

Page 53: olimpiade2009

CAPITOLUL 3. CLASA XI 53

Fie matricele X =

(−1 1−1 1

)si Y =

(a 0a 1

), a ∈ C.

a) Demonstrati ca (XY − Y X)2 = I2.b) Fie A,B ∈ M2(C) astfel incat (AB −BA)n = I2, n ∈ N . Demonstrati ca neste par.c) Fie A,B ∈Mn(C) astfel incat (AB −BA)2 = In. Demonstrati ca n este par.

Dorin Vasile

3.1.12 Salaj

Subiectul 1

Calculati limita: L = limx→1

(1−√x)(1− 3

√x)...(1− n

√x)

(1− x)n−1.

Subiectul 2

Aratati ca sirul (xn)n≥1 , definit prin xn =

n∑k=1

1

n+ k + k2,∀n ≥ 1 este marginit

si calculati limita limn→∞

xn.

Subiectul 3

Sa se calculeze An, n ∈ N∗ , unde A =

⎛⎝ 0 1 11 0 11 1 0

⎞⎠.

Subiectul 4Fie (an)n≥0 dat astfel: a0 = 1, an+1 = 4

√14 + an, n ∈ N . Sa se calculeze

limn→∞ an.

Page 54: olimpiade2009

CAPITOLUL 3. CLASA XI 54

3.1.13 Sibiu

Subiectul 1

Fie A,X ∈M2(C) , unde A =

(2 00 6

)si A ⋅X = X ⋅A.

a) Aratati ca exista o infinitate de matrici X ∈M2(C) cu proprietatea data.b) Aflati Xn pentru n ∈ N .c) Rezolvati in M2(R) ecuatia matriciala:

X6 −X3 =

(2 00 6

).

d) Cate solutii are ecuatia de la punctul c) , daca X ∈M2(C).

Liana Agnola

Subiectul 2FieA,B ∈Mn(C) cu det(B) = d1 ∕= 0,det(AB−BA) = d2 ∕= 0 si det(AB−1 −B−1A) =d3 ∕= 0. Aratati ca:d2 = (−1)nd1

2d3.

Constantin Tarnu

Subiectul 3Determinati a, b, c ∈ R astfel incat sa fie indeplinita egalitatea:

limx→∞

(√x4 + 2x3 − ax2 − bx− c) = 0.

* * *

Subiectul 4Fie sirul (xn)n≥1 strict descrescator cu proprietatea:xn

4 + 4xn ≥ xn2 + xn3xn+1 + 4,∀n ≥ 1.

Aratati ca sirul (xn)n≥1 este convergent si aflati limita sa.

Gazeta Matematica

Page 55: olimpiade2009

CAPITOLUL 3. CLASA XI 55

3.1.14 Timis

Subiectul 1

Sa se studieze convergenta sirului (xn)n≥1 cu termenul general xn =

⎛⎝1 +sin(2n+ 1)

2n

⎞⎠n

, oricare ar fi n ∈ N∗.

Aurel Dobosan

Subiectul 2

Se da matricea A =

⎛⎝ a 0 0a a 0b a a

⎞⎠ unde a, b ∈ R. Calculati An, n ∈ N∗.

Valerica Negru

Subiectul 3Fie A,B ∈M3(C) doua matrice patrate de dimensiuni 3×3 cu elemente numerecomplexe iar ! ∈ C −R o radacina cubica a unitatii. Aratati ca:det(A+B) + det(A+ !B) + det(A+ !2B) = 3(det(A) + det(B)).

Mihai Chis

Subiectul 4Fie a ∈ R un numar real oarecare fixat , iar (xn)n∈N sirul de numere realedefinit prinx0 = a si xn+1 = e−xn + xn,∀n ∈ N .Aratati ca:a) Sirul (xn)n∈N are limita +∞.

b) limn→∞

ln(n)

xn= 1.

Mihai Chis

Page 56: olimpiade2009

CAPITOLUL 3. CLASA XI 56

3.1.15 Valcea

Subiectul 1a) Sa se arate ca det(X+Y ) + det(X−Y ) = 2(detX+ detY ),∀X,Y ∈M2(C).b) Demonstrati ca pentru orice matrice din A,B ∈M2(C) are loc egalitatea:(AB −BA)2 = (det(AB −BA)− 4 detAdetB) ⋅ I2.

* * *

Subiectul 2Fie � ∈ R. Pentru n ≥ 3 definim:

Δk =

∣∣∣∣∣∣∣∣∣cos(�+

2kΠ

n)− cos� sin(�+

2kΠ

n)− sin�)

cos(�+2(k + 1)Π

n)− cos� sin(�+

2(k + 1)Π

n)− sin�

∣∣∣∣∣∣∣∣∣.Sa se calculeze limita sirului (an)n≥1, an =

n−2∑k=1

∣Δk∣.

Gheorghe Croitoru si Gabriel Popa

Subiectul 3

Fie sirurile: (xn)n≥1 definit prin x1 = a > 1, xn+1 =1

2(xn +

a

xn) si yn =

xn −√a,∀n ≥ 1.

a) Sa se arate ca yn+1 <1

2yn.∀n ≥ 1.

b) Sa se calculeze limn→∞ xn.

* * *

Subiectul 4Fie matricea A ∈M2(C) cu proprietatea ca A2009 = O2. Sa se arate ca A2 = O2.

* * *

Page 57: olimpiade2009

CAPITOLUL 3. CLASA XI 57

3.1.16 Vaslui

Subiectul 1a) Sa se arate ca nu exista matrici patratice de ordin 2 , X , cu elemente numererationale astfel incat X4 = −I2.b) Sa se dea un exemplu de matrice X ∈M2(R) cu proprietatea X4 = −I2.

* * *

Subiectul 2Fie sirul de numere reale (xn)n≥1 definit astfel: x1 = a, a ∈ R, a ∕= 1 ,2xn

2 = xn(3xn+1 − 1) + 3xn+1 + 1,∀n ≥ 1. Calculati limn→∞

xn.

Brasoveanu Valeriu

Subiectul 3Se da determinantul:

dn(z) =

∣∣∣∣∣∣∣∣∣∣∣

zn ⋅ ⋅ ⋅ z3 z2 z + 1zn ⋅ ⋅ ⋅ z3 z2 + 1 zzn ⋅ ⋅ ⋅ z3 + 1 z2 z... ⋅ ⋅ ⋅

......

...zn + 1 ⋅ ⋅ ⋅ z3 z2 z

∣∣∣∣∣∣∣∣∣∣∣.Rezolvati in C ecuatia dn(z) = 0.

* * *

Subiectul 4Calculati:

limx→∞

⎛⎝2

2− xx + 4

4− xx + 6

6− xx + 12

12− xx

⎞⎠x

.

* * *

Page 58: olimpiade2009

CAPITOLUL 3. CLASA XI 58

3.2 Olimpiada Judeteana

Subiectul 1.Fie A,B,C trei matrice de ordin 3,care au elemente numere realesi care indeplinesc conditiile det(A + {B) = det(C + {A) si det(A) = det(B) =det(A).Aratati ca det(A+B) = det(C +A).

Subiectul 2.Fie n ∈ N∗ si o matrice A ∈ Mn(C), A = (apq)1≤p,q≤n,cuproprietatea : aij + ajk + aki = 0,∨i, j, k ∈ {1, 2, ..., n}.Aratati ca rang(A) ≤ 2.

Gazeta Matematica

Subiectul 3.Fie (xn)n≥1 un sir definit de x1 = 2, xn+1 =√xn + 1

n ,∨n ≥1.Aratati ca lim

n→∞xn = 1 si calculati lim

n→∞(xn)n.

Subiectul 4.a) Aratati ca functia F : R → R,F (x) = 2[x] − cos(3�x) areproprietatile : functia F este continua pe R si, pentru orice y ∈ R, ecuatiaF (x) = y are exact trei solutii.

b) Fie k > 0 un numar intreg par.Aratati ca nu exista nici o functie f :R → R cu proprietatile : functia f este continua pe R si,pentru orice y ∈ Imf, ecuatia f(x) = y are exact k solutii.

Benjamin Bogosel,Timisoara

3.3 Olimpiada nationala

Subiectul 1.Fie (tn)n un sir convergent de numere reale , tn ∈ (0, 1) pentruorice n ∈ N si lim

n→∞tn ∈ (0, 1).Definim sirurile (xn)n si (yn)n prin relatiile

xn+1 = tnxn + (1− tn)yn, yn+1 = (1− tn)xn + tnyn,pentru orice n ∈ N si x0, y0

numere reale fixate.a) Sa se arate ca sirurile (xn)n, (yn)n sunt convergente si au aceeasi limita.b) Aratati ca daca lim

n→∞tn ∈ {0, 1} concluzia nu mai este in general adevarata.

Mihai Baluna

Page 59: olimpiade2009

CAPITOLUL 3. CLASA XI 59

Subiectul 2.Fie f : R → R o functie continua cu proprietatea ca pentru oricex ∈ R limita

limℎ→0∣f(x+ ℎ)− f(x)

ℎ∣ exista si este finita.

Aratati ca in orice punct din R f este derivabila sau admite derivate lateralefinite,de acelasi modul si semn contrar.

G.Rene

Subiectul 3.Fie A,B ∈Mn(C) cu AB = BA si detB ∕= 0.a) Aratati ca daca ∣det(A + zB)∣ = 1 pentru orice z ∈ C cu ∣z∣ = 1 , atunciAn = On.b) Ramane adevarata concluzia daca eliminam conditia AB = BA ?

Vasile Pop

Subiectul 4.Fie functiile f, g, ℎ : R → R, unde s este derivabila , g si ℎ suntmonotone iar f ′ = f + g + ℎ.Demonstrati ca multimea punctelor de discontinuitate ale functiei g coincide cumultimea punctelor de discontinuitate ale functiei ℎ.

Cristinel Mortici

3.4 Concursuri interjudetene

3.4.1 IMAR - Bucuresti

Subiectul 1Pe o tabla n × n , cu n2 patratele se aseaza un pion intr-un patratel arbitrar.Pionul presupus a fi intr-o coloana k , poate fi mutat in orice patrat din linia k.Aratati ca exista o secventa de n2 mutari astfel incat orice patrat al tablei estevizitat exact o data si pionul se intoarce la pozitia initiala.

Subiectul 2

Page 60: olimpiade2009

CAPITOLUL 3. CLASA XI 60

Un punct P de coordonate intregi , din planul cartezian xOy , se numeste vizibildaca segmentul OP nu contine in interior puncte de coordonate intregi. Aratatica pentru orice n ∈ N∗ exista un punct vizibil Pn aflat la distanta mai maredecat n de orice alt punct vizibil.

Subiectul 3Doua cercuri 1,2 se taie in punctele X si Y . Prin Y se duce paralela la tangentacomuna cea mai apropiata. Aceasta intersecteaza a doua oara cercurile 1,2 in Arespectiv B. Fie O centrul cercului tangent exterior la 1,2 si tangent interior lacercul circumscris triunghiului AXB. Aratati ca XO este bisectoare in unghiul∕ AXB.

Subiectul 4Aratati ca pentru orice functie f : (0,∞) → (0,∞) exista numere reale strictpozitive x, y astfel incat f(x+ y) < y ⋅ f(f(x)).

3.4.2 Concursul ”Myller”

Subiectul 1

Fie sirul (xn)n≥1 de numere reale definit prin x1 = 1 si xn+1 =

∣∣∣∣xn − n

n+ 1

∣∣∣∣ ,

oricare ar fi n ≥ 1. Sa se arate ca sirul este divergent.

Paul Gerogescu , Gabriel Popa

Subiectul 2a) Sa se determine doua matrice A,B ∈M2(C) cu proprietatea ca A2 +B2 = I2si matricea AB −BA este inversabila.b) Fie n un numar natural impar si matricele A,B ∈ Mn(C) astfel incat A2 +B2 = In. Sa se arate ca det(AB −BA) = 0.

Andrei Ciupan

Subiecrul 3Fie P,Q : R→ R doua functii polinomiale neconstatne cu proprietatea ca existase este finita limx→∞([P (x)]− [Q(x)]). Sa se arate ca exista n ∈ Z astfel incatP (x)−Q(x) = n , oricare ar fi x ∈ R.

Page 61: olimpiade2009

CAPITOLUL 3. CLASA XI 61

Gheorghe Iurea

Subiectul 4Fie A ∈ Mn(R) o matrice simetrica cu elementele de pe diagonala principalaegale cu 1 si cu suma modulelor elementelor de pe fiecare linie mai mica sauegala cu 2. Sa se arate ca det(A) ≤ 1.

Cosmin Pohoata

3.4.3 Concursul ”Unirea”

Subiectul 1Aratati ca , daca A ∈M2(Z) satisface A4 = I2 , atunci A2 = I2 sau A2 = −I2.

Subiectul 2Fie matricele A ∈M3,2(R), B ∈M2,3(R), C = AB si D = BA . Aratati ca daca

C =

⎛⎝ 6 9 3−3 −6 −33 9 6

⎞⎠ , atunci :

1) C2 = 3C si D3 = 3D2 ;2) D = 3I2.

Subiectul 3Fie x0 = 0, y0 > 0 si sirurile (xn)n≥0, (y0)n≥0 definite prinxk = xk−1 − yk−1,∀k ≥ 1yk = yk−1 − exk + exk−1 ,∀k ≥ 1.Sa se arate ca sirurile (xn)n≥0, (y0)n≥0 admit limite si sa se calculeze limitelelor.

Subiectul 4Calculatilimn→∞

n ⋅ sin 1 ⋅ sin 2 ⋅ ... ⋅ sinn

Page 62: olimpiade2009

CAPITOLUL 3. CLASA XI 62

3.4.4 Concursul ”Trident”

Subiectul 1

Calculati limx→0

(sin3x+ sinx+ 1)n − 1

sinnx+ 4 sinx, n ∈ N∗

Alfred Eckstein si Viorel Tudoran

Subiectul 2Fie A ∈M2(Z) si Tr(A) = 1. Sa se arate ca:(detA) ⋅ det(A3 +A+ I2) ≤ 2det4A− 4det3A+ det2A+ detA+ 1.

Viorel Botea

Subiectul 3Fie sirurile de numere reale pozitive (xn)n, (yn)n, (zn)n astfel incat

xn+1 =xn + yn + zn

3, yn+1 = 3

√xn ⋅ yn ⋅ zn, zn+1 =

31

xn+

1

yn+

1

zn

.

Demonstrati ca sirurile au limite si limitele lor sunt finite.

Dan Negulescu

3.4.5 Concursul ”Victor Valcovici”

Subiectul 1Se considera sirurile (xn)n≥1 si (yn)n≥1 date de x1 = 3, y1 = 2, xn+1 =

yn10

+ 3

si yn+1 =xn10

+ 2,∀n ≥ 1. Sa se arate ca (xn)n≥1 si (yn)n≥1 sunt convergente si

sa se determine limitele lor.

Page 63: olimpiade2009

CAPITOLUL 3. CLASA XI 63

Marius Mohonea

Subiectul 2Fie A,B ∈Mn(C) cu proprietatile: 3AB2A = 2B2A2 +A si A2B = In. Aratatica det(AB2 −B2A) = 0.

Andrei Alban

Subiectul 3Sa se determine functiile f : R → R , derivabile pe R , cu derivatele continuepe R , ce satisfac conditiile : ∀x ∈ R, ∣f ′(x)∣ ≤ 7f(x) si f(3) = 0.

Iulian Danielescu

3.4.6 Concursul ”Gheorghe Lazar”

Subiectul 1

Fie 0 < a < b < 2a, b ∕= a

(1 +

√2

2

)iar (xn)n≥0, (yn)n≥0, (zn)n≥1 trei siruri

reale ce indeplinesc conditiile:

i) x0 = b, xn+1 = a+√xn(2a− xn),∀n ≥ 0 ;

ii) (yn)n≥0 este un sir convergent ;

iii) ∀n ≥ 1, zn =1

n⋅n∑k=1

xkyk.

Aratati ca (xn)n≥0 este divergent , iar sirul (zn)n≥1 este convergent.

Doru Isac

Subiectul 2Notam cu C[0, 1] familia functiilor continue f : [0, 1]→ [0, 1] si definim aplicatia

Page 64: olimpiade2009

CAPITOLUL 3. CLASA XI 64

! : C[0, 1]→ C[0, 1] prin !(f)(x) =1

2f(x) + 2x(1− x).

a) Aratati ca ! e bine definita.b) Calculati !n(ℎ) = ! ∘ ! ∘ ... ∘ !(ℎ) de n ori , unde ℎ ≡ 0.c) Daca ℎ ≡ 0 atunci , pentru orice x ∈ [0, 1] sirul (!n(ℎ))n este convergent iarfunctia definita prin f(x) = lim

n→∞!n(ℎ) apartine lui C[0, 1].

Nicolae Secelean

Subiectul 3Fie � ∈ (0, 1) si functia f : R → R cu proprietatea ca exista lim

n→∞(f(x + 1) −

�f(x)) = 0. Sa se arate ca limx→∞

f(x) = 0.

Emil C. Popa

Subiectul 4Fie A,B ∈M2(R) . Sa se arate ca det[(AB +BA)4 − (AB −BA)4] ≥ 0.

G.M. 12/2007

3.4.7 Concursul ”Chindia ”

Subiectul 1

a) Calculati I2 +A+A2 + ...+A2009 , unde A =

⎛⎜⎜⎝cos

2−sin�

2

sin�

2cos

2

⎞⎟⎟⎠b) Determinati multimeaM =

{B ∈M2(R)∣

(2009 90022008 8002

)B −B

(2009 90022008 8002

)= I2

}

Cristinel Mortici , Dinu Teodorescu

Page 65: olimpiade2009

CAPITOLUL 3. CLASA XI 65

Subiectul 2

a) Calculati: limn→∞

(1

2009√n2009 + 1

+1

2009√n2009 + 2

+ ...+1

2009√n2009 + n

).

b) Aratati ca sirul xn = 1 +1

22+

1

32+ ...+

1

n2este convergent , apoi calculati

limita sa , folosind eventual faptul presupus cunoscut ca:

limn→∞

(1 +

1

32+

1

52+ ...+

1

(2n− 1)2

)=�2

8

Cristinel Mortici , Dinu Teodorescu

Subiectul 3

Fie functia f : [0,∞)→ R , data prin f(x) = ex − 1− x− x2

2− x3

6.

Demonstrati ca f(0) = f ′(0) = f”(0) = f”′(0) si deduceti ca:

ex ≥ 1 + x+x2

2+x3

6,∀x ≥ 0.

Cristinel Mortici , Dinu Teodorescu

3.4.8 Concursul ” Nicolae Coculescu”

Subiectul 1Fie n ∈ N,n ≥ 3. Sa se determine numarul permutarilor � ∈ Sn astfel incat�(�(�(1))) = 2.

Dinu Serbanescu

Subiectul 2

Fie A = (aij)1≤i,j≤n o matrice cu elemente reale pozitive astfel incat

n∑j=1

aij ,

pentru orice i = {1, 2, ..., n}.

a) Sa se arate ca pentru orice k ∈ N∗, suma elementelor matricei Ak esten.b) Daca � ∈ C verifica relatia det(A− �In) = 0 , atunci ∣�∣ ≤ 1.

Page 66: olimpiade2009

CAPITOLUL 3. CLASA XI 66

Vasile Pop

Subiectul 3

Fie sirul (an)n≥1 definit prin a1 > 1 si an+1an + 1

an + npentru n ≥ 1.

a) Sa se arate ca (an)n≥1 este strict descrescator si limn→∞

an = 0.

b) Sa se calculeze limn→∞

nan.

Florian Dumitrel

Subiectul 4

Fie (an)n≥1 un sir de numere reale cu proprietatea ca exista limn→∞

an1 + an2

=1

2.

Sa se arate ca limn→∞

an = 1.

Florian Dumitrel

3.4.9 Concursul ”Nicolae Teodorescu”

Ziua1Subiectul 1Pentru n ≥ 2, n ∈ N , notam cu M2,n({−1, 1}) multimea matricelor de tip(2, n) cu elemente in multimea {−1, 1}. Daca A ∈ M2,n({−1, 1}), notamcu mAnumarul minorilor nenuli de ordin 2 ai matricei A. Sa se calculezemax{mA∣ A ∈M2,n({−1, 1})}.

Laura Nastasescu

Subiectul 2

Sa se calculeze : limn→∞

n∑k=1

[ke1/k]

n2.

Ionel Tudor

Page 67: olimpiade2009

CAPITOLUL 3. CLASA XI 67

Subiectul 3Consideram numerele a, b, c, d > 1 astfel incat a+b+c+d = 8. Sa se demonstrezeinegalitatea: aa

2

bb2

cc2

dd2 ≥ 65536.

Viorel Tudoran , Ucu Crisan

Subiectul 4Fie n ≥ 2 natural. Sa se arate ca polinomul f = Xn−1 + Xn−2 + ... + X + 1este ireductibil in Z[X] daca si numai daca n este prim.

Marcel Tena

Ziua2Subiectul 1

Sa se rezolve in numere reale ecuatia:x3

3− x+

6= arcctgx.

Aurel Dobosan

Subiectul 2

Se considera functia f : (0,∞) → R, f(x) = x +1

xpe graficul careia luam

punctele An(n, f(n), n ∈ N∗. Notand cu Sn aria triunghiului AnAn+1An+2, sa

se calculeze limn→∞

n∑k=1

Sk.

Marian Teler

Subiectul 3

Demonstrati ca daca 0 < a, b, c, d, e ≤ 1, atunci1

a+ b+ c+ d+ e≥ 1

5+

+(1− a)(1− b)(1− c)(1− d)(1− e).

Marin Chirciu

Subiectul 4Aratati ca ecuatia x5 = 4x+1 are exact doua radacini reale x1 < x2 si calculati[x2]− [x1].

Ionel Tudor

Page 68: olimpiade2009

CAPITOLUL 3. CLASA XI 68

3.4.10 Concursul ”Vasile Dumitrache”

Subiectul 1

Fie A =

(i cos� sin�− sin� −i cos�

), B =

⎛⎝ i sin� cos�

− cos� −i sin�

⎞⎠ , � ∈ R.

a) Aratati ca A2009 = A.b) Gasiti n ∈ N∗ , astfel incat det(A+ �B)n = det(A− �B)n,∀� ∈ R.

Subiectul 2

Fie f : (0,∞)→ R, f(x) = x ⋅ [√x] si xn =

n∑k=1

f(k2 + k),∀n ≥ 1.

Calculati: limn→∞

(4xnn4

)n.

Subiectul 3Se considera f : R→ R o functie continua cu proprietatea ca exista

limx→−∞

(f(x) + x) si limx→∞

(f(x) + x) ambele finite.

Aratati ca:a) Functia f nu este marginita.b) Daca f + 1R este periodica , atunci exista a ∈ R astfel incatf(x) = −x+ a,∀x ∈ R.

Page 69: olimpiade2009

Capitolul 4

Clasa XII

4.1 Etapa locala

4.1.1 Arges

1. Pentru a ∈ R se definesc functiile fa : R → R, fa(x) = x + a, ga : R → R,ga(x) = x+ a. Fie G={fa∣a ∈ R} ∪ {ga∣a ∈ R}.a) Sa se demonstreze ca (G, ∘) este grup necomutativ.b) Sa se determine multimea A = {n ∈ N∗∣(∃)ℎ ∈ G, ord(ℎ) = n}

Marin Ionescu, profesor PitestiMarian Teler, profesor Costesti

2. Pe multimea R, definim legea de compozitie:

x ∗ y = ( 2009√x+ 2009

√y − 1)2009, (∀)x, y ∈ R

a) Sa se arate ca (R, *) este grup comutativ;b) Aratati ca (R, *) este izomorf cu grupul (R+

∗, ⋅ );c) Calculati x ∗ x ∗ x ∗ ... ∗ x, x ∈ R, unde * se repeta de n ori, iar n este unnumar natural nenul.

Stefan Tudosie, profesor Campulung Muscel

69

Page 70: olimpiade2009

CAPITOLUL 4. CLASA XII 70

3.a) Fie f : (0,∞)→ R o functie derivabila si F o primitiva a sa care verificaegalitatea:

F (x) =x

2[2f(x)−x−4] + 2arctgx, (∀)x ∈ (0,∞); Sa se determine functia f.

b) Calculati:

∫1

x(xln18 + 1)dx, x > 0.

Stefan Tudosie, profesor Campulung Muscel

4. Sa se determine cel mai mic numar natural n cu proprietatea ca existax,y,z∈ Zn, distincte doua cate doua, astfel incat x+ y = 2⋅ z si x⋅ y = z2

I.S.J Arges

4.1.2 Braila

1. Se da legea de compozitie ” ∗ ” : R×R→ R, x ∗ y = xy − 3(x+ y) + 12.a) Sa se determine numarul real a astfel incat multimea G = [a,∞) sa fie partestabila a lui R in raport cu legea ” ∗ ”.b) Sa se rezolve in R ecuatia x ∗ x ∗ ... ∗ x︸ ︷︷ ︸

n

= 4, n ∈ N∗.

***

2. Sa se calculeze

∫cosx

(2 + sinx)(3 + sinx)3dx, x ∈ R.

Gazeta Matematica

Page 71: olimpiade2009

CAPITOLUL 4. CLASA XII 71

3. Fie (G, ⋅ ) un grup si a ∈ G. Aratati ca daca x⋅ a3 = a⋅x3, ∀x ∈ G, atuncia⋅x = x⋅ a, ∀x ∈ G.

***

4. Sa se determine toate functiile f : R→ R pentru caref(x)⋅F (x− 1) = 1 + x2, ∀x ∈ R, unde F este o primitiva a lui f.

***

4.1.3 Caras-Severin

1. Fie f : R → (0,∞) o functie bijectiva. Sa se studieze daca exista functiig : R→ [0,∞) care admit primitive pe R si satisfac relatia g ∘ g = f .

RMCS 26

2. Daca G este un grup multiplicativ in care, pentru orice x, y, z ∈ G, avem:xy2 = z2x⇒ y = z, sa se arate ca:a) x2 ∕= e, ∀x ∈ G− {e};b) G este abelian;c) Orice grup cu proprietatile a) si b) are proprietatea din enunt.

Prof. Marian Andronache, Bucuresti, RMCS

3. Fie (G, ⋅ ) un grup cu proprietatea ca x2 = e, ∀x ∈ G. Sa se arate capentru orice functie f : G → G si orice a ∈ G, a ∕= e, functia g : G → G,g(x)=f(x)f(ax) nu este injectiva.

***

4. Sa se determine

∫x2 + x+ 1

x2 + 1⋅ earctgxdx, x ∈ R.

Page 72: olimpiade2009

CAPITOLUL 4. CLASA XII 72

4.1.4 Calarasi

1. Pe multimea Z se defineste legea de compozitie: x ∗ y = 5xy + 6x+ 6y + 6.a) Sa se arate ca legea ” ∗ ” este asociativa.b) Sa se determine elementele simetrizabile ale multimii Z in raport cu legea:” ∗ ”.c) Sa se rezolve ecuatia: x ∗ x ∗ x ∗ ... ∗ x︸ ︷︷ ︸

de 2009 ori

= −1.

***

2. Sa se calculeze

∫xn−1(x2n − 1)

x4n + 1dx, n ∈ N,n ≥ 2, x ∈ R.

***

3. Fie S o multime nevida si ∗ : S × S → S o lege de compozitie interna cuproprietatile:I) a ∗ (a ∗ b) = b; II) (a ∗ b) ∗ b = a, ∀a, b ∈ S.Aratati ca legea de compozitie este comutativa.

***

4. Calculati:

∫2nx − 2x

(1 + 2x)n+1dx, n ∈ N,n ≥ 2.

G.M. 9/2007

4.1.5 Cluj

1. Fie f : R → R, f bijectiva si q ∈ R cu f(q)=2. Pe R se defineste legea decompozitie ” ∘ ” prin: a ∘ b = f(f−1(a) + f−1(b)− q), ∀a, b ∈ R.

Page 73: olimpiade2009

CAPITOLUL 4. CLASA XII 73

a) Determinati elementul neutru al legii de compozitie si simetricul lui a ∈ R inraport cu legea ” ∘ ”;b) Pentru f(x) = x3, x ∈ R rezolvati ecuatia: x2 ∘ x = (6− q)3

prof. Aura Buju, Lic.T. Petru Maior Gherla

2. Determinati matricea X ∈M2(Z5), stiind ca X3 =

(1 2

3 1

).

3.a) Aratati ca ln(1 + x) ≤ x, ∀x ∈ [0,∞);

b) Demonstrati inegalitatea

∫ln(1 + x)

1 + x4dx ≤ �

8

4. Fie p, n ∈ N∗ numere fixate. Verificati daca exista functii f : R→ R∗ care

admit primitive, astfel incat pentru o primitiva F a lui f sa avem:

p∑k=1

F (1 +kx)

-

p∑k=1

F (1− kx) = F (x2n), ∀x ∈ R.

prof. Ilie Diaconu, Lic.T. Avram Iancu

4.1.6 Dolj

1. Fie grupul (G, ⋅ ) cu elementul neutru e si H un subgrup propriu al sau, astfelincat G/H =

{x ∈ G/ {e} ∣ x2 = e

}. Sa se arate ca:

a) H este comutativ;b) G nu este comutativ.

2. Fie sirul Fk, k ∈ N , definit astfel: F0 = 1, F1 = 0, Fk+1 = Fk + Fk−1,

∀k ∈ N∗ si M =

⎧⎨⎩⎛⎝ Fk 0 Fk+1

0 1 0Fk+1 0 Fk+2

⎞⎠ /k ∈ N

⎫⎬⎭. Sa se arate ca M este

monoid comutativ in raport cu inmultirea matricelor.

Gazeta Matematica 4/1998

Page 74: olimpiade2009

CAPITOLUL 4. CLASA XII 74

3. Sa se calculeze integrala:

∫ex(x− �)

x(x� + ex)dx, x ∈ (0,∞), � ∈ R.

Constantin Cristian Dinu

4. Fie f : R→ R o functie injectiva si �, � : R→ R, �(x) = 2x3−x2−x+3,�(x) = −x3 + 3x2 + 3x+ 1, ∀x ∈ R. Sa se demonstreze ca daca functiile � ∘ fsi � ∘ f admit primitive pe R, atunci f este continua.

4.1.7 Galati

Subiectul 1.Fie legea de compozitie ∗ : R→ R, x∗y = 2xy−6x−6y+21,∀x, y ∈R si xn solutia ecuatiei x ∗ x ∗ ... ∗ x︸ ︷︷ ︸

n

= 35, n ∈ N∗. Calculati limn→∞

xn.

Subiectul 2.Fie (G, ⋅) un grup cu 30 de elemente in care exista a, b ∈ G∖{e}distincte astfel incat a2 = b2 = e. Sa se arate ca (G, ⋅) nu este grup abelian , efiind elementul neutru al grupului.

Subiectul 3.a) Calculati

∫(x+ 2)earctgx

(1 + x2)2dx, x > 0.

b) Calculati

∫sinx

2009 sinx+ 2008 cosxdx, x ∈

(0,�

2

).

Subiectul 4.Determinati functia f : [0,∞)→ (0,∞) care admite primitivaF , stiind ca F (x) = 2009xf(x),∀x ≥ 0 si f(0) = 1.

1. Fie G = (k,∞)−k + 1, k > 0, pe care se defineste legea ”∗” astfel incat:

Page 75: olimpiade2009

CAPITOLUL 4. CLASA XII 75

loga[(x ∗ y)− k] = loga(x− k)⋅ loga(y − k), (∀)x, y ∈ G, a > 0, a ∕= 1.a) Sa se determine numarul perechilor de numere (k,a) pentru care elementulneutru al legii ” ∗ ” sa fie e=2009.b) Sa se verifice ca legea ”∗” este asociativa si sa se determine elementele x ∈ Gcare verifica relatia: x=x’, in cazul k=a+1=1005 (s-a notat cu a’ simetricul luia in raport cu legea ” ∗ ”).c) Sa se determine f : (R∗, ⋅ )→ (G, ∗), izomorfism de grupuri, cu proprietatile:I. dreapta y=1 este asimptota orizontala spre −∞ la graficul functiei f;II. f(1)=2010.

Prof. Ion Viorel, Liceul Teoretic Dunarea

2. Fie (G, ⋅ ) un grup cu proprietatea ca exista a ∈ G astfel incat a2x5a2 = x,(∀)x ∈ G. Sa se demonstreze ca x8 = e, (∀)x ∈ G, unde e ∈ G este elementulneutru al grupului (G, ⋅ ).

Dumitru si Rodica Balan, profesori, Galati.

3. Sa se calculeze:

∫sinx− x cosx

x2 − sin2 xdx, x ∈ (0, �2 ).

Prof. Dumbrava Vasile, L.E.R., Galati.

4. Fie In =

∫ 1

0

n√

1 + x+ xndx, n ∈ N,n ≥ 2.

a) Sa se calculeze I2.b) Sa se arate ca sirul (In)n≥2 este convergent si sa se determine limita sa.

Conf. dr. Jenica Cringanu, Univ. Galati

4.1.8 Hunedoara

1. Fie G = (1,∞) si legea x ∗ y = (x− 1)(y − 1) + 1, (∀)x, y ∈ G.a) Sa se arate ca (G, ∗) este grup.

Page 76: olimpiade2009

CAPITOLUL 4. CLASA XII 76

b) Sa se arate ca daca H este un subgrup al lui G care contine toate numerelenaturale din G, atunci H contine toate numerele rationale din G.

2. Calculati:

a)

∫1

x(1 + x)3dx, x ∈ (0,∞).

b)

∫ �2

0

cos 2x

(sinx+ cosx)ndx.

3. Fie (G, ⋅ ) un grup cu elementul neutru e astfel incat x−1 = x, (∀)x ∈ Gsi a ∈ G/ {e}. Aratati ca oricare ar fi functia f : G → G, functia g : G → G,g(x)=f(x)⋅f(ax), (∀)x ∈ G, nu este injectiva.

4. a) Aratati ca: limn→∞

n(1

n2 + 12+

1

n2 + 22+ ...+

1

n2 + n2) =

4.

b) Dovediti ca (∀)y ∈ (0, 1), exista un sir (yn)n≥1 de numere reale astfel incat

limn→∞

n(1

n2 + y12

+1

n2 + y22

+ ...+1

n2 + yn2) = y.

4.1.9 Iasi

1. a) Demonstrati ca functia f : R→ R, f(x)=

{cos

1

x, x ∕= 0

0, x = 0, are primitive

b) Dati exemplu de o functie neintegrabila care sa admita primitive si un exem-plu de functie integrabila care sa nu admita primitive.

2. a) Calculati

∫ln(√

1 + x−√

1− x)dx, x ∈ (0, 1].

b) Fie f : R→ R o functie cu proprietatea ca pentru orice functie continuag : R→ R, functia f+g are proprietatea lui Darboux. Rezulta ca f este in modnecesar continua?

3. a) Determinati toate subgrupurile grupului (Z13∗, ⋅ ).

b) Fie (G, ⋅ ) un grup cu 13 elemente si f : G→ G, f(x) = x6. Demonstrati caf este automorfism al lui G.

Page 77: olimpiade2009

CAPITOLUL 4. CLASA XII 77

Subiect elaborat de Cristian Lazar

4.1.10 Prahova

1. Fie (G, ⋅ ) un grup finit cu n elemente, n ≥ 3, cu proprietatea: ∀x ∈ G,∃y ∈ Gastfel incat x = y2.a) Demonstrati ca n este impar.b) Demonstrati ca grupul (Zn,+), cu n impar, are proprietatea de mai sus.

Prof. Dorin Vasile, Ploiesti

2. Spunem ca o functie f are proprietatea ”P” daca f continua pe [a,b] si

∀c ∈ (a, b),∃u, v ∈ [a, b], u ∕= v astfel incat

∫ v

u

f(x)dx = (v − u)f(c), unde

a, b ∈ R, a < b fixati.a) Pentru a=0 si b=2 aratati ca f(x) = xe−x nu are proprietatea ”P”;b) Sa se demonstreze ca pentru orice functie continua pe [a,b] care admite oprimitiva convexa pe [a,b] are proprietatea ”P”.

Prof. Octavian Purcaru, Ploiesti

3. Fie functiile f, F : [0,∞) → [1,∞), F(0)=1, [F(x)]=[f(x)], ∀x ≥ 0 si Feste o primitiva a functiei f. ([a] - partea intreaga a lui a);a) Sa se arate ca exista o functie f cu aceste proprietati;b) Demonstrati ca F este bijectiva;

c) Calculati limx→∞

F−1(x)

ln x.

Prof. Emil Vasile, Ploiesti

4. Fie n ∈ N∗. Sa se calculeze:

∫x3n−1 + xn−1

x4n − x2n − 1dx, x ∈ R.

Page 78: olimpiade2009

CAPITOLUL 4. CLASA XII 78

Prof. Ion Nedelcu, Ploiesti

4.1.11 Sibiu

1. Fie multimea G=

{A =

(u 00 v

)/u, v ∈ C;A2009 = I2

};

a) Aratati ca G este parte stabila a multimii matricelor patratice de ordinul 2in raport cu inmultirea matricelor.b) Demonstrati ca (G, ⋅ ) este grup abelian.c) Cate elemente are grupul?

Prof. Carmen Reich-Sander

2. a) Determinati o primitiva a functiei:

f : [0, 2�]→ R, f(x) = arcsin(sinx).

Prof. Carmen Reich-Sander

b) Calculati:

limn→∞

3n(sin

3n+ sin

2�

3n+ ...+ sin

(n− 1)�

3n).

Prof. Adriana Pasca

3. Fie (G, ⋅ ) un grup finit cu proprietatea ca exista a ∈ G, a ∕= e, astfel incatx2 = e, ∀x ∈ G/ {a}. Demonstrati ca a2 = e, unde e este elementul neutru algrupului.

Gazeta Matematica

Page 79: olimpiade2009

CAPITOLUL 4. CLASA XII 79

4. Fie f : [a, b]→ (0,∞) si g : [a, b]→ [a, b] functii continue pe [a,b]. Atunciexista c ∈ [a, b] astfel incat:

∫ b

a

g(x)(f(x)dx = a

∫ c

a

f(x)dx + b

∫ b

c

f(x)dx.

Prof. Petru Vlad

4.1.12 Timis

1. Sa se calculeze primitivele functiei f(x) = cos3x⋅ cos8x, x ∈ R.

Prof. Aurel Dobosan, Lugoj

2. Fie G =

⎧⎨⎩Aa,b/Aa,b =

⎛⎝ 0 a 00 a 0a b a

⎞⎠ , a ∈ R∗, b ∈ R

⎫⎬⎭.

a) Aratati ca G este parte stabila a lui M3(R) in raport cu inmultirea matricilor;b) (G, ⋅ ) este grup comutativ;c) Calculati Ana,b, oricare ar fi n ∈ N∗;d) Determinati X ∈ G astfel incat A1,2⋅X = A2,1.

Prof. Aurel Dobosan, Lugoj

3. Fie (G, ⋅ ) un grup multiplicativ cu elementul unitate u, iar a, b ∈ G/udoua elemente cu proprietatea ca:

a = bab si b = aba

Aratati ca au loc egalitatile:i) a2 = b2, ii) a4 = u; iii) b−1ab = a−1.

Page 80: olimpiade2009

CAPITOLUL 4. CLASA XII 80

Prof. Mihai Chis

4. Fie functiile f, g : R→ R, definite prin f(x) = sh(x)=ex − e−x

2, respectiv

g(x)= ch(x) =ex + e−x

2. Aratati ca:

a) Functia f este bijectiva.b) f’=g, g’=f si g2(x)− f2(x)=1, (∀)x ∈ R.

c) Functia inversa f−1 este o primitiva a functiei ℎ : R→ R,1√

x2 + 1.

Prof. Mihai Chis

4.1.13 Vaslui

1. Fie a,c ∈ R, b ∈ RR, Ga,b = (a, b) si legea de compozitie ” ∗ ” definita prinGa,b astfel:x ∗ y = (x− a)(y − a) + c, (∀)x, y ∈ Ga,b. Sa se arate ca:a) Legea de compozitie ” ∗ ” este asociativa daca si numai daca a=c.b) (Ga,b, ∗) este grup daca si numai daca c=a si b =∞.

***

2. Sa se calculeze:

∫sinx+ 2 cosx

3 + sin 2xdx.

Prof. Cozma Constantin

3. Fie functia f : (0,∞)→ G, f(x)=x− 1

x+ 2.

a) Sa se determine multimea G si legea de compozitie Δ : G × G → G astfelincat (R∗+, ⋅ ) ≈ (G,Δ).b) Sa se calculeze xΔxΔ...Δx︸ ︷︷ ︸

n ori

, n ∈ N∗.

Page 81: olimpiade2009

CAPITOLUL 4. CLASA XII 81

Prof. Cozma Constantin

4. Sa se arate ca functia continua f : R→ R este periodica, de perioada T,

daca si numai daca pentru orice x ∈ R avem:

∫ x+T

x

f(t)dt = const.

***

4.1.14 Valcea

1. Fie (G, ∘) un grup finit si H = {x ∈ G/xn = e}; n ∈ N∗, n impar. Sa searate ca H are un numar impar de elemente.

***

2. Pe multimea R∗+ se introduce legea x∗y = an√lognax+logna y−logna b, unde a,b

∈ R∗+/ {1}, n impar, n ∕= 1. Sa se arate ca:a) (R∗+, ∗) este grup abelian;b) Grupul (R∗+, ∗) este izomorf cu grupul aditiv al numerelor reale.

***

3. Fie 0 < a < b. Sa se calculeze limt→0

(

∫ 1

0

[bx+ a(1− x)]tdx).

***

4. Sa se calculeze:

∫ 1

−1

(1 + x4)⋅ ex

(1 + x6)⋅ (ex + 1)dx.

Page 82: olimpiade2009

CAPITOLUL 4. CLASA XII 82

G.M. Nr. 6 / 2007

4.2 Olimpiada judeteana

Subiectul 1.Fie f : [0,∞) → [0,∞) o functie descrescatoare , astfel incat∫ x0f(t)dt < 1,oricare ar fi x ≥ 0.Sa se arate ca:

a) limx→∞

∫ x

0

f(t)dt exista si este finita;

b) limx→∞

xf(x) = 0.

P.M.Tristar

Subiectul 2.Fie A un inel comutativ cu n elemente , n ≥ 2.Sa se arate caurmatoarele afirmatii su8nt echivalente:

a) x2 = x oricare ar fi x ∈ A;

b) numarul functiilor polinomiale f : A→ A este n2.

Gazeta Matematica

Subiectul 3.Fie f : [0, 1]→ R o functie continua astfel incat

∫ 1

0

(x−1)f(x)dx =

0. sa se arate ca:

a) functia H : [0, 1] → R,H(x) = 1x

∫ x

0

tf(t)dt −∫ x

0

f(t)dt daca x ∈ (0, 1]

si H(0) = 0, indeplineste conditiile teoremei lui Rolle pe intervalul [0, 1];

b) Exista un punct a ∈ (0, 1) astfel incat

∫ a

0

xf(x)dx = af(a).

Mateforum,Cezar Lupu

Subiectul 4.fie K un corp finit cu q elemente si n ≥ q, n ∈ N .Sa se determine

Page 83: olimpiade2009

CAPITOLUL 4. CLASA XII 83

probabilitatea ca alegand un polinom din multimea de grad n din K[X],acestasa nu aiba nici o radacina in K.

Marian Andronache,Bucuresti

4.3 Olimpiada nationala

Subiectul 1.Fie functia f : [0, 1] → R derivabila cu derivata continua astfel

incat∫ 1

0(f ′(x))2dx ≤ 2

∫ 1

0f(x)dx.

Sa se determine f stiind ca f(1) = − 16 .

G.Rene

Subiectul 2.Fie (A,+, ⋅) un inel comutativ finit.Notam cu d numarul divizo-rilor lui zero , iar cu n numarul elementelor nilpotente ale inelului. Sa se arateca :a) daca x si y sunt nilpotente,atunci x+ y si x ⋅ y sunt nilpotente.b) n divide d.

Dorel Mihet

Subiectul 3.Sa se determine numerele naturale n ≥ 2 cu proprietatea ca ininelul (Zn,+, ⋅) exact un element nu se poate scrie ca suma de doua patrate.

Marian Andronache

Subiectul 4.Sa se determine functiile f : [0, 1]→ [0, 1] continue si bijective cuproprietatea ca∫ 1

0g(f(x))dx =

∫ 1

0g(x)dx,

pentru orice functie continua g : [0, 1]→ R.

Dorin Andrica si Mihai Piticari

Page 84: olimpiade2009

CAPITOLUL 4. CLASA XII 84

4.4 Concursuri interjudetene

4.4.1 Concursul”Alexandru Myller”

1. Fie f, g doua functii polinomiale reale, de acelasi grad, ambele avand coefi-cientul dominant egal cu 1. Daca g nu are radacini reale pozitive, calculati:

limn→∞

n

∫ 1

0

x(f(nx)

g(nx)− 1)dx

Radu Gologan

2. Fie A un inel.a) Aratati ca daca x ∈ A este nilpotent (exista k ∈ N∗ cu xk = 0) atunci 1+xeste inversabil.b) Daca A este finit, numarul elementelor inversabile este un numar prim iarelementele neinversabile sunt nilpotente si numarul elementelor nein- versabileeste mai mare sau egal cu numarul elementelor inversabile, aratati ca A are 4elemente.

***

3. Sa se determine functiile continue f : R→ R care verifica egalitatea:

f(arctg x) = (1 + x2)f(x),

pentru orice x ∈ R.

Al. Gabriel Mirsanu

4. Fie p, p > 2, un numar prim si f un polinom cu coeficienti intregi de gradp-1 cu proprietatea ca: pentru orice a,b numere intregi pentru care p dividef(a)-f(b) rezulta ca p divide a-b. Aratati ca f are coeficientul dominant divizibilcu p.

Page 85: olimpiade2009

CAPITOLUL 4. CLASA XII 85

Marian Andronache

4.4.2 Concursul ”UNIREA”

1. Intr-un grup (G, ⋅ ) de ordin 6 si element neutru e se considera elementele a,ba ∕= e, b ∕= e, astfel incat a3 = e, b2 = e. Fie c = ab.a) Aratati ca c = ba2 sau c = ba.b) Calculati ordinul lui c cand c = ba2, respectiv c = ba.

2. Fie matricea A =

(1 4

3 2

)∈ M2(Z5). Decideti daca multimea G =

{A,A2, A3, A4} este grup in raport cu inmultirea matricelor.

3. Fie f : [0,∞)→ R, derivabila, cu derivata continua si satisfacand f(0) =0. Aratati ca, daca f(x)f ′(x) ≥ x pentru orice x ∈ [0,∞), atunci ∣f(x)∣ ≥ ∣x∣oricare ar fi x ∈ [0,∞).

4. Fie f : [0, 1]→ R o functie integrabila Riemann si (xn)n≥1 un sir conver-gent de numere reale. Aratati ca sirul (yn)n≥1 dat de:

yn =

n∑k=1

xknf(k

n)

este convergent si sa i se calculeze limita.

4.4.3 Concursul ”Petru Morosan-Trident”

1. Calculati:

∫2nx − 2x

(1 + 2x)n+1dx, n ∈ N , x ∈ R.

Prof. Cantemir Iliescu, Curtea de Arges, Gazeta matematica nr. 3/2008

Page 86: olimpiade2009

CAPITOLUL 4. CLASA XII 86

2. Pe R se defineste legea de compozitie:

x ∘ y = xy − n(x+ y) + n(n+ 1), n ∈ N∗, n fixat.

Notam cu:

x(k) = x ∘ x ∘ ... ∘ x︸ ︷︷ ︸kori

, k ∈ N∗, k ≥ 2.

a) Aratati ca: x(2p) ≥ n, (∀)p ∈ N∗, (∀)x ∈ R.b) Sa se calculeze: (n+ 1)(3) + (n+ 2)(3) + ...+ (n+ n)(3).c) Sa se determine partile stabile finite ale lui R, in raport cu legea de compozitie” ∘ ”.

Prof. Gheorghe Alexe, Braila

3. Fie f : [0, 1]→ R, f(x) =

⎧⎨⎩ 1− (1− x2)n

x2, x ∈ (0, 1]

n, x = 0, (∀)n ∈ N∗

a) Aratati ca f este derivabila in 0.

b) Demonstrati ca

∫ 1

0

f(x)dx = −1 + 2n

∫ 1

0

(1− x2)n−1dx.

c) Deduceti identitatea:

C1n −

1

3C2n +

1

5C3n − ...+ (−1)n−1 1

2n− 1Cnn = −1 +

2⋅ 4⋅ 6⋅ ...⋅ (2n)

1⋅ 3⋅ 5⋅ ...⋅ (2n− 1).

Prof. Radu Vasile, Braila

4.4.4 Concursul ”Grigore Moisil”

1. Fie f : [0, 3] → R o functie de doua ori derivabila, cu f” continua, astfel

incat f(3)=2, f’(3)=1 si

∫ 3

0

f(x)dx=6. Calculati

∫ 3

0

x2f”(x)dx.

Page 87: olimpiade2009

CAPITOLUL 4. CLASA XII 87

***

2. Determinati functiile derivabila f : R→ R, cu proprietatea ca:

(f(x))2009 =

∫ x

0

(f(t))2008

dt, oricare ar fi x ∈ R.

Cristinel Mortici

3. Determinati toate tripletele de numere complexe (p,q,r) astfel incat

G = {z ∈ C∣ z3 + pz2 + qz + r = 0} ⊂ C∗

sa fie grup in raport cu inmultirea numerelor complexe.

Revista Minus

4. Calculati:

∫ 1

−1

1

(3 + x2)(1 + ex)dx.

Gazeta Matematica

4.4.5 Concursul ”Laurentiu Duican”

1. Se considera un grup comutativ (G, ⋅ ), cu cel putin trei elemente. Fie asi b doua elemente distincte ale grupului, diferite de elementul neutru. Sa sedetermine functiile injective f : G→ G, cu proprietatea:

f(af(x)y)=bf(xy), ∀x, y ∈ G.

Page 88: olimpiade2009

CAPITOLUL 4. CLASA XII 88

D.M. Batinetu, Giurgiu.

2. Se considera o functie f : [0, 1]→ R, cu proprietatea:

∫ 1

0

(x− 1)2f(x)dx = 0.

Sa se arate ca exista � ∈ (0, 1) astfel incat sa avem:

∫ �

0

(�− x)f(x)dx = 0.

I.V. Matei

3. Fie f : [a, b]→ R o functie de doua ori derivabila pe [a,b], cu f” continuape [a,b]. Presupunem ca f si derivatele sale sunt strict pozitive pe [a,b]. Sa sedemonstreze inegalitatea:∫ b

a

√f”(x)

f(x)dx ≤ ln

√f(b)f ′(b)

f(a)f ′(a).

Sa se determine functiile f care satisfac cerintele enumerate si realizeaza egali-tatea in relatia de mai sus.

Cristinel Mortici

4. Fie a, b ∈ R, cu a ∕= 0. Sa se determine functiile continue f : R → R, cuproprietatea:

f(ax+ b

∫ x

0

f(t)dt) = ax+b, ∀x ∈ R

Romeo Ilie

Page 89: olimpiade2009

CAPITOLUL 4. CLASA XII 89

4.4.6 Concursul ”Gheorghe Lazar”

1. Fie f : [0, 1]→ R continua si crescatoare pe [0,1]. Sa se arate ca oricare ar fin ∈ N∗ avem inegalitatea:

∫ 1

0

(tn + tn−1)f(t)dt ≥ 2n+ 1

n(n+ 1)

∫ 1

0

f(t)dt.

Emil C. Popa, Sibiu

2. Fie f(x) = anxn + an−1x

n−1 + ...+ a1x+ a0, n ≥ 2, un polinom care aretoate radacinile reale si distincte.a) Aratati ca polinomul pf (x) = (n−1)[f ′(x)]2−nf(x)f”(x) are toate radacinilecomplexe nereale.b) Demonstrati ca:

(n− k)(k + 2)(ak+12 − akak+2) ≥ (n+ 1)ak+1

2, k = 0, n− 2.

Ioan Tincu, Sibiu

3. Functia f : [−1, 1]→ R, este integrabila si para.

a) Aratati ca I =

∫ 1

−1

f(x) arccosxdx =�

2

∫ 1

−1

f(x)dx.

b) Calculati J =

∫ 1

−1

arccosx

1 + x2dx.

Doru Isac, Sibiu

4. Se considera multimea:

M =

{(a b

b a

)/ a, b ∈ Z7

}

Page 90: olimpiade2009

CAPITOLUL 4. CLASA XII 90

si matricea A =

(4 3

3 4

)∈M . Sa se rezolve in M ecuatia X6 = A.

GM 1/2008

Page 91: olimpiade2009

Partea II

Probleme propuse

91

Page 92: olimpiade2009

92

4.5 Probleme propuse

4.5.1 Clasa a IX a

1.Daca a, b, c > 0 si abc=1,aratati ca:a2(b5 + c5) + b2(a5 + c5) + c2(a5 + b5) ≥ 2(a+ b+ c)

Dragoi Marius,Posa Bogdan ,Revista Cardinal 1/2006

2.Sa se determine functia f : R→ R cu proprietatea ca:f(x) + 2f([x]) + 2f({x}) = 3x2

Grigore C.Moisil 2003

3.Fie ABC si MNP doua triunghiuri avand H,respectiv H1 ortocentre,iarO,respectiv O1 centrele cercurilor circumscrise.Aratati ca, daca

−−−→HH1 + 2

−−→OO1 =−→

0 ,atunci cele doua triunghiuri au acelasi centru de greutate.

Olimpiada locala Olt 2003

4.Fe a,b,c numere reale strict pozitive astfel incat abc=1.Sa se arate ca:1 + bc

1 + a+

1 + ac

1 + b+

1 + ab

1 + c≥ 3

Olimpiada locala Prahova 2003

5.Fie a,b,c numere strict pozitive astfel incat ab+ bc+ ac = 1.Sa se demon-

streze:1

a+ b+

1

b+ c+

1

a+ c≥√

3 +ab

a+ b+

bc

b+ c+

ac

a+ c

Olimpiada Harghita 2003

6.In triunghiul ABC,D,E si F sunt punctele in care bisectoarele interioare aleunghiurilor A,B si respectiv C intersecteaza laturile sale.Demonstrati ca dacaAD,BE,CF formeaza un triunghiu,atunci triunghiul ABC este echilateral.

***

Page 93: olimpiade2009

93

7.Fie f : R→ R o functie cu proprietatea ca f(x+y)=f(x)+f(y) pentru oricarex,y reale.Determinati multimea F = {x ∈ R∣f(x) = x},stiind ca este finita.

Olimpiada locala Sibiu 2005

8.Determinati m,n ∈ R,astfel incat [x] + [x+m] = [nx],∀x ∈ R.

9.Daca x3 + y2 > 4 si y3 + x2 > 4, x, y ∈ R,aratati ca x+ y > 1.

Alexandru Cojocaru 2005

10.Fie triunghiul ABC si M mijlocul laturii (BC).Daca triunghiul ABC veri-fica proprietatea ca pentru orice P ∈ (AB) exista Q ∈ (AC) astfel incat centrulde greutate al triunghiului MPQ apartine bisectoarei unghiului BAC,atunci tri-unghiul ABC este isoscel.

Olimpiada locala Constanta 2007

11.Sa se arate ca exista o infinitate de valori ale lui a ∈ Z∗ pentru careecuatia 2a2x2 + 2x− 1 = 0 sa admite radacini rationale.

Olimpiada locala Suceava 2007

12.Aflati aria unui patrat pentru care exista un punct P in interiorul sau cu∣PA∣ =, 1∣PB∣ = 2, ∣PC∣ = 3.

Vranceanu-Procopiu 2005

13.Aratati ca oprice functie f : R→ R cu proprietatea f(x+1)+f(x−1) =√2f(x),∀x ∈ R este periodica.Dati exemplu de astfel de functie.

Mathematica-Modus Vivendi 2005

14.Sa se determine functiile f : R→ R care verifica relatia f(x3)− f(y3) =(x2 + xy + y2)(f(x)− f(y))

Grigore C.Moisil 2005

Page 94: olimpiade2009

94

15.Fie ABCD un patrulater convex,M ∈ (AD) si N ∈ (BC) cu AMMD =

BNNC .Notam cu P si Q intersectia dreptei MN cu BD si respectiv AC.Sa se arateca [MP]=[NQ] daca si numai daca AB∥CD

Cristian S.Calude 2007

16.Demonstrati inegalitatea :∣x+ y∣+ ∣y + z∣+ ∣z + x∣ ≤ ∣x∣+ ∣y∣+ ∣z∣+ ∣x+ y + z∣,∀x, y, z ∈ R

Inegalitatea lui E.Hlawka

4.5.2 Clasa a X a

1.Fie numerele reale a,b,c astfel incat a ≥ b ≥ c > 1.Sa se demonstreze ca esteadevarata inegalitatea locc(logcb) + locb(logba) + loca(logac) ≥ 0.

Dan Barbilian 2007

2.Sa se afle solutiile pozitive ale ecuatiilor:a)4x + 4

1x = 18

b)9x + 91x = 84

3.Sa se arate ca pentru orice a, b ∈ C de modul 1,are loc inegalitatea:∣a+ 1∣+ ∣b+ 1∣+ ∣ab+ 1∣ ≥ 2

Olimpiada Buzau 2008

4.Sa se determine functiile injective f : R→ R cu proprietatea ca :f(f(x) + y) = f(x) + f(y),∀x, y ∈ R

Olimpiada Caras 2008

5.Fie a,b,c trei numere complexe cu proprietatile : ∣a∣ = ∣b∣ = ∣c∣ si abc=ab+bc+ac=1.Sase calculeze a+b+c.

Olimpiada Neamt 2008

Page 95: olimpiade2009

95

6.Sa se arate ca ecuatia z3 + z2 + z − 6 = 0 nu are radacini de modul egalcu 2.

Grigore Moisil 2008

7.Fie A,B,C,D patru puncte din plan,distincte,de afixe a,b,c,d astfel incata2 + b2 = b2 + c2 = c2 + d2 = 0.Aratati ca ABCD este patrat.

Radu Miron 2007

8.Fie ABC un triunghi si G centrul sau de greutate astfel incat:BC +AG = AC +BG = AC +CG.Aratati ca triunghiul ABC este echilateral.

Arhimede 2007

9.Fie numerele a, b ∈ N cu a+b=9.Care este valoarea minima a expresiei2a + 4b.

Ion Barbu 2007

10.Rezolvati in R sistemul format din ecuatiile: 3x + 4y = 5z;3y + 4z =5x;3z + 4z = 5y

Traian Lalescu 2007

11.Rezolvati ecuatia : 4x + 9x + 25x = 6x + 10x + 15x.

Olimpiada Iasi 2004

12.Fie a un numar complex nenul, cu proprietatea ca exista un numar intregn astfel incat an si an+1 sunt intregi.Sa se arate ca a sau 1

a sunt intregi.

Nicolae Teodorescu 2007

4.5.3 Clasa XI

1.Fie A,B ∈ Mn(C) doua matrice ce satisfac realatiile Ap+q + Bp+q = On siApBp −BqAq = In, unde p, q ∈ Z.Aratati ca BpAp −AqBq = In

Page 96: olimpiade2009

96

Posa Bogdan,Marius Dragoi RMT 3/2008

2.Fie A ∈ M3(R) o matrice simetrica si inversabila. Aratati ca det(A2 +I3) + det(A−2 + I3) ≥ 16

Posa Bogdan,Marius Dragoi RMT 3/2008

3.Fie f : R→ R o functie continua pe Q cu proprietatea:f(x) < f(x+ 1

n ),pentru orice x real si n natural.Sa se arate ca f este crescatoarepe R .

Al.Myller 2008

4.Fie f : R+ → R+ pentru care are loc relatia f(f(x)) + 9x = f(6x),∀x ∈R+.Aratati ca f(x) ≥ 3x.

Posa Bogdan,Marius Dragoi Recreatii Matematice 2/2008

5.Fie A o matrice de ordinul doi astfel incat Tr(A)=1.Sa se arate ca det(A2 +A+ I3) ≥ 3

Olimpiada Mures 2004

6.Fie A ∈ M2(Z) o matrice pentru care exista un n natural astfel incatAn = I2.Sa se arate ca exista k natural 1 ≤ k ≤ 6 astfel incat Ak = I2

Olimpiada Mures 2005

7.Fie f : R → R o functie cu proprietatea lu Darboux .Sa se arate ca f estecrescatoare pe multimea numerelor irationale atunci f este continua.

Al.Myller 2005

8.Se considera o functie f : R→ R cu proprietatea:f(x + 1)f(x) + f(x + 1) + 1 = 0,∀x ∈ R.Demosntrati ca urmatoarele afirmatiisunt echivalente:a)Ecuatia f(x)=-1 nu are solutii in R .b) f nu este continua

Page 97: olimpiade2009

97

Arhimede 2007

9.Determinati toare polinoamele P cu coeficienti reali cu proprietatea capentru orice matrice reale de ordinul 2 distincte avem P (A) ∕= P (B).

Olimpiada Dolj 2003

10.Fie A si B doua matrice de ordinul doi.Sa se arate ca daca det(AB+BA) ≤0 atunci det(A2 +B2) ≥ 0

Olimpiada Harghita 2003

11.Sa se arate ca p(x) = det(A + xAt) este un polinom simetric , undeA ∈Mn(R).

12.Fie n ∈ N ,n impar si A ∈ Mn(R).Daca A2 = O2 sa se demonstreze cadet(A+ In) ≥ det(A− In).

13.Fie A,B ∈ Mn(R) cu proprietatea ca 3A2 + B2 = AB − BA.Sa se de-mosntreze da daca A este inversabila atunci n este divizibil cu 3.

Laurentiu Duican 2007

4.5.4 Clasa XII

1.Sa se determine functiile continue f, g : R → R astfel incat∫ baf(x)dx =

g(a)− g(b) si∫ bag(x)dx = f(a)− f(b)

2.Sa se determine functiile continue f : R+ → R cu proprietatea:f(x) =

∫ x0f(x− t)f(t) pentru orice x ∈ R+ .

Laurentiu Duican 2007

3.Fie (G⋅ ) un grup cu 2004 elemente si fie a ∈ G, ord(a)=3.Sa se rezolve ingrupul G ecuatia x2003 = a

Dana Heuberger

Page 98: olimpiade2009

98

4.Fie G un grup finit cu proprietatea ca ord(G) este liber de patrate.Sa sedemonstreze ca G este ciclic.

5.Fie A(+⋅ ) un inel cu proprietatea ca x6 = x5,∀x ∈ A.Sa se arate ca ineluleste comutativ.

6.Sa se arate ca daca intr-un inel A cu element-unitate avem x6 = x, pentruorice x ∈ A , atunci x2 = x pentru orice x ∈ A.

7.Fie A un inel astfel incat xn+5 = xn,pentru orice x ∈ A.Aratati ca inelulA este comutativ.

8.Fie A un inel cu element-unitate avand 2n − 1 elemente inversabile si celmult tot atatea elemente neinversabile.Sa se demonstreze ca A este corp cu 2n

elemente.

9.Determinati functiile derivabilef : R → R, g : R → R∗+ care indeplinescurmatoarele conditii:a)f ′g + fg′ = f2 + g2

b)g2(x) + f2(x) = xc)f(0)=0

Bogdan Stoean

Page 99: olimpiade2009

Partea III

Indicatii si raspunsuri

99

Page 100: olimpiade2009

100

4.6 Clasa IX

Arges

2.Avem x2 = x + nt unde t este ratia progresiei,iar n ∈ N .Deci x3 =x2 + xnt = x + nt + xnt = x + n(x + 1)t.Deci si x3 este termen al progre-siei.Prin inductie se arata ca xn apartine progresiei.

3.a)f(x + 4) + f(x) = f(x + 2) ⇒ f(x + 4) = f(x − 2) deci functia f esteperiodica cu perioada 6.b)Se verifica

4.A se vedea manualul

Cluj

1.x ∈ [−4, 4] ⇒ 1 ≤ x2 + 1 ≤ 17 ⇒ 15 ≤

x2+15 ≤ 17

5 ,x2+1

5 ∈ Z ⇒ x2+15 ∈

{1, 2, 3} etc...

2.Se explicita partile reale si avem de efectuat intersectia [3k+2, 3k+5)∩[2k+3, 2k+5).Se analizeaza cele 4 cazuri posibile si se obtine solutia [1, 2)∪[3, 7)∪[8, 9)

3.Se trec toti memebri in partea stanga si fie q ratia progresiei:

b21(1− q2 + q4 − ...− q2n−2)(1− q4n) = b21(1−q4n)2

1+q2 ≥ 0

4.a)Se foloseste inegalitatea Cauchy-Buniakowskib)Se aplica inegalitatea de la punctul a)

5.Fie O centrul cercului circumscris patrulaterului.Daca Ha este ortocen-trul triunghiului BCD atunci conform relatiei lui Sylvester avem:

−−→OHa =−−→

OB +−−→OC +

−−→OD,deci

−−−→OMa = 1

2 (−→OA+

−−→OB +

−−→OC +

−−→OD =

−−−→OMb =

−−−→OMc =

−−−→OMd

Dolj

Page 101: olimpiade2009

101

1.Se tine cont de observatia : exista 2k+1 numere naturale p cu [√p] = k.

2.Se obtine f(x) = x2 + 1

3.Inegalitatea data este echivalenta cu urmatoarea:1 + q2 + ...+ q10 ≥ 11q5,inegalitate care reiese din inegalitatea mediilor.

4.Totul rezulta din relatia lui Sylvester.

Galati

1.Solutiile ecuatiei sunt{− 2

3 , 0,23

}

2.E =sin 53∘ − 2 cos 40∘ sin 13∘

cos 63∘=

sin 40∘ cos 13∘ + cos 40∘ sin 13∘ − 2 cos 40∘ sin 13∘

cos 63∘=

sin 27∘

cos 63∘= 1

3.Folosind inegalitatea din enunt se ajunge la:1

1−a1 + 11−a2 + ...+ 1

1−an−1< 1

an− 1

a1.Cum 1

1−ai > 1, i = 1, n obtine ca nan < 1

si cum nan > 0 rezulta concluzia.

4.a)Se aplica teorema lui Menelaus in triunghiurile ABA’ si ACA’.b) si c) Se aplica teorema lui Ceva

Iasi

1.Fie x = n+ a, n ∈ Na ∈ (0, 1).Ecuatia data devine (n+ k)a = (n+ a)k,deunde obtinem ca n(k − a) = 0.In concluzia: daca k=0 soluia este [0, 1) ∪ Z,iarcand k ∈ Z solutia este [0, 1)

2.Inegalitatea de demonstrat se sparge in inegalitati de tipul:a+ba2+b2 ≤

12 ( 1a + 1

b )

Page 102: olimpiade2009

102

3.Consideram P mijlocul lui (NQ).Atunci 2MA+ 5MD = 7MN si 3MB +4MC = 7MQ.Deci 2MA + 3MB + 4MC + 5MD = 7

2MP .Fie k valoarea con-stanta data.Prin urmare locul geometric cerut este un cerc de centru P si raza 7

2k.

4.a)Se arata ca MD = − 114MC.

b)Se aplica teorema lui Menelaus in triunghiul BDC si se foloseste punctul an-terior.

Sibiu

1.Pentru n=1 se obtine x1 = 1.Se demonstreaza prin inductie ca xn = n

2.a)Calcul directb)Daca n=0 atunci S0 = 2,si S1 = 12Avem Sn+1 = 12Sn−35Sn−1 .Presupunem ca exista n ≥ 1 astfel incat Sn∣Sn+1,deciexista k astfel incat Sn+1 = kSn ⇒ 35Sn−1 = (12− k)Sn.Cum 35 nu divide pe12-k,5 sau 7 trebuie sa divida pe Sn,imposibil.

4.a)A se vedea manualulb)Se foloseste punctul a)

Timis

1.Se folosesc urmatoarele proprietati:i)Daca x ∈ Z atunci [x] + [−x] = 0i)Daca x ∕∈ Z atunci [x] + [−x] = 1

2.a)Se exprima vectorial AA’ si AM in functie de laturile triunghiului ,deunde si AA’A”.b)Se foloseste punctul a)

3.Avem a2 + 1 > a, b2 + 1 > a, c2 + 1 > c,care prin insumare dau a2 + b2 +c2 + 3 > a+ b+ c.Folosind relatia a2 + b2 + c2 = (a+ b+ c)2 − 2(ab+ bc+ ac)se ajunge la concluzie.

Page 103: olimpiade2009

103

4.a)Teorema lui Ceva

b)−→PS =

−→PA+

−→AS =

−→PA+ y(

−−→PB −−→PA) + z(

−−→PC −−→PA) = x

−→PA+ y

−−→PB + z

−−→PC

Vaslui

2.xy + yz + zx− xyz = (1− x)(1− y)(1− z)1− x = y + z ≥ 0 si analoagele iar concluzia se obtine usor

3.Se scriu vectorii de pozitie ale centrelor de greutate G1, G2, G3, G,G′, G”

si se calculeaza GG′

4.a)Se scrie relatia din enunt pentru toate valorile de la 1 la n si se insumeaza.b)Se obtine f(n) = 2n+2 − 2(n+ 2) si n=1 si n=2 verifica cerinta.

Alexandru Myller

1.Pentru n=1 putem alege A orice multime cu 1 elementDaca n ≥ 2 ,schimband a si b intre ele rezulta a(b3 + 6) = b(a3 + 6) , de undeab(a+ b) = 6.Apoi,daca a,b,c sunt trei element diferite ale multimii A atunci a, b, c ∕= 0 sia+ b+ c = 0.Deci A are cel mult 3 elemente.In sfarsit, pentru n=2 putem lua A=1,2 si pentru n=3 putem lua A=1,2-3

2.Trebuie eliminate cel putin unul dintre{a, a2

},deci cel putin 10 elemente.

Pe de alta parte,daca eliminam 1,2...10 ramane o multime de tipul cerut.

Unirea

1.Fie n ∈ N, [x2 ] = n.Avem {4x} = 4√

5 − 7 − n.Din 0 ≤ 4√

5 − 7 − n < 1

obtinem n=1 si deci {x} =√

5− 2 etc

2.Fie T intersectia dreptelor AD si BC.Se aplica teorema lui Menelaus intriunghiul ATC cu D,E,B coliniare si folosind si relatia din enunt se ajunge laasemanarea triunghiurilor TDC si TBA.Observatie:in cazul in care AD si BC sunt paralele atunci se arata usor ca ABCDeste trapez isoscel,iar orice trapez isoscel este inscriptibil.

Page 104: olimpiade2009

104

3.Fie an = a0 + n�, bn = b0 + n�.Deducem ca anbn = A + Bn + n2C,undeA = a0b0, A = a0� + b0�,C = ��.Folosind acestea se obtine un = 0.

4.Fie A’ mijlocul lui BC.Paralela IL la AB taie BA’ in K.OL taie BC inM.Aplicam relatia lui Menelaus in triunghiul AA’C si se obtine ca MC=LCceea ce este echivalent cu concluzia noastra.

Trident

1.Se face x → 1x si se ajunge f(x) = (x + 3)2 iar suma ceruta se calculeaza

usor.

2.Aplica inegalitatea CBS si obtinem:

(n2 − 1)([√

1]2 + [√

2]2 + ... + [√n2 − 1]2) ≥ ([

√1] + [

√2] + ... + [

√n2 − 1])2

Tinand cont de egalitatea [√

1] + [√

2] + ... + [√n2 − 1] = n(n−1)(4n+1)

6 rezultainegalitatea ceruta.

3a)Aplica relatia medianei in triunghiurile FMH,EFG si EGHb)Construim bisectoarele unghiurilor ADB si ADC care taie laturile AB si ACin punctele E si F.Fie M si N mijloacele segmentelor AD si EF.Aplicand punc-tul a) in patrulaterul AEDF si tinand cont ca triunghiul EDF este dreptunghicobtinem AE2 + AF 2 = AD2 + 4MN2 ⇒ AE2 + AF 2 ≤ AD2 (1).Aplicandteorema bisectoarei in triunghiurile ABD si ADC si folosind relatiile obtinutese ajunge la echivalenta relatiei (1) cu concluzia.

Vasile Dumitrache

1.Scazand primele doua ecuatii obtinem (x−y)(x+y−z) = 0 si se analizeazaambele cazuri.

2.a1

a1a2 + n− 1+

a2

a2a3 + n− 1+ ... +

aaana1 + n− 1

≤ a1

a1a2...an + n− 1+

a1

a1a2...an + n− 1+ ...+

aaa1a2...an + n− 1

=a1 + a2...+ ana1a2...an + n− 1

Deci ramane de

aratat ca a1 +a2...+an ≤ a1a2...an+n−1.Aceasta ultima inegalitate se obtinetinand ca daca x, y ∈ (0, 1) atunci (1− x)(1− y) ≥ 0 adica x+ y ≤ xy + 1.

Page 105: olimpiade2009

105

3.Fie ABC triunghiul data.Se duc paralele prin punctul P la laturile tri-unghiului ABC ca taie latura BC in M si Q , latura AC in N si T si latura ABin R si S . In triunghiurile PMQ,PNT si PRS avem PP1, PP2, PP3 mediane.Sescrie relatia medianei vectorial si se aduna cele 3 relatii.

Victor Valcovici

1.Se aplica inegalitatea mediilor si apoi CBS.

2.Se exprima GI1 si GI2 ,vectorial , in functie de AB si AC.

3.Fie (an)n≥1 progresia aritmetica.Printre termenii progresiei se gasesec a1, a2 =

(1 + r)a1, a1(2r + 1) = a1(1 + r)2, ..., a1(1+r)k−1

r = (1 + r)ka1 .

Grigore Moisil

1.Se aplica CBS.

2.a = 0, b = 1, c = 8, d = 2

3.12a = ∣f(2) − 2f(1) + 2f(−1) − f(−2)∣ ≤ ∣f(2)∣ + 2∣f(1)∣ + 2∣f(−1)∣ +∣f(−2)∣ ≤ 12.Se obtine a = 1, b = 0, c = −3, d = 0

4.� = 1

Laurentiu Duican

1.AvemA = B∪C, undeB = {x ∈ R∣f(x) = x} iar C = {x ∈ R∣af(x) + ax+ b+ 1 = 0}si raman de studiat doua ecuatii de gradul 2.

2.Functia f(x) este crescatoare.Fie k ∈ Z.Avem f(x) ∈ [k, k + 1],∀x ∈ (k, k + 1).Presupunem ca exista a ∈(k, k + 1) astfel incat f(a)=k+1.Atunci f(na) ≥ nf(a) = n(k + 1).Contradictiecu faptul ca f este monoton crescatoare.

Page 106: olimpiade2009

106

3.a)x,y,z,t se pot reprezenta sub forma:x = m−b, y = m−a, z = m+a, t = m+b.Cu acestea inegalitatea este evidenta.b)Se aplica punctul a)

4.Din AP=AQ cu P situat pe inaltimea din A obtinem P ∈ (B,Q),MA per-pendicular pe BC si unghiurile PAQ si ABC egale.

Se obtineAPQ

ABC=cosB(1− cosB)

2(1 + cosB)≤ (√

2− 1)2

2

Gheorghe Lazar

1.Se cauta o functie liniara cu proprietatea din enunt.Se obtine f(x) =− 2

9x+ 59

2.Se foloseste inegalitatea dintre media aritmetica si media patratica.

4.6.1 Olimpiada Judeteana

Subiectul 1.Observam ca vectorii DA+DB si EA+EC au , respectiv directiilevectorilor necoliniari AB si AC , deci suma lor este 0 doar dacac amandoi sunt0.Rezulta ca punctele D si E sunt mijloacele segmentelor AB si AC , deci T estecentrul de greutate al triunghiului ABC .Din relatia TA+ TB+ TC = 0 rezulta� = −1.

Subiectul 2.Vom incerca sa eliminam 10 elemente astfel incat printre celeramase sa nu existe 10 in progresie aritmetica.Observam ,mai intai, ca numereledin fiecare linie si din fiecare coloana sunt in progresie aritmetica , deci va trebuisa eliminam cate un numar din fiecare linie si din fiecare coloana .Implicit rezultaca oricare 2 dintre cele 10 numere eliminate trebuie sa fie din linii si coloanediferite.Apoi, daca numarul eliminat din linia i, 1 ≤ i ≤ 9 , se afla pe coloana k , atuncinumarul eliminat din linia i+ 1 trebuie sa se afle pe o coloana l, cu l < k ,altfel,intre cele 2 numere raman in tabel neeliminate cel putin 10 numere consecutive,care formeaza o progresie aritmetica.Deducem ca,pentru a nu ramane in tabel 10 numere pe aceeasi linie sau coloanasau 10 numere consecutive , trebuie sa eliminam numerele 10,19,28,...,91(adicanumerele de pe diagonala secundara a tabelului ). Dar atunci raman neeliminate

Page 107: olimpiade2009

107

numerele 11,20,29,38,47,56,65,74,83 si 92 care formeaza o progresie aritmeticacu ratia 9.Sa remarcam ca pozitionarea numerelor intr-un tabel a fost facuta doar pentrua indica solutia de mai sus.

Subiectul 3.a) Inegalitatea este echivalenta cu

(x2 + 2xy + y2)(a3

x2 + b3

y2 ) ≥ a3 + 3a2b+ 3ab2 + b3

saux3b3

y2 + 2a3yx + 2 b

3xy + a3y2

x2 ≥ 3a2b+ 3ab2.Observam ca;x2b3

y2 + 2a2yx = x2b3

y2 + a3yx + a3y

x ≥ 33√b3a6 = 3a2b.

Analog,

2 b3xy + a3y2

x2 ≥ 3ab2.de unde prin adunare obtinem inegalitatea ceruta.b)Folosind inegalitatea precedenta , avem:a3

x2 + b3

y2 + c3

z2 ≥(a+b)3

(x+y)2 + c3

z2 ≥(a+b+c)3

(x+y+z)2 = a+ b+ c.

Subiectul 4.Pentru x = y obtinem :f(2x)+f(x)

2x+f(x) = 2x+f(x)f(2x)+f(x)

de undef(2x)+f(x)

2x+f(x) = 1

deci f(2x) = 2x,adica f(n) = n pentru orice n par.Fie acum x si y impare , deci x+ y este par, deci f(x+ y) = x+ y.Din ipotezaobtinem(x− y)2 + f(x)(y − x) + f(y)(x− y) = 0sau(x− y)(x− y − f(x) + f(y)) = 0Deducem ca f(x) − x = f(y) − y, pentru orice x ∕= y deci functia f(x) − x econstanta.Fie f(x)− x = k deci f(n) = n+ k pentru n impar.Fie acum x par si y impar.Obtinemx+y+k+x2x+y+k = 2y+x

x+y+k+y+k ,de unde k = 0 .Deducem ca f(n) = n,pentru oricen ∈ N∗.

4.6.2 Olimpiada nationala

Subiectul 1.Fie d paralela prin A la dreapta BC si P ,respectiv O , punctelede intersectie a dreptei MN cu dreptele BC,respectiv d.In cele ce urmeaza ,XYdesemneaza segmentul orientat de la X la Y .Conform teoremei lui Menelaus ,PCPB = MA

MB ⋅NCNA

Tinand cont de relatia din enunt si de faptul ca PC = PB+BC , din egalitatea

Page 108: olimpiade2009

108

de mai sus rezulta caMAMB ⋅ PB = − 1

k ⋅BCDin asemanarea triunghiurilor orientate MAQ si MBP , obtinemAQ = −MA

MB ⋅ PBdeci AQ = ( 1

k ) ⋅BC.Prin urmare,punctul Q este fix.

Subiectul 2.Inegalitatile se pot scrie bc > a(−e), (−e)(−f) > c(−g), (−g)(−ℎ) >(−f)d, da > (−ℎ)b,cu toti factorii numere reale strict pozitive.Inmultind mem-bru cu membru aceste inegalitati se obtibe bcefgℎda > aecgfdℎb absurd , cacicei doi membrii au aceeasi valoare abcdefgℎ.

Subiectul 3.Pornim de la inegalitatea x2+y ≥ y(x+1)2

y+1 adevarata pentru x, y

numere reale pozitive,fiind echivalenta cu (x − y)2 ≥ 0.Inmultind inegalitatilecorespunzatoare perechilor (ai, bi), 1 ≤ i ≤ n obtinem inegalitatea din enunt.

Subiectul 4.Una dintre implicatii este imediata , caci pentru ratia comunad rezulta ai = a1+(i−1)d si bj = b1+(j−1)d, deci ai+bj = a1+b1+(i+j−2)d,in total n+m− 1 valori distincte.Pentru implicatia inversa , sa remarcam ca a1 + b1 < a2 + b2 < ... < an + b1 <an + b2 < ... < an + bm , deci multimea sumelor contine intotdeauna cel putinn+m− 1 valori.Vom proceda prin inductie dupa n+m.Cazul n = m = 2 conduce la a1 + b1 <a1 + b2, a2 + b1 < a2 + b2 , de unde a1 + b2 = a2 + b1 , adica a2 − a1 = b2 − b1.Pentru n+m > 4 , cel putin una dintre valorile n,m (fie ea m) este mai maredecat 2.Atunci secventa obtinuta prin eliminarea lui bm contine m− 1 elementesi se pierde cel putin suma an + bm , maximul care nu mai poate fi obtinut ,deci raman cel mult n + m − 2 valori posibile ale sumelor .Conform cu primaobservatie facuta , acesta este numarul minim posibil de valori, deci raman exactn+ (m− 1)− 1 valori .In mod analog , prin eliminarea lui b1 se pierde cel putinsuma a1 + b1 (minimul care nu mai poate fi obtinut),cu concluzie identica.Daratunci secventele initiale sunt in progresie aritmetica de aceeasi ratie.

4.7 Clasa X

Cluj

1.Notam x = lga, y = lgb, z = lgc avem x ≤ y ≤ z < 0 siA = x

y + yz + z

x si B = yx + z

y + xz

Page 109: olimpiade2009

109

Atunci A−B = (x−y)(y−z)(z−x)xyz ≤ 0.Deci A ≤ B

2.2008 = 3⋅ 669 + 1⇒ z = (1 + �)669⋅ (1 + �2)669⋅ 2669(1 + �)⇒z = 2669(1 + �) = 2669(cos

3+ isin

3)

Deci ∣z∣ = 2669 si arg(z) = �3

3.a) S = n(2n+1)4

b)Folosind punctul a) ajungem la relatia:

(√x1 − 1− 1

2)2 + (

√x2 − 2− 1

2)2 + (

√xn − n−

1

2)2 = 0, de unde concluzia .

4.a)Se face x = y = 1b)Avem 0 = f(1) = f(x⋅ 1

x ) = f(x) + f( 1x ) ⇒ f( 1

x ) = −f(x).Fie x si y astfelincat f(x) = f(y)Avem f(xy ) = f(x) + f( 1

y ) = f(x) − f(y) = 0.Cum 1 este singura solutie a

ecuatiei f(x) = 0 obtinem ca x = y

Dolj

1.Daca n = 2 atunci x ∈ Zx = 5 este solutie pentru orice nDaca n = 1 atunci x = −4, x = −5Daca n > 2 si x > 0 atunci x = 5 este singura solutieDaca n > 2 , n impar si x < 0 nu avem solutiiDaca n > 2 , n par si x < 0 solutie ⇒ -x solutie ⇒ x=-5

2.Alegem t si t’ astfel incat t2 = z si t′2 = z′ , t⋅ t′ = uRelatia de demonstratt este echivalenta cu urmatoarea2(∣t2∣ + ∣t′∣2) = ∣t + t′∣2 + ∣t − t′∣2 care reprezinta teorema medianei in numerecomplexe.

3.Inpunem conditiile de existenta:sin2x ∈ (0, 1), x > 0cos2�x ≥ 1, x ≤ 5 .Obtinem x=4 , y=-2 sau y=1

4.Prin inductie se arata ca f((

2n∑i=1

ai)/2n) ≤ (

2n∑i=1

f(ai))/2n

Page 110: olimpiade2009

110

Apoi se arata ca f((

n∑i=1

ai)/n) ≤ (

n∑i=1

f(ai))/n .Aceasta ultima relatie se demon-

streaza tinand cont de prima inegalitate si de faptul ca orice numar natural sepoate scrie ca o suma de puteri ale lui 2.

Galati

1.In ipoteza data avem sin(a) > 0 si cos(a) > 0.Se logaritmeza natural inegalitatea din enunt si se obtine urmatoarea inegalitatede demonstrat:sin(a)ln(sin(a))+cos(a)ln(cos(a))+(sin(a)+cos(a))ln(cos(a)+sin(a)) ≤ 0⇔

sin(a)sin(a)+cos(a) ln(sin(a)) + cos(a)

sin(a)+cos(a) ln(cos(a)) ≤ −ln(cos(a) + sin(a)).

Aplicam inegalitea lui Jensen functiei concave x→ lnx si obtinem:

f(sin(a)

sin(a) + cos(a)sin(a)+

cos(a)

sin(a) + cos(a)cos(a)) ≥ sin(a)

sin(a) + cos(a)f(sin(a))+

cos(a)

sin(a) + cos(a)f(cos(a)) c.c.t.d

2.Se obtine solutia x ∈{√

22 + k, k ∈ Z

}3.Se inmulteste inegalitatea cu 2 si se sparge in inegalitati de tipul a3 + b3 ≥

a2b+ b2a .

4.Se foloseste urmatoarea teorema :Fie z1, z2 ∈ C∗ atunci ∣z1 + z2∣ ≤ ∣z1∣+ ∣z2∣. Avem egalitate atunci cand exista� > 0 astfel inca z1 = �z2.(vezi judetul Arges /problema 1)

Hunedoara

1.a) Calcul directb)Caz particular al punctului a):

x3 + y3 + z3 = x + y + z = 0 ⇒ xyz = 0 .Deci ln( b2

ac ) = 0 sau ln( c2

ab ) = 0 sau

ln(a2

bc ) = 0 , de unde concluzia.

2.Se obtine AP∥BQ∥CP∥DS.Deci APDS este paralelogram,deci simetricullui S fata de mijlocul lui AD este chiar P .

3.Notam a = z + 1z . Relatia din ipoteza devine ∣a∣ = ∣a3 − 3a∣ = r si cum

Page 111: olimpiade2009

111

3∣a∣ + ∣a3 − 3a∣ ≥ ∣a∣3 (1) obtinem 4r ≥ r3 deci 2 ≥ r. Acum daca r > 2 nuavem solutii. Daca r = 2 vom avea egalitate in (1), deci exista t ≥ 0 astfel incata3 − 3a = 3ta de unde obtinem a ∈ {−2, 2}, de unde z ∈ {−1, 1}

Iasi1.Observatie: in aceasta problema apare notiunea de inegalitatea in numerecomplexe.Enuntul problemei este nemodificat.Se intelege deci prin expresia z >0 ca z este un numar real si pozitiv.Revenind la rezolvarea problemei fie z1 = a + bi, z2 = c + di.Cum z1 + z2 =a + c + (b + d)i > 0 obtinem ca b+d=0 si a + c > 0 .Cum a2 + b2 = c2 + d2

obtinem ca a2 = c2 ⇒ a = c (a=-c nu convine). Deci z1 = z2 etc...

2.Se analizeaza cazurile:i)a−1a+2 ∈ (0, 1)

ii)a−1a+2 ≤ 1

3.Avem [ 3√x− 3] ∈ N si [ 3

√x− 2] ∈ N iar [ 3

√x− 3] ≤ [ 3

√x− 2].Deci [ 3

√x− 3] =

0 si [ 3√x− 3] = 1.Se obtine solutia orice numar din intervalul [3, 4).

4.Fie Pi, i = 1, n cei n sahisti iar ai, i = 1, n cu numarul invinsilor jucatoru-lui Pi.Din ipoteza obtinem ca ai ≥ 1, i = 1, n.Multimea finita {a1, a2..., an} areun cel mai mic element,putem presupune ca acesta este a1 iar P2, P2..., Pa1+1

invinsii acestuia.Cum a1 ≤ a2 exista un participant Pk invins de catre P2 , undek > a1 + 1.Alegand P1, P2, Pk cei 3 sahisti problema este rezolvata.

Sibiu

1.Se obtine solutia x=10,y=100,z=1000.

2.Aplicam inegalitatea mediilor si obtinem:

2x+ 4y = 2

132 +2y2 + 2

2x2 ≥ 2

174 +y2+ 1

x2 .Deci x+ 4

y ≥174 + y2 + 1

x2 .

Analog y + 4x ≥

174 + x2 + 1

y2 .Adunam inegalitatile si obtinem:

0 ≥ (x− 1

2)2 + (y − 1

2)2 + (

1

x− 2)2 + (

1

y− 2)2, de unde x = y = 1

2

3.a)Se ridica la puterea a treia si se obtine f(x)=1 sau f(x)=-1.

Page 112: olimpiade2009

112

b)Se foloseste punctul a)

4.Aplicam inegalitatea mediilor si obtinem:a+ b√

(s− a)(s− b)≥ 2

a+ b

2s− a− bsi analoagele.

E =∑ a+ b√

(s− a)(s− b)+ 6 ≥ 2(

∑ a+ b

2s− a− b+ 3) = 4s

∑ 1

2s− a− bDeci E ≥

∑[2s− a− b]

∑1

2s−a−b ≥ 9 , de unde concluzia.

Timis

1.Se obtine n=par.Ecuatia se scrie [lgn!] = n

2 ⇔ 10n2 ≤ n! ≤ 10

n2 +1

Prin inductie se demonstreaza ca n! > 10n2 +1 pentru n ≥ 10, n=par.

Se obtine singura solutie n=8

2.Evident daca toate cele n numere sunt nule atunci si suma lor este nula.Reciproc, daca z1 ∕= 0∣z1+z2+...+zn∣ = ∣z1−(z2−...−zn) ≥ ∣z1∣−∣z2+...+zn∣ ≥ ∣z1∣−∣z2∣−...−∣zn∣ =∣z1∣(1−

1

2− ... 1

n2) > 0.

Prin urmare daca z1 ∕= 0 atunci z1 + z2 + ...+ zn ∕= 0

3.Fie ∣u∣ = ∣v∣ = ∣w∣ = ∣z∣ = rDaca r=0 atunci E=0Daca r ∕= 0 atunci z = u + v + w ⇒ z = u + v + w ⇒ zz

z = uuu + uu

u + uuu ⇒

∣z∣2z = ∣u∣2

u + ∣v∣2v + ∣w∣2

w ⇒ 1z = 1

u + 1v + 1

w ⇒ uvw = z(vw + uw + uv).Dar E = z3 − z2(u+ v + w) + z(vw + uw + uv)− uvw = 0

4.Daca este indeplinita conditia a = d > b ≥ c = 0 atuncif(n) = [an+b

cn+d ] = [an+ba ] = [n + b

a ] = n,astfel ca f este aplicatia identica pe N∗

care este bijectiva.Presupunem ca functia f este bijectiva.Daca c ∕= 0 atunci pentru orice numarnatural care verifica inegalitatea

n > ∣bc−ad∣c2 − d

c rezulta caan+ b

cn+ d=a

c+

bc− adc(cn+ d)

∈ (a

c− 1,

a

c).Deci f(n) ∈ ([ac ] − 1, ]ac )] si functia f nu

poate fi injectiva,deci nici bijectiva.Rezulta ca c=0.Se arata ca f este monotom crescatoare.Cum singura functie bijectiva si mono-

Page 113: olimpiade2009

113

ton crescatoare de la N∗ la N∗ este aplicatia identica obtinem ca f este aplicatiaidentica.Rezulta ca pentru orice n natural au loc inegalitatile:n ≤ an+b

d < n+ 1 de unde se obtine ca a = d si b < d

Vaslui

1.xyz = 1⇒ ∣x∣ = ∣y∣ = ∣z∣ = 1∣z+ y+ z∣ = 1⇒ (x+ y+ z)(x+ y+ z)⇒ (x+ y)(x− 1)(z− 1) = 0⇒ x+ y = 0sau x = 1 sau y = 1.Pentru x+y=0 obtinem solutia {i,−i, 1}Pentru x = 1 obtinem solutia {1, i,−i}Pentru y = 1 obtinem solutia {i, 1,−i}

2.Din conditiile de existenta ale radicalilor obtinem x ∈ (− 12 ,

12 )

Aplicam C-B-S si obtinem

(5x√

12 + x+ 7x

√12 − x)2 ≤ (25x + 49x)( 1

2 + x+ 12 − x) = 25x + 49x

Avem egalitate pentru 25x−49x = 2x(25x+49x),ecuatie cu x=0 singura solutie.

3.Notand lna = x si lnb = y inegalitatea devine:

(x2 + y2)(1

x2+

1

y2) ≥ (x+ y)2

xy

Daca a > 1 si b > 1 atunci ea se sparge in (x2 + y2) ≥ (x+y)2

2 si1

x2+

1

y2≥ 2

xyDaca a > 1 si b < 1 memebrul drept este negativ iar cel stang este pozitiv.Cazul a < 1 si b < 1 se reduce la primul caz.

4.Daca A(z1), B(z2), C(z3),iar O(0) atunci din relatia din ipoteza avem C ∈(AB).Relatia de demonstrat este echivalenta cu OA2⋅BC + BO2⋅AC = OC2⋅AB +AC⋅BC⋅AB,care reprezinta relatia lui Stewart in triunghiul OAB cu cevianaOC.

Valcea

Unirea

Page 114: olimpiade2009

114

1.Considerand partea intreaga si partea fractionala, deducem[5x3] = 1 si

{2x2}

= 3−√

5.Deci 1 ≤ 5x3 < 2⇒ 0 ≤ 2x2 < 2

Daca [2x2] = 0 atunci 2x2 = 3−√

5 etc..Daca [2x2] = 1 atunci 2x2 = 4−

√5 > 1, 7⇒ 5x3 > 3, 6 ceea ce nu se poate

2.a)Functia f este strict crescatoare pe intervalul [m,∞) deci este injec-tiva.Pentru surjectivitate observam ca ecuatia f(x)=y are solutia x = m+

√y − 1

b)Ecuatia se poate scrie sub forma f(f(x)) = x.Deoarece f este crescatoareaceasta ecuatia este echivalenta cu ecuatia f(x) = xetc...

3.Se obtine f(0)=0 si f(x2) = xf(x) si f(−y2) = −f(−y2),de unde f(−x) =−f(x) pentru orice x real.

Apoi f(x2) = f(2x2 − x2) = x(√

2 + 1)(f(x√

2)− f(x)Dar f(2x2) = x

√2f(x

√2) de unde f(2x2) = 2f(x2).Folosind si imparitatea

functiei deducem ca f(2x) = 2f(x) pentru orice x real.Rationand inductiv obtinem f(nx) = nf(x) pentru orice n natural si x real.

4.Avem√a−√b = 1⇒ a− b =

√a+√b si a2 − b2 = (a+ b)(

√a+√b).

De aici obtinem (a2− b2)2 = (a+ b)2(√a+√b)2 ≤ (a+ b)3 de unde inegalitatea

ceruta.

Alexandru Myller

1.Avem modulo 41 : 102 ≡ 18,103 ≡ 16,104 ≡ −4,105 ≡ 1 .Se consideran = 5m+ i cu i = 1, 4,apoi se analizeaza toate cazurile.

2.Se exprima vectorial AM,BN si CP in functie de laturile triunghiului.

3.Daca M = {a, b, c, d} , a < b < c < d atunci f(a, b)+f(b, c)+f(c, d) ≤ d−asi d− a+ f(a, d) ≤ 1.Apoi f(a, c) ≤ 1

2 si f(b, d) ≤ 12 deci s(M) ≤ 2. O multime M pentru se real-

izeaza egalitatea este M ={

0, 14 ,

12 ,

34

}4.Daca a = p

q atunci sirul are perioada 2q.

Reciproc,sa presupunem ca sirul are perioada m.Atunci[a(n + m)] − [an] estepar.Pe de alta parte, am − 1 < [a(n + m)] − [an] < am + 1 , iar intervalaul

Page 115: olimpiade2009

115

(am − 1, am + 1) contine cel putin un numar par A,deci [a(n + m)] − [an] =A,constant.Deducem [amp] = pA pentru orice p natural ceea ce implica a = A

m ∈ Q

Victor Valcovici

1.Se noteaza 2x = a, 3x = b, 4x = c, 5x = d si ecuatia devine:a4 + b4 + c4 + d4 = a3b + b3c + c3d + d3a.Folosind inegalitatea mediilor sedemonstreaza ca a4 + b4 + c4 + d4 ≥ a3b+ b3c+ c3d+ d3a cu egalitatea pentrua = b = c = d

2.� = − 12 + i

√3

2 .

In repereul cartezian XOY notam A(1, 0) si B(− 12 ,√

32 ).Construim cele doua

cercuri cu centrele in A si respectiv B.Conform ipotezei problemei punctul deafix z se afla in zona comuna celor doua cercuri .Fie M intersectia celor doua cer-curi.Avem OAMB romb si triunghiul OAM echilateral.Deci OM=1,adica ∣z∣ ≤ 1

3.(x − 4)log45 = x − 1 + (2x − 5)log54 ⇒ (x − 4)log45 + x − 4 = 2x −5 + (2x − 5)log54 ⇒ 5log4(x−4) + 4log4(x−4) = 5log5(2x−5) + 4log5(2x−5), de undelog4(x− 4) = log5(2x− 5) = t.Aceasta ultima ecuatia este echivalenta cu urmatoarea:1 = 3( 1

5 )t + 2( 45 )x .Folosind faptul ca functia f(x) = 3( 1

5 )t + 2( 45 )x este strict

crescatoarea si ca g(3)g(4) < 0 obtinem existenta si unicitatea solutiei.

Grigore Moisil

1.Se exprima u2 + w2 si u3 + w3 in functie de uw si u + w de unde se aflauw.Avand uw si u+ w se afla u si w.

2.x=1 este solutia

3.lga+ lgb =√

2 si se foloseste inegalitatea 2(a2 + b2) ≥ (a+ b)2

4.Se foloseste urmatoarea observatie:Daca A,B,C sunt coliniare atunci ∣z1 − z3∣ = ∣z2 − z3∣+ ∣z1 − z2∣

Page 116: olimpiade2009

116

Laurentiu Duican

1.Relatia din enunt se obtine din urmatoarele doua relatii:∑∣a∣2 −

∑Re(ab) =

1

2∣a− b∣2∑

∣a− b∣2 ≥ 13 (∑∣a− b∣)2

2.Se aplica inegalitatea mediilor pentru ambele inegalitati.

3.Solutia problemei se bazeaza pe urmatoarea observatie:Fie f(x) = ax2 + bx+ c atunci relatia f(x)=f(y) implica x=y sau x+y=-a.

4.Centrele celor doua tetraedre coincid daca si numai daca:−−→AM +

−−→BN +

−−→CP +

−−→DQ =

−→0

Gheorghe Lazar

1.1

3x+1+

1

12x− 3

3x2

12x= 3∣⋅ 12x.Se ajunge la:

(3x2 +1 − 1)[4x(3

x2 +1 + 1) + 3] = 0,de undex = −2

3.Relatia data se poate scrie:(sin2B + cos2B)− 2(sinB + cosB)⋅ 2 sinA2 + sinA+ 1 ≤ 0 care se priveste ca oineecuatie de gradul 2.

4.Avem 1 + 22 + ...n2 = n(n+1)(2n+1)6 . an = n!

1+22+...n2 = 6n!n(n+1)(2n+1) =

6(n−1)!(n+1)(2n+1) .Se arata ca n+1 si 2n+1 sunt prime intre ele .

Daca n ≥ 6 si n+1 este prim sau 2n+1 este prim este an nu este natural deoarecen+ 1 > n− 1 si n+ 1 > 2n+ 1.Asadar daca n ≥ 6 si cel putin dintre numerele n+1,2n+1 este prim atunci annu este natural.Fie acum n ≥ 6 si n+1,2n+1 numere compuse.Daca n+ 1 = pq avem ca p < n− 1 si q < n− 1 deci p si q sunt termeni in n!.La fel pentru 2n+1=compus.

Trident

Page 117: olimpiade2009

117

1.Avem zAzA = zBzB = zCzC = 1 ⇒ zA = 1zA

si analoagele.Relatia data

devine∑cicli

(b+ c+ ka)zA = 0⇒ zA + zB + zC =(1− k)(azA + bzB + czC)

a+ b+ c

Daca k = 1⇒ zA + zB + zC = 0 deci triunghiul este echilateralDaca k = 0⇒ zH = zI deci triunghiul este echilateral.Daca k = −2⇒ zG = zI deci triunghiul este echilateral.

2.Se priveste egalitatea data ca una de gradul 2 in c.

3.Fie logn−7(n + 2009) = ab (a, b) = 1 ⇒ (n + 2009)b = (n − 7)a.Notam

n − 7 = x si avem (x + 2016)b = xa.Deci a > b.Fie a = b + c ⇒ (x + 2016)b =xb+c ⇒ x∣2006 = 25⋅ 32⋅ 7.Se analizeaza toate cazurile si singurul convenabil esten = 39

4.7.1 Olimpiada judeteana

Subiectul 1. a) Din f(g(x)) = g(f(x)) = −x deducem ca g(f(g(x))) =g(−x),pentru orice x real.Din g(f(x)) = −x obtinem ca g(f(g(x))) = −g(x)pentru orice x ∈ R.cele doua relatii de mai sus implica imparitatea functieig.Analog pentru functia f .b) Functiile f, g : R→ R date prin f(x) = x si g(x) = −x verifica proprietateadin enunt.

Subiectul 2.Din conditia data deducem ∣z1∣ = ∣z2∣ = ∣z3∣.Prin conjugare,din z1 + z2 + z3 = 1 deducem z1z2 + z2z3 + z3z1 = 1 = z1z2z3.Din cele douaegalitati precedente rezulta ca ecuatia cu radacinile z1, z2, z3 este z3−z2+z−1 =0, de unde {z1, z2, z3} = {1, i,−i}.

Subiectul 3.a) Fie x ∈ A.Atunci 3x = x + 2 de unde x = log3(x + 2) si1 = log2(3x − x) .Prin adunare obtinem log3(x + 2) + log2(3x − x) = 3x − 1 ,adica x ∈ B,deci A ⊂ B.b) 1 ∈ B deci B ∕⊂ R∖QFie a ∕= 1 astfel incat 3a = a + 2 .Atunci a ∈ B si aratam ca a ∈ R∖Q’deci va rezulta ca B ∕⊂ Q.Prin absurd ,daca a = m

n fractie ireductibila,implica3mn = m

n +2 ∈ Q, deci 3mn ∈ Q,adica n = 1 si prin urmare 3m = m+2,implicand

m = 1,absurd.

Subiectul 4. a) De exemplu,geometric,faptul ca numerele au acelasi modul

Page 118: olimpiade2009

118

si verifica z1 +z2 +z3 = 0 inseamna ca triunghiul cu aceste afixe are G = H,decieste echilateral.b) Daca n nu e multiplu de 3,nu exista triunghiuri echilaterale cu varfurile inafixe din Un, deci maximul cerut este n.Daca n = 3k atunci cele k triunghiuri achilaterale cu varfuri in Un putem alegecel mult cate doua varfuri din fiecare , adica cel mult 2k elemente.De exempluA = {!3, !6, ..., !3k} ∪ {!2, !5, ..., !3k−1} unde ! = cos( 2�

n ) + i sin( 2�n ).

4.7.2 Olimpiada Natioanala

Subiectul 1.a) Daca x = y avem solutiile x = m, y = m,m > 0.Daca x ∕= ynotam y = mx. Inlociund in egalitatea y⋅lgx = x⋅lgy obtinem solutiile x =

m1

m−1 , y = mmm−1 ,m ∕= 1.

b) Daca xy > yx rezulta ca (xy)(xy) > (yx)(yx).Ridicand la puterea 1xy , vom

obtine ca xxy−1

> yyx−1

, de unde xy + xxy−1

> yx + yyx−1

, fals.Deci xy = yx.Solutiile sunt cele indicate la punctul a).

Subiectul 2.Fie z1 = z2 = a2 + i

√4a−a2

2 radacinile ecuatiei z2 − az + a = 0.Avem ∣z2−az+a∣ = ∣z− z1∣∣z− z2∣ = MA ⋅MB,unde M,A,B sunt punctele dinplan corespunzatoare afixelor z, z1,respectiv z2.Fie T punctul de afix 1 si M1

intersectia dintre AM si cercul unitate daca unghiul AM1B este ascutit (in cazcontrar, M1 va fi intersectia dintre BM si cercul unitate).Avem MA ⋅MB ≥M1A ⋅M1B ≥ TA ⋅ TB = (TA)2 , deci

min∣z2 − az + a∣ = ∣z1 − 1∣2 = (a2 − 1)2 + (√

4a−a22 )2 = 1.

Subiectul 3.Inlocuind y = 0 in relatia data si intelegand toate relatiileurmatoare ca avand loc pentru orice valoare reala a variabilelor , avem : f(x3) =xf(x2) deci si f(x3 +y3) = f(x3)+f(y3) adica f(x+y) = f(x)+f(y).Observamca f verifica ecuatia data atunci, pentru orice constanta c reala, si cf verificarelatia.Putem presupune atunci ca f(1) = 1 sau f(1) = 0.Daca f(1) = 1 , atunci din f((x+1)3) = (x+1)f((x+1)2) avem 2f(x2)+f(x) =2xf(x) + x iar cu substitutia x → x + 1 obtinem 2f((x + 1)2) + f(x + 1) =(2x + 2)f(x + 1) + x + 1. Ultimele doua relatii duc la f(x) = x. Conformobservatiei avem f(x) = cx, c ∈ R∗.Daca f(1) = 0 , atunci f((x+ 1)3) = (x+ 1)f((x+ 1)2) adica 2f(x2) + f(x) =2xf(x) . Analog cu cazul precedent avem 2f((x+1)2)+f(x+1) = (2x+2)f(x+1)ceea ce implica f(x) = 0,∨x ∈ R.In concluzie f(x) = cx, c ∈ R.

Subiectul 4.Vom arata ca numerele neobisnuite sunt n = 4 si n = 5.Pentru

Page 119: olimpiade2009

119

n ≥ 6 suma elementelor tuturor patratelor 3×3 din patratul mare este egala cusuma elementelor tuturor patratelor 4×4 din acelasi patrat.Pentru a exemplificain acelasi timp ca n = 4 si n = 5 sunt numere neobisnuite propunem cititoruluiurmatorul exemplu: un patrat de dimensiune 5 × 5 in care coloana din mijloccontine numai −5 celelalte fiind 2.

4.8 Clasa XI

ClujSubiectul 1.Pentru k = 1⇒ B = In +APentru k = 2⇒ B +B2 = 2In + 3AB+B2 = 2In + 3A+A2, deci A2 = On ⇒ Am = Om,∨m ≥ 2,m ∈ N ⇒ Bm =(In +A)m = In +mA , de unde rezulta relatia A2 = On

Subiectul 2.a) Prin calcul rezulta a = d deci b = c

b) Fie r =√a2 + b2 =

√c2 + d2 > 0 si �, � ∈ (0,

2) astfel incat a = r cos�, b =

r sin�, c = r cos�, d = r sin�.Se demonstreaza prin inductie.

Subiectul 3.−∣x3∣ ≤ f(x) − x sinx ≤ ∣x3∣ , adunam x sinx si impartim la

x2, x ∕= 0⇒ sin xx − ∣x∣ ≤

f(x)x2 ≤ sin x

x + ∣x∣

limx→0

(sinx

x− ∣x∣) = lim

x→0(sinx

x+ ∣x∣) = 1 deci lim

x→0(f(x)

x2).

Subiectul 4.Avem inegalitatile : k3 < (k + 1)(k2 + 1) < (k + 1)3,∨k =1, n,∨n ≥ 1⇒ k2 < 3

√(k + 1)2(k2 + 1)2,∨k = 1, n,∨n ≥ 1

Deducem can∑k=1

1

(k + 1)2 + n< xn <

n∑k=1

1

n+ k2,∨n ≥ 1

Avem inegalitatile : n+ k2 ≥ 2k√n,∨k = 1, n,∨n ∈ N∗

Asadar

0 <

n∑k=1

1

n+ k2≤

n∑k=1

1

2k√n,∨n ∈ N∗

Avem can∑k=1

1

2k√n

=1 + 1

2 + ...+ 1n

2√n

,∨n ≥ 1

.

Page 120: olimpiade2009

120

Utilizand teorema Stolz-Cesaro , se arata ca

limn→∞

n∑k=1

1

2k√n

=1 + 1

2 + ...+ 1n

2√n

DoljSubiectul 1. a) (detA)2 = (detA) ⋅ (detAT ) = det(A ⋅AT ).b) Fie x = ab+ bc+ ca atunci cu observatia de la a) gasim

(detA)2 = det

⎛⎝ 1 x xx 1 xx x 1

⎞⎠ = 2x3 − 3x2 + 1 = x2(2x− 3) + 1

c) Cumx = ab+ bc+ ca ≤ a2 + b2 + c2 = 1

din b) rezulta concluzia.

Subiectul 2. a) Pentru inceput observam ca un ⋅ un+1 = n!, pentru n =0, 1, 2b) Probam prin inductie ca relatia de la a) are loc pentru orice n.Presupunemca uk ⋅ uk+1 = k! pentru orice k ≤ n.Avem apoi

n!(n− 1)! = un ⋅ un+1 ⋅ det(un−1 unun+1 un+2

)= det

(un−1un unun+1

unun+1 un+1un+2

)

det

((n− 1)! n!n! un+2

)= (n− 1)!un+1un+2 − n! ⋅ n!⇒ un+1un+2 = (n+ 1)!

si inductia este probata.c) Din b) gasim ca

unun−2

=un ⋅ un−1

un−1 ⋅ un−2=

(n− 1)!

(n− 2)!= n− 1

sau echivalent ,un = (n− 1)un−2

pentru orice n ≥ 2.Prin inductie se gaseste ca un = (n−1)!! , de unde concluzia.

Subiectul 3.a) Conditia din enunt este echivalenta cu (n + 1)xn+1 > (n +2)xn.b) Din a) si x1 = 1 rezulta ca termenii sirului sunt toti strict pozitivi.Fie k ∈N, k ≥ 2.Scriind inegalitatea de la a) pentru n = 1, 2, ..., k − 1 si inmultind

Page 121: olimpiade2009

121

inegalitatile respective obtinem2 ⋅ 3 ⋅ 4 ⋅ ... ⋅ x2x3...xk > 3 ⋅ 4 ⋅ 5 ⋅ ... ⋅ (k + 1) ⋅ x1x2...xk−1 , de unde

xk >k + 1

2x1 =

k + 1

2,∨k ∈ N, k ≤ 2

.Aceasta arata ca sirul este nemarginit.

Subiectul 4.a) Notam cu Sn urmatorul sir :

Sn =∑

1≤i ∕=j ∕=k ∕=i≤n

1

i

1

j

1

k= 3!

∑1≤i<j<k≤n

1

i

1

j

1

k

.b) Aplicam identitatea lui Newton :

n∑i=1

x3i = (

n∑i=1

xi)(

n∑i=1

x2i )− (

∑1≤i<j≤n

xixj)(

n∑i=1

xi) + 3∑

i≤i<j<k≤n

xixjxk

c) Atunci

∑1≤i<j<k≤n

xixjxk =1

3[

n∑i=1

x3i − (

n∑i=1

xi)(

n∑i=1

x2i ) + (

∑1≤i<j≤n

xixj)(

n∑i=1

xi)]

=1

3[

n∑i=1

x3i − (

n∑i=1

xi)(

n∑i=1

x2i ) +

(

n∑i=1

xi)2 −

n∑i=1

x2i

2(

n∑i=1

xi)]

=1

3[

n∑i=1

x3i

1

2(

n∑i=1

xi)3 − 3

2(

n∑i=1

xi)(

n∑i=1

x2i )]

.d) Aplicand ultima identitate cu xi = 1

i obtinem

Sn =

n∑i=1

1

i3+ (

n∑i=1

1

i)3 − 3(

n∑i=1

1

i)(

n∑i=1

1

i2)

Atunci

Sn(lnn)2

− lnn =

n∑i=1

1

i3+ (

n∑i=1

1

i)3 − 3(

n∑i=1

1

i)(

n∑i=1

1

i2)

(lnn)2− lnn

e) In continuare vom folosi ca limn→∞

(1 +1

2+ ... +

1

n− lnn) = c , constanta lui

Page 122: olimpiade2009

122

Euler.

Ca o consecinta limn→∞

1 + 12 + ...+ 1

n

lnn= 1

f) Din identitatea de la punctul d) obtinem ca

limn→∞

(Sn

(lnn)2− lnn) = lim

n→∞(

(

n∑i=1

)3

(lnn)− lnn),

unde am folosit ca sirurile an =

n∑k=1

1

k2si bn =

n∑k=1

1

k3

sunt marginite si deci ,

limn→∞

n∑i=1

1

i3

(lnn)2= 0

si

limn→∞

n∑i=1

1

i

lnn

n∑i=1

1

i2

lnn= 0

g) Revenim la relatia de la punctul d)

limn→∞

(Sn

(lnn)2− lnn) = lim

n→∞(

n∑i=1

1

i− lnn)

(

n∑i=1

1

i)2 + lnn(

n∑i=1

1

i) + (lnn)2

(lnn)2= 3e.

GalatiSubiectul 1.Fie functia f : R→ R, f(x) = det(A+ xB)− det(xA+B), A,B ∈M2(R).Dupa calcule se obtine ca f(x) = �x2 + �x+ , �, �, ∈ RDin ipoteza se obtine f(2008) = f(2009) = f(1) = 0 adica ecuatia de gradcel mult doi are trei radacini reale distincte . Rezulta ca f este functie nula:f(x) = 0,∨x ∈ R.Asadar pentru x = 0⇒ f(0) = 0⇒ detA = detB.

Subiectul 2.”⇐” Presupunem det(aX2 +bX+c) ≥ 0,∨X ∈M2(R).Pentru

X =

(0 x1 0

), x ∈ R obtinem det(aX2 + bX + cI2) = (ax + c)2 − b2x ≥

0,∀x ∈ R , adica a2x2 + (2ac − b2)x + c2 ≥ 0,∀x ∈ R, conditie echivalenta cu(2ac− b2)2 − 4a2c2 ≤ 0⇒ b2 − 4ac ≤ 0.” ⇒ ” Sa demonstram ca daca X ∈ Mn(R), n ∈ N,n ≥ 2 si k ≥ 0 atuncidet(X2 + kIn) ≥ 0Intr-adevar det(X2 + kIn) = det(X + i

√(k)In) det(X − i

√(k)In) = det(X +

i√

(k)In)det(X + i√

(k)In) = ∣det(X + i√

(k)In)∣2 ≥ 0.b2 − 4ac ≤ 0 si fie X ∈M2(R).

aX2 + bX + cI2 = a[(X +b

2aI2)2 +

4ac− b2

4a2I2] = a(Y 2 + kI2).

Page 123: olimpiade2009

123

unde Y = X +b

2aI2, k =

4ac− b2

4a2

Atunci det(aX2 + bX + cI2) = det[a(Y 2 + kI2)] = a2 det(Y 2 + kI2).

Subiectul 3.”⇐” Daca � ∈ Z rezulta xn = 0,∀n ∈ N si evident (xn)n≥0

este convergent.” ⇒ ” Presupunem ca (xn)n≥0 este convergent si fie lim

n→∞xn = x ∈ R.Atunci

xn+1 − xn → 0.Cum xn+1 − xn = {(n+ 1)�} − {n�} = (n+ 1)�− [(n+ 1)�]−n� + [n�] = � − yn , unde yn = [(n+ 1)�] - [n �] ∈ Z , rezulta ca � − yn → 0deci yn → �. Dar yn este un sir cu termeni intregi ⇒ � ∈ Z.

Subiectul 4.Notam limita cu L

L = limx→0

tg2008(x)− arctg2008(x)

x2009=

= limx→0

tgx− arctgxx2

⋅ 1

x2007⋅

2007∑k=0

tg2007−k(x) ⋅ arctgk(x) =

= limx→0

tgx− arctgxx2

⋅ limx→0

2007∑k=0

(tgx

x

)2007−k

⋅(arctgx

x

)k= L1 ⋅ L1 ,

unde L1 = limx→0

tgx− arctgxx2

si

L2 = limx→0

2007∑k=0

(tgx

x

)2007−k

⋅(arctgx

x

)kL1 = lim

x→0

tgx− arctgxx2

= limx→0

tgx− xx2

+ limx→0

x− arctgxx2

.

Deoarece sinx ≤ x, ∀x ∈(−�

2,�

2

)− {0} , avem

0 ≤∣∣∣∣ tgx− xx2

∣∣∣∣ ≤ ∣∣∣∣ tgx− sinx

x2

∣∣∣∣ ≤ ∣∣∣∣ tgx(1− cosx)

x2

∣∣∣∣ ≤∣∣∣∣∣∣tgx ⋅ 2 sin2 x

2x2

∣∣∣∣∣∣ , pentru

x ∈(�

2,�

2

)− {0}.

Cum limx→0

∣∣∣∣∣∣tgx ⋅

(2 sin2 x

2

)x2

∣∣∣∣∣∣ =

∣∣∣∣∣∣∣1

2⋅ limx→0

tgx ⋅ limx→0

⎛⎝ sinx

2x

2

⎞⎠2∣∣∣∣∣∣∣ =

∣∣∣∣12 ⋅ 0 ⋅ 12

∣∣∣∣ = 0

, rezulta ca limx→0

tgx− xx2

= 0

Notam arctgx = y

limx→0

x− arctgxx2

= limy→0

tgy − yy2

⋅ limy→0

y2

tg2y= 0 ⋅ 1 = 0

L1 = 0.

Page 124: olimpiade2009

124

L2 = limx→0

2007∑k=0

(tgx

x

)2007−k

⋅(arctgx

x

)k=

2007∑k=0

limx→0

(tgx

x

)2007−k

⋅ limx→0

(arctgx

x

)k=

2007∑k=0

1 ⋅ 1 = 2008

L = L1 ⋅ L2 = 0 ⋅ 2008 = 0. Sibiu

Subiectul 1

a) Fie A =

(a bc d

). Din XA = AX gasim b = c = 0 , deci X =

(a 00 d

).

b) Se demonstreaza prin inductie ca Xn =

(an 00 dn

), n ≥ 1.

c) Inmultind la stanga si la dreapta cu X , obtinem: X7 −X4 = XA = AX ,

deci X =

(a 00 d

), iar ecuatia devine

(a6 − a3 0

0 d6 − d3

)=

(2 00 6

).

Rezulta ca a ∈{

3√

2,−1}

si d ∈{

3√

3,− 3√

2}

.Multimea solutiilor pentru X ∈M2(R) este:

S =

{(3√

2 0

0 3√

3

),

(3√

2 0

0 − 3√

2

),

(−1 0

0 3√

3

),

(−1 0

0 − 3√

2

)}d) Daca X ∈ M2(C) , ecuatiile a6 − a3 − 2 = 0 si d6 − d3 − 6 = 0 au cate 6solutii distincte. Numarul de matrici pentru care verifica relatia va fi 6 ⋅ 6 = 36.

Subiectul 2Fie x = (−1)nd1

2d3 = (−1)n det(B) det(AB−1 −B−1A) det(B).Folosind acum proprietatea : ∀X,Y ∈Mn(C),det(XY ) = detX ⋅ detY .Gasim ca x = (−1)n det(B(AB−1 −B−1A)B) = (−1)n det(BA−AB).Se cunoste de asemenea ca det(−X) = (−1)n ⋅ detX.Asadar , avem: x = (−1)n det((−1) ⋅ (AB − BA)) = (−1)n ⋅ (−1)n ⋅ det(AB −BA) = d2.

Subiectul 3Daca a ≤ 0 , limita este +∞ , ceea ce nu corespunde cerintei , deci a > 0.Avand nedeterminarea ∞−∞ , expresia se poate scrie:

L = limx→∞

(a− a2)x4 + (2− 2ab)x3 − (b2 + 2ac)x2 − 2bcx− c2√x4 + 2x3 + (ax2 + bx+ c)

, de unde a = 1

L = limx→∞

2(1− b)x3 − (b2 + 2c)x2 − 2bcx− c2√x4 + 2x3 + (x2 + bx+ c)

.

Daca b ∕= 1 , limita este infinita , rezulta b = 1.

Page 125: olimpiade2009

125

L = limx→∞

−(1 + 2c)x2 − 2cx− c2

x2

(√1 +

2

x+ 1 +

1

x+

c

x2

) = 0 , de unde c = −1

2.

Subiectul 4Folosind inegalitatea data si stricata monotonie , gasim

xn3 ≥ (xn − 2)2

xn − xn+1≥ 0,∀n ≥ 1 , de unde rezulta ca xn > 0,∀n ≥ 1.

Fiind strict descrescator si marginit inferior de 0, sirul este convergent.Fie x ∈ R, x = lim

n→∞xn. Trecand la limita in inegalitatea data gasim

(x− 2)2 ≤ 0 , deci x = 2.

TimisSubiectul 1

x2n =

⎛⎝1 +sin(4n+ 1)

22n

⎞⎠2n

=

⎛⎝1 +sin(2n� +

2)

2n

⎞⎠2n

=

(1 +

1

2n

)2n

→ e

x2n+1 =

⎛⎝1 +sin(4n+ 3)

22n+ 1

⎞⎠2n+1

=

⎛⎜⎝1 +sin(2n� +

3�

2)

2n+ 1

⎞⎟⎠2n+1

=

(1− 1

2n+ 1

)2n+1

→ e−1 =1

e

Cum e ∕= e−1 rezulta ca limn→∞

xn nu exista adica sirul este divergent.

Subiectul 2

A = aI3 +B,B =

⎛⎝ 0 0 0a 0 0b a 0

⎞⎠

B2 =

⎛⎝ 0 0 00 0 0a2 0 0

⎞⎠ , B3 =

⎛⎝ 0 0 00 0 00 0 0

⎞⎠ = O3 ⇒ Bn = O3,∀n ≥ 3.

Cum aI3 ⋅B = B ⋅ (aI3) , aplicam binomul lui Newton:

An = anI3+nan−1B+n(n− 1)

2an−2B2 =

⎛⎜⎝ an 0 0nan an 0

nan−1b+n(n− 1)

2an nan an

⎞⎟⎠

Page 126: olimpiade2009

126

.

Subiectul 3Notand cu Ci(X) coloana a i-a a unei matriceX ∈M3(C) , iar cu ∣C1(X)C2(X)C3(X)∣determinantul matricei X , pentru orice numar complex z ∈ C are loc egalitateadet(A+ zB) = det(A)++z(∣C1(B)C2(A)C3(A)∣+ ∣C1(A)C2(B)C3(A)∣+ ∣C1(A)C2(A)C3(B)∣)++z2(∣C1(B)C2(B)C3(A)∣+ ∣C1()C2(A)C3(B)∣+ ∣C1(A)C2(B)C3(B)∣)++z3 det(B),sau, notand� = ∣C1(B)C2(A)C3(A)∣+ ∣C1(A)C2(B)C3(A)∣+ ∣C1(A)C2(A)C3(B)∣� = ∣C1(B)C2(B)C3(A)∣+ ∣C1()C2(A)C3(B)∣+ ∣C1(A)C2(B)C3(B)∣ ,avem

det(A+ zB) = det(A) + �z + �z2 + det(B)z3.

In particular, pentru z ∈{

1, !, !2}

, obtinem

det(A+B) = det(A) + �+ �+ det(B),

det(A+ !B) = det(A) + �! + �!2 + det(B),

det(A+ !2B) = det(A) + �!2 + �! + det(B).

Adunand ultimele trei egalitati si tinand cont de faptul ca 1 + ! + !2 = 0,obtinem atunci egalitatea dorita

det(A+B) + det(A+ !B) + det(A+ !2B) = 3(det(A) + det(B)).

Subiectul 4Din relatia de recurenta avem ca

xn+1 = e−xn + xn > xn,∀n ∈ N. astfel ca sirul dat este strict crescator , si prin urmare are o limita l = lim

n→∞xn.

Daca limita l este finita, atunci trecand la limita in relatia de recurenta obtinemca e−1 = 0 , egalitate care este imposibila. Prin urmare limita sirului (xn)n∈Neste +∞. Rezulta atunci ca lim

n→∞e−xn = 0 astfel ca

limn→∞

ln(n+ 1)− ln(n)

xn+1 − xn= limn→∞

ln

(1 +

1

n

)e−xn

= limn→∞

ln

(1 +

1

n

)⋅ e

xn

n= 1.

Folosind din nou lema Cesaro-Stolz, rezulta atunci ca exista limita limn→∞

ln(n)

xn

si ca limn→∞

ln(n)

xn= 1.

Page 127: olimpiade2009

127

Vaslui

Subiectul 1Din ipoteza detX = ±1,∀X ∈M2(R). Daca det(X) = 1 se obtine X2 = tX−I2, unde t este urma matricei. Atunci avem : X4 = (t3 − 2t)X − (t2 − 1)I2.Ecuatia din enunt devine: t(t2 − 2)X = (t2 − 2)I2 ⇒ tX = I2 daca X ∈M2Q.−t4I2 = (tX)4 = I2 ⇒ −t4 = 1 si analog cazul det(X) = −1.

Subiectul 2Se demonstreaza prin inductie ca xn ∕= −1,∀n ≥ 1 , obtinand astfel:

3xn+1 − 2xn + 1 = 0,∀n ≥ 1⇒ xn =

(2

3

)n−1

(a+ 1)− 1⇒ limn→∞

xn = −1.

Subiectul 3Se aduna toate coloanele la prina coloana , apoi se scade ultima linie din cele-lalte.

dn(z) = (

n∑k=0

zk)(−1)

n(n− 1)

2 .

Cum dn(z) = 0 ⇒ zn+1 = 1 ⇒ zk = cos2k�

n+ 1+ i sin

2k�

n+ 1, k ∈ {1, 2, 3, ..., n +

1}.

Subiectul 4

limx→∞

⎛⎝2

2− xx + 4

4− xx + 6

6− xx + 12

12− xx

⎞⎠x

=

= limx→∞

⎛⎜⎜⎝1 +2

2

x − 1

2+

4

4

x − 1

4+

6

6

x − 1

6+

12

12

x − 1

12

⎞⎟⎟⎠x

.

Utilizand limita fundamentalaau(x) − 1

u(x)obtinem lim

x→∞

⎛⎝2

2− xx + 4

4− xx + 6

6− xx + 12

12− xx

⎞⎠x

=

576.

Page 128: olimpiade2009

128

IMAR - Bucuresti

Subiectul 1Este suficient sa gasim un astfel de drum care pleaca din coltul stanga sus sise intoarce tot acolo , pentru ca alegand oricare punct facem exact mutarileconsecutive din acest drum considerat.Notam cu (m,n) patratul de pe linia m si coloana n.Pentru n = 2 exista drumul (1, 1)⇒ (1, 2)⇒ (2, 2)⇒ (2, 1)⇒ (1, 1).Presupunem ca pentru n mai mare sau egal cu 2 exista un astfel de drum careincepe cu (1, 1) si are un ultim patrat (n, 1) (dupa care evident ne putem in-toarce in (1, 1)).Atunci pentru un patrat de latura n + 1 consideram drumul care pleaca din(1, 1) , parcurge patratul de latura n care contine pe (1, 1) si are ultimul varf in(1, n) (aici folosind ipoteza de inductie).Atunci consideram succesiunea de mutari:(n, 1)⇒ (1, n+ 1)⇒ (n+ 1, 2)⇒ (2, n+ 1)⇒ ...⇒ (n+ 1, n)⇒ (n, n+ 1)⇒(n+ 1, n+ 1)⇒ (n+ 1, 1)⇒ (1, 1).Astfel am gasit un drum si pentru n + 1 . Asadar din inductie problema esterezolvata.

Subiectul 3Consideram o inversiune de centru X si putere oarecare. Notam cu ′ transfor-matele prin inversiune.Atunci 1

′, 2′ sunt doua drepte perpendiculare pe diametrele care trec prin X

ale cercurilor initiale. Cele doua drepte se intersecteaza in Y ′. Dreapta ABse transforma in cercul A′B′Y ′ care trece prin X. Daca notam cu t tangentacomuna a cercurilor initiale cea mai apropiata de Y si cu S, T puncetele de tan-genta , atunci t′ este un cerc ce trece prin X, este tangent in X la cercul A′B′Y ′

(pentru ca AB∣∣t) si este tangent la 1′, 2

′ respectiv in S′, T ′. Mai mult , XS′

, XT ′ sunt bisectoarele unghiurilor ∕ Y ′XB′ , ∕ Y ′XC ′ , respectiv (1). Cerculde centru O se transforma in cercul inscris in triunghiul A′B′Y ′ .Cercul t′ este tangent laturilor triunghiului si cercului circumscris. Se deduceralativ usor ca O′ ∈ S′T ′ si este chiar mijlocul sau.Mai departe , deoarece Y ′ este intersectia tangentelor la cercul circumscris tri-unghiului XT ′S′ rezulta ca XY ′ este simediana lui X in acest triunghi. Dinacest fapt si (1) rezulta ca XO′ este bisectoare unghiului ∕ A′XB′ , adica XOeste bisectoarea unghiului ∕ AXB.

Unirea

Page 129: olimpiade2009

129

Subiectul 1Se stie ca A2 = aA+ bI2 , unde TrA = a si −det(A) = b. Atunci avem:A4 = a2A2 + b2I2 + 2abA = a(a2 + 2b)A+ (a2b+ b2)I2. (1)Apoi , b4 = (det(A))4 = det(A4) = det(I2) = 1 si b ∈ Z , deci b = ±1; cuma ∈ Z rezulta ca a2 + 2b ∕= 0.Astfel din (1) si A4 = I2 rezulta ca a = 0 sau A = cI2, c ∈ Z ; in ambele cazurireiese A2 = dI2, d ∈ Z.In sfarsit , din I2 = A4 = d2I2 , rezulta d = ±1.

Subiectul 2Egalitatea C2 = 3C rezulta prin calcul direct , iarD3 = BABABA = BC2A = 3BCA = 3BABA = 3D2 (2)Apoi , din rang(C2) = rang(C) = 2siC2 = ABAB = ADA rezulta rand(D) ≥2 , deci rang(D) = 2 , adica matricea D este inversabila. Astfel , inmultind in(2) cu (D−1)2 obtinem D = 3I2.

Subiectul 3Avem x1 = −y0 < 0 = x0 , y1 = y0 + 1 − e−y0 > y0 , astfel , inductiv avemca xn+1 < xn, yn+1 > yn , deci (xn)n≥0 este descrescator , iar (yn)n≥0 estecrescator.In plus , prin adunare , yn = y0 − exn + ex0 < y0 + ex0 = y0 + 1 , deci sirul(yn)n≥0 este marginit superior.Astfel (yn)n≥0 ⇒ y ∈ (0,∞) , iar (xn)n≥0 ⇒ x ∈ [−∞, 0).Prin trecere la kimita in relatia de recurenta , rezulta x = x − y , de undex = −∞. Apoi tot prin trecere la limita , y = y0 + ex0 = y0 + 1.

Subiectul 4Observam ca distanta dintre orice doua intervale consecutive de forma [n� +�/6, n�+(5�/6)] este �/3 < 2, iar un astfel de interval are lungimea (2�)/3 > 1.Astfel, pentru orice k ∈ N , dintre numerele k, k + 1, k + 2 , cel putin unul seafla intr-un interval de aceasta forma.Rezulta

∣n ⋅ sin 1 ⋅ sin2 ⋅ ... ⋅ sinn∣ ≤ n ⋅ sin[n/3]�

6≤ n

2n/3

si dinlimn→∞

n

2n/3= 0

reiese ca limita ceruta este 0.

Gheorghe Lazar

Page 130: olimpiade2009

130

Subiectul 1Avem xn+1 = a+

√a2 − (xn − a)2 = a+(xn−1−a) = xn−1 , iar b = x0 = x1 =

... = x2k = ... si a+√

2ab− b2 = x1 = x3 = ... = x2k+1 = ...

Cum b ∕= a ⋅(

1 +1

2

)⇒ x2n+1 ∕= x2n deci (xn)n≥0 este divergent pentru ca are

2 subsiruri cu limite diferite.Fie y = lim

n→∞yn

zn =1

n

n∑k=0

xkyx + xk+1yk+1

2+x0y0 − xn+1yn+1

2n

tn =xnyn + xn+1yn+1

2are subsirurile t2n si t2n+1 convergente la l = y ⋅(

x0 + x1

2

). Rezulta tn convergent.l

Fie un =t1 + t2 + ...+ tn

n⇒ un convergent cu aceeasi limita l = y ⋅

(x0 + x1

2

).

Cum sirul (xnyn)n≥0 este marginit rezulta limn→∞

xn+1 ⋅ yn+1

2n= 0

Asadar l = limn→∞

zn.

Subiectul 2a) Se observa ca !(f) este continua.

f ∈ C[0, 1] , 0 ≤ 1

2f(x) ≤ 1

2

0 ≤ 2x(1− x) ≤ 1

2⇒ !(f)(x) ∈ [0, 1] i.e. !(f) ∈ C[0, 1].

b) !(ℎ)(x) = 2x(1−x) si !n(ℎ)(x) = �n ⋅x ⋅(1−x) cu �1 = 2 , �n+1 =1

2�n+2.

Rezolvand �n+1 = a ⋅ �n + b, a ∕= 1 avem �n = 4 − z2−n i.e. !n(ℎ)(x) =(4− 22−n)x(1− x).c) lim

n→∞!n(ℎ)(x) = 4x(1− x) , unde 4x(1− x) ∈ C[0, 1].

Subiectul 3∀� > 0,∃�� > 0 a.i. �f(x)− � < f(x+ 1) < �f(x) + �Aratam ca pentru orice k ∈ N∗

�kf(x)− �

�< f(x+ k) < �kf(x) +

(1)Inductie dupa kPentru k = 1 avem:�f(x)− �

�< �f(x)− � < f(x+ 1) < 2f(x) + � < �f(x) +

�.

Presupunem (1) adevarata pentru un k ∈ N∗ si demonstram ca:

Page 131: olimpiade2009

131

�k+1f(x)− �

�< f(x+ k + 1) < �k+1f(x) +

�.

Avem: �k+1f(x)− �

�= �

[�kf(x) +

]+

1− ��

� < �k+1f(x) +�

�.

Cum �k → 0 pentru orice x > ��,∃k ∈ N∗ cu ∣�kf(x)∣ < �.

Asadar −(

1 +1

)� < f(x+ k) <

(1 +

1

)� de unde avem ca lim

x→∞f(x) = 0.

Subiectul 4Se stie ca pentru oricare X,Y ∈M2(R) cu XY = Y X avem det(X2 + Y 2) ≥ 0.Din ecuatia Hamilton-Cayley pentru AB −BA avem:(AB −BA)2 − tr(AB −BA) ⋅ (AB −BA) + det(AB −BA) ⋅ I2 = O2

Cum tr(AB −BA) = 0 ⇒ (AB −BA)2 = det(AB −BA) ⋅ I2.Asadar (AB −BA)2 comuta cu orice matrice , deci si cu (AB +BA)2.In concluzie X = (AB −BA)2 si Y = (AB +BA)2.

Trident

Subiectul 1Notand cu t = sinx , limita din enunt se scrie:

L = limt→0

(t3 + t+ 1)n − 1

tn + 4t=

= limt→0

(t3 + t+ 1− 1)[(t3 + t+ 1)n−1 + (t3 + t+ 1)n−2 + ...+ 1

]tn + 4t

=

= limt→0

t(t2 + 1)[(t3 + t+ 1)n−1 + ...+ 1

]tn + 4t

=n

4

daca n ≥ 2.

Daca n = 1 limita este1

5, iar daca n = 0 limita este 0.

Subiectul 2Din teorema Cayley-Hamilton avem:A2 − tA + dI2 = O2 unde Tr(A) = t,det(A) = d si fA(x) = det(A − xI2) =x2 − x+ d atunciA2 − A + dI2 = O2 ⇒ A2 = A − dI2 ⇒ A3 = A2 − dA ⇒ A3 + A + I2 =A2 − dA+A+ I2 = A− dI2 − dA+A+ I2 = (2− d)A− (d− 1)I2CazulI

Daca detA = d ∕= 2⇒ A3+A+I2 = (2−d)

(A− d− 1

2− dI2

)⇒ det(A3+A+I2) =

(2− d)2f

(d− 1

2− d

)= (2− d)2

[(d− 1

2− d

)2

− d− 1

2− d+ d

]= d3 − 2d2 − d+ 3.

Page 132: olimpiade2009

132

Relatia de demonstrat devine:d(d3 − 2d2 − d+ 3) ≤ 2d4 − 4d3 + d2 + d+ 1⇔ d4 − 2d3 + 2d2 − 2d+ 1 ≥ 0⇔d2(d− 1)2 + (d− 1)2 ≥ 0.CazulIIDaca detA = 2⇒ A3+A+I2 = −I2 atunci relatia devine: 2 ≤ 32−32+4+2+1adevarat.

Subiectul 3Din egalitatea mediilor avem ca:

31

xn+

1

yn+

1

zn

≤ 3√xnynzn ≤

xn + yn + zn3

⇒ zn+1 ≤ yn+1 ≤ xn+1,∀n ∈

N ⇒ zn ≤ yn ≤ xn,∀n ∈ N∗.xn+1 =

xn + yn + zn3

≤ 3xn3

= xn,∀n ∈ N ⇒ (xn) descrescator ⇒ ∃x ∈ [0,∞)

astfel incat xn → x.

zn+1 =3

1

xn+

1

yn+

1

zn

≥ 33

zn

= zn,∀n ∈ N ⇒ (zn) crescator ⇒ ∃z ∈ [0,∞)

astfel incat zn → z.zn < xn ⇒ 0 < z ≤ x ⇒ x, z ∈ (0,∞). Notam cu sn = xn + yn + zn,∀n ∈ N .

Din xn+1 =sn3, yn+1 ≤

sn3

si zn ≤sn3

obtinem sn+1 = xn+1 + yn+1 + zn+1 ≤sn,∀n ∈ N ⇒ (sn) descrescator si pozitiv ⇒ ∃s ∈ [0,∞) astfel incat sn → s.Dar yn = sn− (xn + zn)→ s− (x+ z) si zn ≤ yn ≤ zn ⇒ ∃y ∈ [z, x] astfel incatyn → y.Trecand la limita in relatia de recurenta , obtinem:

x =x+ y + z

3, y = 3

√xyz, z =

31

x+

1

y+

1

z

⇒ x = y = z.

Victor Valcovici

Subiectul 1

Notand cu an = xn + yn avem an+1 =an10

+ 5⇔ an+1 −50

9=an −

50

910

⇒(an −

50

9

)n≥1

progresie geometrica cu ratia1

10⇒(an −

50

9

)n≥1

convergent la 0⇒ (an)n≥1 convergent la50

9.

Notand cu bn = xn − yn avem bn+1 =−bn10

+ 1⇔

bn+1−10

11= −

bn −10

1110

⇒(bn −

10

11

)n≥1

progresie geometrica cu ratia − 1

10⇒

Page 133: olimpiade2009

133

⇒(bn −

10

11

)n≥1

convergent la 0⇒ (bn)n≥1 convergent la10

11.

Atunci xn =an + bn

2este convergent la

50

9+

10

112

=320

99.

De asemenea yn =an − bn

2este convergent la

50

9− 10

112

=230

99.

Subiectul 2Avem 3AB2 = 2B2A+ In (1)

Din 3(AB2 −B2A) = In −B2A⇒ det(AB2 −B2A) =1

3ndet(In −B2A) (2)

Din (1) obtinem 2(AB2 −B2A) = In −AB2 ⇒ det(AB2 −B2A) =

=1

2ndet(In −AB2) (3)

Din egalitatea det(In +XY ) = det(In + Y X) rezulta det(In −AB2) == det(In −B2A) (4)

Din (2) , (3) , (4) avem

(1

2n− 1

3n

)det(In −AB2) = 0⇒

⇒ det(AB2 −B2A) = 0.

Subiectul 3Din ∣f ′(x)∣ ≤ 7f(x) avem ∀x ∈ R, f(x) > 0. Vom arata ca f(x) = 0,∀x ∈ R.Presupunem prin reducere la absurd ca exista b ∈ R astfel incat f(b) ∕= 0.Sa presupunem, fara a restrange generalitatea, ca b ≥ 3.Fie multimea A = {x ∈ R/x ≥ 3, f(x) > 0. Deoarece b ∈ A rezulta ca multimeaA este nevida si este minorata de numarul 3. Notam M = infA.Daca M = 3 atunci f(M) = 0.Daca M > 3 vom demonstra ca f(M) = 0.Din alegerea lui M avem f(x) = 0,∀x ∈ [3,M).Deoarece f este derivabila rezulta ca f este continua pe R.Fie un sir arbitrar (xn)n∈N cu xn ∈ [3,M) si lim

n→∞xn = M .

Trecand la limita in egalitatea f(xn) = 0 si folosind continuitatea functiei favem: lim

n→∞f(xn) = f( lim

n→∞xn) = f(M) = 0.

Din alegerea lui M rezulta ca exista d > M astfel incat f(x) > 0,∀x ∈ (M,d).Deoarece f ′ este continua si nenula pe o vecinatate (M, s) ⊂ (M,d) rezulta caf ′(x) > 0,∀x ∈ (M, s) , astfel ar rezulta f strict descrescatoare pe [M, s) , adicaf(x) < 0,∀x ∈ (M, s).Inmultind inegalitatea ∀x ∈ [M, s), f ′(x) ≤ 7f(x) cu e−7x obtinem:(f(x)e−7x)′ ≤ 0,∀x ∈ [M, s) , adica functia g(x) = f(x)e−7x este descrescatoarepe [M, s).Avem ∀x ∈ (M, s), g(M) ≥ g(x), 0 ≥ f(x)e−7x, 0 ≥ f(x). Aceasta contradictieincheie demonstratia.

Page 134: olimpiade2009

134

Vasile Dumitrache

Subiectul 1a) Cum Tr(A) = 0,det(A) = 1 , din Cayley-Hamilton avem ca: A2 = −I2.Astfel avem:A2009 = A ⋅A2008 = A ⋅ (A2)1004 = A.b) det(A+ �B)n = det(A− �B)n ⇒ (det(A+ �B))n = (det(A− �B))n

dar det(A+�B) = (cos�+� sin�)2 +(sin�+� cos�)2 = 1+2� sin 2�+�2 ≥ 0iar det(A− �B) = 1− 2� sin 2�+ �2 ≥ 0.Atunci:(det(A+ �B))n = (det(A− �B))n,∀� ∈ R⇒ 4� sin 2� = 0,∀� ∈ R⇒

⇒ sin 2� = 0⇒ � =k�

2, k ∈ Z.

Asadar:

i) pentru � ∈{k�

2∣k ∈ Z

}egalitatea are loc ∀n ∈ N∗,∀� ∈ R;

ii) pentru � /∈{k�

2∣k ∈ Z

}nu exista n ∈ N∗ cu proprietatea ceruta.

Subiectul 2k2 < k2 + k < (k + 1)2,∀k ≥ 1⇒ [

√k2 + k] = k.

f(k2 + k) = (k2 + k)k ⇒ xn =

n∑k=1

(k3 + k2) =

[n(n+ 1)

2

]2

+n(n+ 1)(2n+ 1)

6

⇒ xn =n(n+ 1)(3n2 + 7n+ 2)

12.

Atunci:

limn→∞

(4xnn4

)n= limn→∞

[n(n+ 1)(3n2 + 7n+ 2)

3n4

]n=

= limn→∞

[(n+ 1

n

)n(3n2 + 7n+ 2

3n2

)n]=

= e ⋅ limn→∞

⎡⎢⎢⎣(1 +7n+ 2

3n2

) 3n2

7n+ 2

⎤⎥⎥⎦7n+ 2

3n

=

= e ⋅ e7

3 = e

10

3

Page 135: olimpiade2009

135

.

Subiectul 3a) Daca presupunem prin absurd ca f este marginita inferior, atunci existam ∈ R cu f(x) ≥ m,∀x ∈ R , de unde rezulta: f(x) + x ≥ m+ x, ∀x ∈ R.Trecand la limita in ultima relatie pentru x→∞ obtinem lim

x→∞(f(x) + x) =∞

ceea ce contrazice ipoteza.Asadar f nu este marginita inferior si in mod analog se arata ca f nu estemarginita superior.b) Daca presupunem ca functia f + 1R nu ar fi constanta , atunci cum ea esteperiodica nu ar exista lim

x→±∞f(x) , ceea ce ar contrazice ipoteza.

In concluzie f + 1R este constanta , adica exista a ∈ R cu proprietatea :f(x) = −x+ a,∀x ∈ R.

4.8.1 Olimpiada judeteana

Subiectul 1.Fie functia f : R→ R, f(x) = det(A+xB)−det(C = xA).Aceastafunctie este polinomiala si are gradul cel mult egal cu 3..Din ipoteza , coeficientul lui x3 este det(B) − det(A) = 0, f(0) = det(A) −det(C) = 0 si f(i) = det(A+ iB)− det(C + iA) = 0.Rezulta ca functia este deforma f(x) = ax2 + bx cu −a+ bi = 0, a, b ∈ R,deci a = b = 0, f(x) = 0,∨x ∈ Rsi det(A+B) = det(C +A) = f(1) = 0.

Subiectul 2.Aratam ca, daca n ≥ 3 atunci orice minor de ordin 3 estenul.Luand i = j = k rezulta aii = 0,∨i = 1, 2, ..., n,iar k = l ⇒ aij + aji =0,∨i, j = 1, 2, ..., n.Sa consideram un minor oarecare D de ordin 3,format cu elemente de pe liniilei, j, k si coloanele p, q, r.In D, scazand prima linie din a doua obtinem liniaL = (ajp − aip, ajq − aiq, ajr − air).Avem ajs − ais = ajs + asi = −aij pentrus = p, q, r, deci linia L are elementele egale .Analog,linia obtinuta prin scadereaprimei linii din linia a treia are elementele egale.De aici,D = 0.

Subiectul 3.Avem,inductiv, 1 ≤ xn ≤ 2,∨n ∈ N∗ si xn ≥ xn+1,∨n ∈ N∗,deci sirul (xn)n are o limita L ≥ 1 care verifica relatia L =

√L, deci L = 1.

Pentru a doua limita , scriem xnn = uvnn , cu un = (1 + xn − 1)1

xn−1 → e ,iarpentru limita sirului vn = n(xn − 1) observam inductiv ca xn ≥ 1 + 1

n ,∨n ≥ 5

Page 136: olimpiade2009

136

si xn < 1 + 1n−4 ,∨n ≥ 5,deci ,din teorema clestelui,vn → 1.

Subiectul 4.a) Deoarece functia [.] este continua pe R∖Z ,F este pro-dus,compunere si diferenta de functii continue,deci este continua pe R∖Z.Inpunctele a ∈ Z, lim

x↘aF (x) = 2a− cos 0 = 2a− 1, lim

x↗aF (x) = 2(a− 1)− cos 3� =

2a − 1, F (a) = 2a − 1,deci F este continua.In plus,daca y /∈ 2Z + 1,ecuatiaf(x) = y are trei solutii ,situate in intervalul ([y+1

2 ], [y+12 ]+1) ,iar daca y ∈ 2Z+1

ecuatia f(x) = y are solutiile y+12 , 3y+7

6 , 3y−16 .

b) Sa presupunem ca exista o astfel de functie .Fie � ∈ Imf si x1 < x2 < ... < x2k

solutiile ecuatiei f(x) = �.Atunci, pe fiecare din intervalele I0 = (−∞, x1), I1 =(x1, x2), ..., I2k = (x2k,∞) diferenta d(x) = f(x)− � are semn constant.Daca d(x) are semnul ′+′ pe I0 si I2k, atunci f este marginita inferior pe fiecaredintre intervalele I0, I1, ..., I2k,deci pe R.In plus , marginea inferioara m = inffeste atinsa in [x0, x2k].Fie t1 < t2 < .. < t2k punctele in care se atinge margineainferioara .In acest caz,alegand vecinatati disjuncte V1, V2, ..., V2k , ale punctelort1, t2 <, ..., t2k si y > m , suficient de apropiat de m , gasim in fiecare Vi catedoua solutii ale ecuatiei f(x) = y.Rezulta astfel cel putin 4k solutii ale ecuatieif(x) = y contradictie.Cazul cand d(x) are semnul ′−′ pe I0 si pe I2k se trateazaanalog.Daca d(x) are semne contrare pe I0 si I2k,pe cel putin k dintre cele 2k − 1intervale I1, ..., I2k−1 functia d(x) are acelasi semn ,de exemplu ′+′.In acestcaz,pentru y > � ,suficient de apropiat de � , ecuatia f(x) = y are cel putin 2ksolutii pe [x0, x2k] si o solutie pe I0 sau I2k contradictie.

4.8.2 Olimpiada nationala

Subiectul 1.a) Fie In intervalul inchis cu capetele xn, yn.Rezulta imediat ca... ⊂ In ⊂ In−1 ⊂ ... ⊂ I1 ⊂ I0, incluziunile fiind stricte .Notand cu ln lungimea intervalului In , avem ln = ∣xn − yn∣.Prin inductie deducem ln = ∣(2t0 − 1)(2t1 − 1)...(2tn − 1)∣∣x0 − y0∣. Daca exista

n0 cu tn0= 1

2 avem ln = 0 pentru n ≥ n0.Altfel avem ln+1

ln= ∣2tn+1−1∣ ∈ (0, 1)

si criteriul raportului ne da limn→∞

ln = 0.Conform axiomei lui Cantor exista un

unic a0 ∈ ∩In.Cum xn si yn sunt, pentru fiecare n, simetrice fata de mijloculintervalului I0 , rezulta ca a0 = x0+y0

2 este limita comuna a sirurilor (xn)n si(yn)n.b) De exemplu se poate lua tn = 1

2(n+1)2

Subiectul 2.Fie lx = limℎ→0∣f(x+ ℎ)− f(x)

ℎ∣. Daca lx = 0 , f este evident

derivabila in x si derivata este zero.Daca lx > 0 , sa presupunem de exemplu , ca f nu este derivabila la dreapta in

Page 137: olimpiade2009

137

x . Exista atunci siruri (un)n, (vn)n , cu termeni pozitivi , convergente la 0 , si

astfel ca limn→∞

f(x+ un)− f(x)

un= −lx lim

n→∞

f(x+ vn)− f(x)

vn= lx prin urmare

functia definita prin '(t) = f(x+t)−f(x)t isi schimba semnul de o infinitate de ori

pe orice vecinatate la dreapta a lui x.Din proprietatea valorii intermediare , ' este continua , rezulta ca exista unsir (ℎn)n cu termeni pozitivi ,convergent la 0, cu f(x + ℎn) − f(x) = 0, incontradictie cu ipoteza.Derivabilitatea la stanga se demonstreaza analog iar faptul ca numerele derivatesunt opuse este evident.

Subiectul 3. a) Functia f(z) = det(A+ zB) = a0 + a1z+ a2z2 + ...+ anz

n

este polinomiala de grad n(an = detB ∕= 0).Conditia ∣det(A+ zB)∣ = 1 pentru orice z , cu ∣z∣ = 1 se scrie f(z) ⋅ (f(z)) = 1, pentru orice z cu z = 1

z .

Avem (a0 + a1z + a2z2 + ... + anz

n)(a0 + a1z + a2z2 + ... + anzn) = 1 ⇔(a0 + a1z + a2z

2 + ...+ anzn)(a0z

n + a1zn−1 + a2z

n−2 + ...+ an) = zn

Ultima egalitate avand loc pentru o infinitate de valori ale lui z , ea esteidentitate de polinoame . Prin identificarea coeficientilor obtinem succesiv ,a0 ⋅an = 0, a1 ⋅an = 0, a2 ⋅an = 0, ..., an−1 ⋅an = 0 , deci a0 = a1 = ... = an−1 = 0si an ⋅ an = 1 (adica ∣detB∣ = 1).In concluzie f(z) = anz

n adica det(A + zB) = detB ⋅ zn sau det[B(B−1 ⋅A + zIn)] = detB ⋅ zn adica detB ⋅ det(B−1 ⋅ A + z ⋅ In) = detB ⋅ zn , decidet(B−1 ⋅ A + z ⋅ In) = zn cu ℎ(z) = zn care este polinomul caracteristic almatricei C = B−1 ⋅ A.Conform teoremei Cayley-Hamilton (−B−1 ⋅ A)n = 0 deunde An = 0.b) Conditia A⋅B = B ⋅A este necesara dupa cum se vede din urmatorul exemplu:

A =

⎛⎜⎜⎜⎜⎝0 ... ... 0

1...

0 1

⎞⎟⎟⎟⎟⎠,

B =

⎛⎜⎜⎜⎜⎝0 ... ... 11 . 0

...0 1 0

⎞⎟⎟⎟⎟⎠A ⋅ B ∕= B ⋅ A , det(A+ zB) = (−1)n+1 ⋅ zn , deci ∣det(A+ zB)∣ = 1 si evidentAn = A ∕= 0.

Subiectul 4.Prin inmultire cu e−x relatia data se scrie :

Page 138: olimpiade2009

138

(e−xf(x))′ = e−xg(x) + e−xℎ(x).Notam cu g1 respectiv ℎ1 functiile din membrul drept.Functiile g1 si ℎ1 au limitelaterale in fiecare punct deci si u = g1 + ℎ1. Cum u are in plus proprietatea luiDarboux , fiind o derivata , rezulta ca u este continua pe R.Notand Dg multimea punctelor de discontinuitate ale functiei g avem Dg =Dg1 , Dℎ = Dℎ1 .Daca prin absurd Dg1 ∕= Dℎ1 pentru x ∈ Dg1∖Dℎ1 rezultax ∈ Dg1+ℎ1

, fals.

4.9 Clasa XII

Cluj

1. a) f−1(e) = f−1(2) ⇒ e=2 si a−1 = f(2q − f−1(a));

b) q = 3√

2; se ajunge la ecuatia:3√x2 + 3

√x − 6 = 0 notam 3

√x = t ⇒

t1 = 2⇒ x1 = 8 si t = −3⇒ x2 = −27

2. det(x3) = 0⇒ detX=0; fie X =

(a b

c d

)Aplicam ecuatia Cayley-Hamilton ⇒ X3 = (a+ d)2⋅X ⇒ X =

(4 3

2 4

).

3. a) fie f : [0,∞)→ R, f(x)=x-ln(1+x);

f ′(x) =x

1 + x≥ 0, ∀x ∈ [0,∞)⇒ f este strict crescatoare pe [0,∞)⇒ f(x) ≥ 0

b) Folosind inegalitatea de mai sus si faptul ca 1+x4 > 0⇒ ln(1 + x)

1 + x4≤ x

1 + x4;

Integrand de la 0 la 1, obtinem inegalitatea din enunt.

4. Presupunem ca exista functii cu proprietatea din enunt. Derivand relatiadata, obtinem:p∑k=1

kf(1 + kx) +

p∑k=1

kf(1− kx) = 2nx2n−1f(x2n), ∀x ∈ R;

Daca punem x=0, obtinem:p(p+1)f(1)=0 ⇒ f(1)=0 ceea ce este fals pentru ca f(x) ∕= 0, ∀x ∈ RDeci nu exista functii cu proprietatea din enunt.

Page 139: olimpiade2009

139

Dolj

1. a) Fie a ∈ G/H. Atunci a2 = e, de unde rezulta a = a−1.Daca x ∈ H, atunci a⋅x ∈ G/H (alftel a ∈ H), deci (a⋅x)2 = e si, cu ajutorulrelatiei de mai sus, rezulta

x⋅ a⋅x = a. (1)

Fie x, y ∈ H arbitrare. Avem

x⋅ y = x⋅ a⋅ a⋅ y = (x⋅ a⋅x)⋅x−1⋅ y−1⋅ (y⋅ a⋅ y).

Folosind (1) obtinem

x⋅ y = a⋅x−1⋅ y−1⋅ a,

apoi

a⋅x⋅ y = x−1⋅ y−1⋅ a,

de unde

(x⋅ y)⋅ a⋅ (x⋅ y) = x⋅ y⋅x−1⋅ y−1⋅ a

Folosind din nou (1), deducem

a = x⋅ y⋅x−1⋅ y−1⋅ a

de unde rezulta

x⋅ y⋅x−1⋅ y−1 = e

sau, echivalent,

x⋅ y = y⋅x.

Page 140: olimpiade2009

140

b) Daca pentru un a ∈ G/H si un x ∈ H am avea a ⋅ x = x ⋅ a, atunci,folosind (1), am obtine x⋅x⋅ a = a, adica x2 = e, ceea ce ar insemna x ∈ G/H,fals. Asadar, a⋅x ∕= x⋅ a si grupul G nu este comutativ.

2. Uk =

⎛⎝ Fk 0 Fk+1

0 1 0Fk+1 0 Fk+2

⎞⎠, k ∈ N . Se arata ca:

UkUl = Uk+l, ∀k, l ∈ N , ceea ce este echivalent cu:

FkFl + Fk+1Fl+1 = Fk+l, ∀k, l ∈ N .

Fixam l ∈ N si demonstram prin inductie matematica dupa k ≥ 0.Pentru k=0 avem F0Fl + F1Fl+1 = Fl = F0+l. Presupunem adevarata relatiapentru k si k+1 si o demonstram pentru k+2. Adunand membru cu membrurelatiile

FkFl + Fk+1Fl+1 = Fk+l,Fk+1Fl + Fk+2Fl+1 = Fk+l+1,

obtinem

(Fk + Fk+1)Fl + (Fk+1 + Fk+2)Fl+1 = Fk+l+ Fk+l+1

Folosind relatia de recurenta din ipoteza, relatia (1) se scrie:

Fk+2Fl + Fk+3Fl+1 = Fk+l+2,

adica ce trebuia demonstrat.Se verifica acum imediat ca (M, ⋅ ) este monoid comutativ.

3. Avem ca:

∫ex(x− �)

x(x� + ex)dx =

∫ex(x� − �x�−1)

x�(x� + ex)dx

Consideram functiile f,g : (0,∞)→ R,

Page 141: olimpiade2009

141

f(x) = x�, g(x) = x� + ex.

Avem ca f(x)g′(x)− g(x)f ′(x) = ex(x� − �x�−1). Astfel rezulta ca

∫ex(x� − �x�−1)

x�(x� + ex)dx =

∫f(x)g′(x)− g(x)f ′(x)

f(x)g(x)dx =

=

∫g′(x)

g(x)dx -

∫f ′(x)

f(x)dx = lng(x)− lnf(x) + C = ln(1 +

ex

x�) + C.

4. Functia a� + b� este strict monotona, de exemplu, pentru a = b = 1.Deoarece � + � este si functie polinomiala de gradul 3, ea este bijectiva. Cumfunctia g := (a� + b�) ∘ f admite primitive si este injectiva, ea este continua.Prin urmare, f = (�+ �)−1 ∘ g este continua.

Galati

1. Din calcul se obtine: x∗y = k+(x−k)loga(y−k), x, y ∈ G = (k,∞)−{k+1},k > 0;Din e ∈ G element neutru ⇒ x ∗ e = e ∗ x = x, (∀)x ∈ Gx ∗ e = x ⇔ k + (x− k)loga(e−k) = x⇔ (x− k)loga(e−k) = x− k ⇔a) loga(e− k) = 1⇔ e− k = a⇔ e = k + a ∈ G.k+a = 2009, k, a ∈ Z, k > 0⇒ (k, a) ∈ {(1, 2008), (2, 2007), (3, 2006), ..., (2008, 1)}.Numarul de perechi este 2008.

b) Se verifica asociativitatea;k=1005, a=1004

x ∗ x′ = e⇔ x′ = k + aloga(e−k)loga(k−k) ∈ G.

Pentru x=x’ obtinem: log21004(x− 1005) = 1⇒

I) log1004(x− 1005) = 1⇒ x1 = 2009 ∈ GII) log1004(x− 1005) = −1⇒ x2 = 1005 +

1

1004∈ G

c) Fie f : R− {0} → G, f(x) = k + ax, k > 0, a > 0, a ∕= 1.Se verifica faptul ca functia f este izomorfism de grupuri.Dreapta y=1 este asimptota orizontala spre −∞⇔ lim

x→−∞(k+ax) = 1⇔ k = 1;

Page 142: olimpiade2009

142

f(1) = 2010⇔ 1 + a = 2010⇔ a = 2009.f(x) = 1 + 2009x, x ∈ R− {0}.

2. Pentru x=e, relatia din enunt devine: a2⋅ e5⋅ a2 = e ⇔ a2⋅ e⋅ a2 = e ⇔a4 = e.

Prin compunere la stanga si la dreapta cu a2, relatia din ipoteza devine

a2⋅ (a2⋅x5⋅ a2)⋅ a2 = a2⋅x⋅ a2 ⇔ a4⋅x5⋅ a4 = a2⋅x⋅ a2 ⇔ x5 = a2⋅x⋅ a2,(∀)x ∈ G (1)

Inlocuind in relatia (1) pe x cu x⋅ y, unde x, y ∈ G sunt arbitrare, obtinem:

(x⋅ y)5 = a2⋅ (x⋅ y)⋅ a2 ⇔ (x⋅ y)⋅ (x⋅ y)⋅ (x⋅ y)⋅ (x⋅ y)⋅ (x⋅ y) = a2⋅ (x⋅ a4⋅ y)⋅ a2 ⇔

x⋅ (y⋅x⋅ y⋅x⋅ y⋅x⋅ y⋅x)⋅ y = (a2⋅x⋅ a2)⋅ (a2⋅ y⋅ a2)⇔

x⋅ (y⋅x⋅ y⋅x⋅ y⋅x⋅ y⋅x)⋅ y = x5⋅ y5 ⇔

x⋅ (y⋅x⋅ y⋅x⋅ y⋅x⋅ y⋅x)⋅ y = x⋅ (x4⋅ y4)⋅ y ⇔

y⋅x⋅ y⋅x⋅ y⋅x⋅ y⋅x = x4⋅ y4, (∀)x, y ∈ G

In ultima egalitate, punand y = a2, obtinem:

a2xa2xa2xa2x = x4a4 ⇔ (a2xa2)x(a2xa2)x = x4 ⇔ x5xx5x = x4 ⇔ x4x8 =x4 ⇔ x8 = e, (∀)x ∈ G.

3. I =

∫sinx− x cosx

x2 − sin2 xdx =

∫ sinx− x cosx

sin2xx2 − sin2 x

sin2x

dx =1

2⋅ ln

∣∣∣∣∣∣x

sinx− 1

x

sinx+ 1

∣∣∣∣∣∣ + C

Page 143: olimpiade2009

143

=1

2⋅ ln∣∣∣∣x− sinx

x+ sinx

∣∣∣∣ + C.

4. I2 =

∫ 1

0

√1 + x+ x2dx =

∫ 1

0

(x+

1

2

)2

+3

4√(x+

1

2

)2

+3

4

dx=

=

∫ 1

0

(x+

1

2

)2

√(x+

1

2

)2

+3

4

dx +3

4

∫ 1

0

1√(x+

1

2

)2

+3

4

dx =

=

∫ 1

0

(x+

1

2

)⋅

(x+

1

2

)√(

x+1

2

)2

+3

4

dx +3

4ln

(x+

1

2+√

1 + x+ x2

)∣10 =

=

∫ 1

0

(x+

1

2

)⋅(√

1 + x+ x2)′dx +

3

4ln

(x+

1

2+√

1 + x+ x2

)∣10 =

=

(x+

1

2

)⋅(√

1 + x+ x2)∣10 -

∫ 1

0

√1 + x+ x2 +

3

4ln

(x+

1

2+√

1 + x+ x2

)∣10 =

=3

2⋅√

3− 1

2− I2 +

3

4ln

(3

2+√

3

)− 3

4ln

3

2.

a) I2 =1

2⋅

(3√

3− 1

2+

3

4ln

3 + 2√

3

3

).

b)Pentru x ∈ [0, 1] si n ≥ 2, se obtine:

∣∣ n√1 + x+ xn − n√

1 + x∣∣ =

=

∣∣∣∣ xn

( n√

1 + x+ xn)n−1 + ( n√

1 + x+ xn)n−2⋅ ( n√

1 + x) + ...+ ( n√

1 + x)n−1

∣∣∣∣ ≤

Page 144: olimpiade2009

144

≤ xn, de unde rezulta n√

1 + x− xn ≤ n√

1 + x+ xn ≤ n√

1 + x+ xn.

Integrand aceste inegalitati pe [0,1], se obtine:

21n + 11

n+ 1

− 11

n+ 1− 1

n+ 1≤ In ≤

21n + 11

n+ 1

− 11

n+ 1

+1

n+ 1.

Trecand la limita, se obtine: In → 1.

Sibiu

1. a) A⋅B =

(u 00 v

)⋅(x 00 y

)=

(ux 00 vy

)∈ G, deoarece produsul

are forma matricelor din G si din A⋅B = B⋅A avem (A⋅B)2009 = A2009⋅B2009 == I2;b) Se verifica axiomele grupului: asociativitatea, existenta elementului neutru,orice element este simetrizabil, comutativitatea;

c) A2009 =

(u2009 0

0 v2009

), si din A2009 = I2 rezulta ca u si v sunt radacini

complexe de ordinul 2009 ale unitatii, deci in G sunt 2009 ⋅ 2009 elemente.

2. a) Avem

f : [0, 2�]→ R, f(x)=

⎧⎨⎩x, x ∈ [0,

2]

� − x, x ∈ (�

2,

3�

2)

x− 2�, x ∈ [3�

2, 2�]

F : [0, 2�]→ R, F(x)=

⎧⎨⎩

x2

2+ c1, x ∈ [0,

2]

�x− x2

2+ c2, x ∈ (

2,

3�

2)

x2

2− 2�x+ c3, x ∈ [

3�

2, 2�]

Din conditia de continuitate a primitivei in punctele�

2si

3�

2se obtine o prim-

itiva.

Page 145: olimpiade2009

145

b) Fie Sn =�

3n(sin

3n+ sin

2�

3n+ ...+ sin

(n− 1)�

3n) =

=

n∑i=1

sin(i− 1)�

3n(i�

3n− (i− 1)�

3n) = �Δn

(f, �ni ),

unde f(x) = sinx, f : [0,�

3]→ R o functie continua, deci integrabila;

Δn =

(0,

3n,

2�

3n, ...,

(n− 1)�

3n,n�

3n=�

3

)iar �ni =

(i− 1)�

3n, i=1,...,n ,

⇒ limn→∞

Sn = limn→∞

�Δn(f, �ni ) =

∫ �3

0

sinxdx = − cosx∣�30 =

1

2.

3. Fie G = {e, a, x1, x2, ..., xn}.Presupunem ca a2 ∕= e⇒ a ∕= a−1, a ∈ G ⇒ a−1 ∈ {e, x1, x2, ..., xn}.Deci (a−1)2 = e si a2 = e, contradictie cu ipoteza, deci a2 = e.

4. F : [a, b]→ R, cu F(t) =

∫ t

a

(g(x)− a)f(x)dx -

∫ b

t

(b− g(x))f(x)dx;

F(a)=−∫ b

a

(b− g(x))f(x)dx si F(b)=

∫ b

a

(g(x)− a)f(x)dx;

Dar F (a)⋅F (b) ≤ 0 si F-continua pe [a,b], atunci exista c ∈ [a, b] astfel incatF(c)=0;

deci

∫ c

a

(g(x)− a)f(x)dx -

∫ b

c

(b− g(x))f(x)dx = 0,

deci

∫ b

a

g(x)⋅ f(x)dx = a

∫ c

a

f(x) + b

∫ b

c

g(x).

Page 146: olimpiade2009

146

Timis

1. f(x)=1

8⋅ 8 cos3 x⋅ cos 8x = =

1

8⋅ 4 cosx⋅ cos 8x⋅ 2 cos2 x =

=1

8⋅ 4 cosx⋅ cos 8x(cos 2x+ 1) =

1

82 cos 8x(2 cos 2x⋅ cosx+ 2 cosx) =

=1

8⋅ 2 cos 8x(cos 3x+ cosx+ 2 cosx) =

1

8(2 cos 8x⋅ cos 3x+ 6 cos 8x⋅ cosx) =

=1

8[cos 5x+ cos 11x+ 3(cos 7x+ cos 9x)].

Atunci

∫f(x)dx =

1

8

∫(cos 5x+ 3 cos 7x+ 3 cos 9x+ cos 11x)dx =

=1

8

(sin 5x

5+

3 sin 7x

7+

3 sin 9x

9+

sin 11x

11

)+ C =

=1

40sin 5x+

3

56sin 7x+

1

24sin 9x+

1

88sin 11x+ C.

2.a)Aa,b⋅Aa′,b′ =

⎛⎝ 0 a 00 a 0a b a

⎞⎠⋅⎛⎝ 0 a′ 0

0 a′ 0a′ b′ a′

⎞⎠=

⎛⎝ 0 aa′ 00 aa′ 0aa′ (a+ b)a′ + ab′ aa′

⎞⎠ =

= Aaa′,(a+b)a′+ab′ ∈ G, deoarece aa′ ∈ R daca a ∈ R, b ∈ R∗ si (a + b)a′ +ab′ ∈ R.

b) Se verifica axiomele grupului comutativ.

c) A2a,b = Aa,b⋅Aa,b = Aa2,a2+2ab

A3a,b = Aa,b⋅Aa2,a2+2ab = Aa3,2a3+3a2b, etc.

Page 147: olimpiade2009

147

Ana,b =

⎛⎝ 0 an 00 an 0an (n− 1)an + nan−1b an

⎞⎠ inductie dupa n ∈ N∗.

d) A1,2⋅X = A2,1/⋅A′

1,2

X = A′

1,2⋅A2,1

Dar A′

1,2 = A1,−4. Atunci X = A1,−4⋅A2,1 = A2,−5 =

⎛⎝ 0 2 00 2 02 −5 2

⎞⎠.

3. Cu identitatile din enunt, folosind asociativitatea inmultirii din grupul G,rezulta ca:

a2 = a⋅ a = a⋅ bab = aba⋅ b = b⋅ b = b2.

Atunci

a−1 = a−2⋅ a = b−2⋅ bab = b−1ab

si obtinem ca

a−2 = (a−1)2 = (b−1ab)2 = b−1a2b = b−1b2b = b2 = a2,

de unde rezulta

a4 = a2⋅ a2 = a2⋅ a−2 = u.

4. Pentru orice y ∈ R, ecuatia f(x)=y, cu x ∈ R, se poate scrie echivalent:

Page 148: olimpiade2009

148

f(x) = y ⇔ ex − e−x = 2y ⇔ e2x − 2yex − 1 = 0 ⇔ ex = y +√y2 + 1 ⇔

x = ln(y +√y2 + 1).

Prin urmare, pentru orice y ∈ R, ecuatia f(x)=y are o unica solutie x ∈ R,

data de x = ln(y +√y2 + 1). Rezulta ca functia f este bijectiva, iar functia sa

inversa f−1 : R→ R este definita prin f−1(x) = ln(x+√x2 + 1)

Prin calcul direct avem ca f’=g si g’=f. De asemenea, pentru orice numarreal x ∈ R avem ca:

g2(x)− f2(x) =1

4((ex + e−x)2 − (ex − e−x)2) =

1

4⋅ 2ex⋅ 2e−x = 1.

Fara a folosi tabelul de primitive, folosind proprietatile functiilor derivabile,avem:

(f−1)′(y) =1

f ′(f−1(y))=

1

cℎ(f−1(y))=

1√sℎ2(f−1(y)) + 1

=1√y2 + 1

,

(∀)y ∈ R, astfel ca:

f−1 ∈∫

1√x2 + 1

dx.

Vaslui

1.a) (x ∗ y) ∗ z = ((x− a)(y − a) + c− a) (z − a) + c;x ∗ (y ∗ z) = (x− a)((y − a)(z − a) + c− a) + c⇒ legea de compozitie ” ∗ ” este asociativa daca si numai daca a=c.

b) (Ga,b, ∗)− grup→ c = a, e = a+ 1→ b > a+ 1;

x′ = a+1

x− a∈ Ga,b ⇒ b > x′;

x→ a⇒ x′ →∞⇒ b→∞.

2. Fie I =

∫sinx+ 2 cosx

3 + sin 2xdx si J =

∫2 sinx+ cosx

3 + sin 2xdx;

Page 149: olimpiade2009

149

I + J =3

2ln

2 + sinx− cosx

2− sinx+ cosx+ C

I − J =1√2arctg

sinx+ cosx√2

+ C

⇒ I =1

2

(3

2ln

2 + sinx− cosx

2− sinx+ cosx+

1√2arctg

sinx+ cosx√2

)+ C.

3. a) G =

(−1

2, 1

);

f−1(x) =1 + 2x

1− x; Aplicand conditia de morfism pentru f−1, obtinem:

xΔy = f

(1 + 2x+ 2y + 4xy

1− x− y + xy

)⇒ xΔy =

x+ y + xy

1 + 2xy.

b) Fie an = xΔxΔ...Δx︸ ︷︷ ︸n ori

, n ∈ N∗ ⇒ f−1(an) =

(1 + 2x

1− x

)n⇒ an =

(1 + 2x)n − (1− x)n

(1 + 2x)n + 2(1− x)n.

4. f - periodica → f(x+ T ) = f(x), (∀)x ∈ R;

Fie F - primitiva a lui f, g : R→ R, g(x) =

∫ x+T

x

f(t)dt;

g - derivabila; g(x) = F (x+ T )− F (x), (∀)x ∈ R; g′(x) = 0 → g - constanta →concluzia.

Al VII-lea Concurs National de Matematica ”Alexandru Myller”

1. Schimbarea de variabila t=nx duce la:1

n

∫ n

0

t

(f(t)

g(t)− 1

)dt;

Limita de mai sus gandita ca limita de functii (de forma 0/0), cu regula l’Hospital,

devine: limu→∞

u

(f(u)

g(u)− 1

);

Daca f = Xk+a1Xk−1+..., g = Xk+b1X

k−1+..., limita de mai sus este a1−b1.

Page 150: olimpiade2009

150

2.a) Rezulta din identitatea (1+x)(1−x+x2−...+xk−1) = 0, pentru k imparb) Fie p numarul elementelor inversabile si N multimea elementelor neinversabile.Cum functia f : N → A, f(x)=1+x este injectiva, rezulta din a) ca N are celmult p elemente. Atunci A are exact 2p elemente;Daca p > 2 rezulta ca A este izomorf cu Z2p care are p-1 elemente inversabile,fals.

3. Functia f fiind continua, fie F o primitiva a sa. Rezulta ca F(arctg x) =F(x) + c, cu c - constanta, iar valoarea in 0 da c = 0. Asadar F(arctg x) = F(x)pentru orice x ∈ R;Consideram g(x)=arctg x. Se arata ca sirul (xn)n definit prin x0 ∕= 0 sixn+1 = g(xn) converge la 0.Rezulta F (xn) = F (x0) deci F (0) = F (x0) prin urmare F(x)=F(0) pentru oricex ∈ R. De aici f=0.

4. Fie f = aXp−1 + ... privit ca o functie polinomiala in Zp.Conditia din enunt este echivalenta cu faptul ca f este injectiva (∈ Zp), decibijectiva.Rezulta ca f(0) + f(1) + ...+ f(p− 1) = 0 + 1 + ...+ p− 1 = 0. Pe de alta parte

f(0) + f(1) + ...+ f(p− 1) = (p− 1)a. Rezulta p∣a.

Concursul de Matematica ”UNIREA”, Editia a IX-a

1. a) Evident ord a = 3 si ord b = 2 si elementele e, a, a2 sunt distincte iarb /∈

{e, a, a2

}(ord b = 2, ord e = 1, ord a2 = 3 = ord a).

Elementele b, ba, ba2 sunt distincte si fiecare diferit de e, a, a2 (de exemplu, prinabsurd, din ba = a2, prin simplificare am deduce b=a).Rezulta ca prezentarea lui G este:

G ={e, a, a2, b, ba, ba2

}si cum c /∈

{e, a, a2, b

}deducem c ∈

{ba2, ba

}.

b) Daca c = ba2 avem c2 = ba2ba2 = baaba2 = baba2a2 = baba = bba2a =e⋅ e = e, deci ordinul lui c este 2;Daca c=ba atunci c2 = baba = bbaa = a2 ∕= e. Analog se arata ca c3 ∕= e, c4 ∕=e, c5 ∕= e iar c6 = c5c = ba2ba = b2a3 = e, deci ord c = 6.

Page 151: olimpiade2009

151

2. Prin calcul se stabileste urmatoarea tabla de operatii:

A A2 A3 A4

A A2 A3 A4 AA2 A3 A4 A A2

A3 A4 A A2 A3

A4 A A2 A3 A4

de unde rezulta ca G este grup cu elementul neutru A4 (in fapt izomorf cu(Z4,+))

3. Prin integrare pe intervalul [0,x] deducem

∫ x

0

f(t)f ′(t)dt ≥∫ x

0

tdt,

adica1

2(f(x))2 ≥ 1

2x2 de unde rezultatul.

4. Notam cu l limita sirului (xn)n. Vom arata ca limn→∞

yn = l

∫ 1

0

f(t)dt.

Avem yn − l∫ 1

0

f(t)dt =1

n

n∑k=1

(xk − l)f(k

n)+l

(n∑k=1

f(k

n)−

∫ 1

0

f(t)dt

).

Al doilea termen din membrul drept converge catre 0 din definitia integraleiRiemann.Pentru primul avem:

∣∣∣∣∣ 1nn∑k=1

(xk − l)f(k

n)

∣∣∣∣∣ ≤ sup[0,1]

∣f(t)∣n∑k=1

∣xk − l∣n

,

ce converge la 0 din lema lui Cesaro aplicata pentru sirul (∣xn − l∣)n.

Concursul interjudetean de matematica ”Petru Morosan-Trident”

1. Cu substitutia 1 + 2x = t, integrala devine:

Page 152: olimpiade2009

152

I =1

ln2

∫(t− 1)n−1 − 1

tn+1dt =

1

ln2

(∫ (t− 1

t

)n−11

t2dt−

∫1

tn+1dt

)=

=1

ln2

(∫ (1− 1

t

)n−1(1− 1

t

)′dt+

1

ntn

)=

1

ln2

(1

n

(1− 1

t

)n+

1

ntn

)+C

=1

ln2⋅ (t− 1)n + 1

ntn+ C =

2nx + 1

n⋅ ln2⋅ (1 + 2x)n+ C, pentru n ≥ 2;

Pentru n=1 obtinem

∫0dx = C.

2.a) Prin inductie dupa ”k” avem:

x1 ∘ x2 ∘ ... ∘ xk = (x1 − n)(x2 − n)...(xk − n) + n, (∀)x1, x2, ..., xk ∈ R ⇒

⇒ x(k) = (x− n)k + n, (∀)x ∈ R, k ∈ N∗, k ≥ 2 ⇒ x(2p) = (x− n)2p + n,

p ∈ N∗, (x− n)2p ≥ 0⇒ (x− n)2p + n ≥ n, (∀)x ∈ R ⇒ x2p ≥ n.

b) Avem x(3) = (x− n)3 + n

x = n+ 1⇒ (n+ 1)(3) = 13 + nx = n+ 1⇒ (n+ 2)(3) = 23 + n....................................................x = n+ n⇒ (n+ n)(3) = n3 + n

⎫⎬⎭ +⇒ (n+1)(3)+(n+2)(3)+...+(n+n)(3)

= 13 + 23 + ...+ n3 + n2 =n2(n+ 1)2

4+ n2 =

n2(n2 + 2n+ 5)

4.

3.a) Pentru x > 0⇒ 1−(1−x2)n = 1−(1−C1nx

2 +C2nx

4−...+(−1)nCnnx2n)

de unde:f(x) = n− C2

nx2 + C3

nx4 − ...+ (−1)n−1Cnnx

2n−2, (∀)x ∈ [0, 1].

Page 153: olimpiade2009

153

b)

∫ 1

0

f(x)dx = lim�→0

∫ 1

(1

x)′[1− (1− x2)n]dx = lim

�→0(− 1

x)[1− (1− x2)n∣1� ]+

+lim�→0

∫ 1

1

x2nx(1− x2)n−1dx = −1 + 2n

∫ 1

0

(1− x2)n−1dx

c) Integrand de la 0 la 1 expresia lui f(x) de la a) obtinem:

∫ 1

0

f(x)dx = C1n − C2

n

x3

3∣10 + C3

n

x5

5∣10 − ... + (−1)n−1 x

2n−1

2n− 1∣10Cnn = C1

n −1

3C2n +

1

5C3n − ...+ (−1)n−1 1

2n− 1Cnn .

Stim ca:

∫ 1

0

(1− x2)n−1dx =

∫ 1

0

x′(1− x2)n−1dx = x(1− x2)n−1∣10 -

-

∫ 1

0

x(n− 1)(1− x2)n−2(−2x)dx = (2n− 2)

∫ 1

0

[(1− x2)− 1](1− x2)n−2dx.

Notand In−1 =

∫ 1

0

(1− x2)n−1dx⇒ (2n− 1)In−1 = (2n− 2)In−2, (∀)n ≥ 2.

Aplicam relatia de mai multe ori si obtinem 2nIn−1 =2⋅ 4⋅ 6...(2n)

1⋅ 3⋅ 5...(2n− 1)si deci

identitatea.

Concursul ”Grigore Moisil”, Urziceni, Editia a V-a

1.

∫ 3

0

x2f”(x)dx = 9f ′(3)−∫ 3

0

2xf ′(x)dx;

∫ 3

0

xf ′(x)dx = 3f(3)−∫ 3

0

f(x)dx;

⇒∫ 3

0

x2f”(x)dx = 9.

Page 154: olimpiade2009

154

2. 2009(f(x))2008⋅ f ′(x) = (f(x))2008; f(x) = 0⇔ x = 0

f ′(x) =1

2009, x ∕= 0; f(x) =

x

2009.

3. ord G ≤ 3;ord G = 1 ⇒ G={1}, finalizare;ord G = 2 ⇒ G={1,−1}, finalizare;ord G = 3 ⇒ G=

{1, !, !2

}, finalizare;

4. Se foloseste schimbarea de variabila x=-t.

Concursul national de matematica ”Laurentiu Duican”, Brasov, Editia aXVI-a

1. Fie f o functie care satisface ipoteza.Avem: f(af(x)y) = bf(xy) = f(af(y)x), ∀x, y ∈ G;f injectiva, deci af(x)y = af(y)x, ∀x, y ∈ G;Deducem ca exista c ∈ G astfel incat sa avem x−1f(x) = c, ∀x ∈ G;Obtinem c = a−1b. Astfel, f(x) = a−1bx, x ∈ G, este unica functie care satis-face ipoteza.

2. Fie G : [0, 1] → R, G(t) = t2∫ t

0

f(x)dx− 2t

∫ t

0

xf(x)dx+

∫ t

0

x2f(x)dx,

t ∈ [0, 1].Avem G(0)=G(1)=0;

G derivabila, cu G′(t) = 2

∫ t

0

(t− x)f(x)dx, t ∈ [0, 1];

Concluzia se obtine prin aplicarea teoremei lui Rolle.

4.9.1 Olimpiada judeteana

Subiectul 1.a) Concluzia rezulta din faptul ca funtia F : [0,∞) → R,F (x) =∫ x0f(t)dt,este crescatoare si marginita.

b) Daca x > 0 atunci F (x)− F (x2 ) =∫ xx2f(t)dt ≥ x

2 f(x) ≥ 0.

Page 155: olimpiade2009

155

Cum limx→∞

(F (x)− F (x

2)) = 0,rezulta ca lim

x→∞xf(x) = 0.

Subiectul 2.Mai intai aratam ca numarul functiilor polinomiale ale ineluluiA este cel putin n2.Fie f, g ∈ A[X], f = aX + b si g = cX + d.Daca f si g au aceeasi functiepolinomiala atunci f(0) = g(0) si f(1) = g(1), de unde b = d si a = c, decif = g.Intrucat exista n2 polinoame de grad cel mult 1, rezulta ca exista celputin n2 functii polinomiale.Aratam ca a) implica b) .Fie f = akX

k+ak−1Xk−1 = ...+a1X+a0, k ≥ 1.Cum

f(x) = (ak +ak−1 + ...+a1)x+a0 = ax+a0,oricare ar fi x ∈ A,rezulta ca f = g, unde g = aX + a0.Prin urmare , numarul functiilor polinomiale este exact n2.Pentru a demonstra implicatia inversa ,consideram polinoamele f = X2 si g =aX + b,unde f = g .Rezulta ca x2 = ax + b,oricare ar fi x ∈ A .Pentru x = 0,obtinem b = 0,apoi, pentru x = 1 ,obtinem a = 1.Deci x2 = x oricare ar fix ∈ A.

Subiectul 3.a) Fie functiile derivabile F,G : [0, 1] → R,F (x) =∫ x

0f(t)dt

si G(x) =∫ x

0tf(t)dt.Rezulta H(x) = G(x)

x−F (x) , 0 < x < 1.Intrucat limx→0

H(x) =

G′(0) − F (0),rezulta ca H este continua in 0.Concluzia rezulta din faptul caH(0) = H(1) = 0 si H este derivabila pe (0, 1].b) Din teorema lui Rolle rezulta ca exista un punct b ∈ (0, 1) ,astfel incatH ′(b) = 0.Intrucat

H ′(x) = x2f(x)−G(x)x2 − f(x) = −G(x)

x2 , 0 < x < 1,obtinem G(b) = 0.Aplicand teorema lui Rolle functiei K : [0, b] → R,K(x) = e−xG(x),rezulta caexista un punct a ∈ (0, b),astfel incat K ′(a) = 0, i.e.G(a) = G′(a).

Subiectul 4.Fie f un polinom de grad n si g restul impartirii lui la polinomulXq−X.Cum radacinile lui Xq−X sunt toate elementele lui K ,polinoamele dinK[X] de grad n , care au aceeasi functie polinomiala cu f , sunt cele de forma(Xq −X)c+ g , unde gradul lui c este n− q,deci numarul lor este qn−q(q − 1).Intrucat numarul polinoamelor de grad n din K[X] este qn(q − 1),rezulta canumarul functiilor polinomiale atasate polinoamelor de grad n esteqn(q−1)qn−q(q−1) = qq

adica exact numarul functiilor de la K la K.Cum numarul functiilor de la K la K∗ este (q − 1)q, rezulta ca numarul poli-noamelor de grad n, care nu au nici o radacina in K este (q−1)q ⋅ (q−1)qn−q =(q − 1)q+1qn−q.Deci probabilitatea ceruta este(q−1)q+1qn−q

qn(q−1) = (1− 1q )q.

Page 156: olimpiade2009

156

4.9.2 Olimpiada nationala

Subiectul 1. Avem

0 ≤∫ 1

0

(f ′(x) + x)2dx =

∫ 1

0

(f ′(x))2dx+ 2

∫ 1

0

xf ′(x)dx+1

3=

∫ 1

0

(f ′(x))2dx+

2xf(x)∣10 − 2

∫ 1

0

f(x)dx+1

3=

∫ 1

0

(f ′(x))2 − 2

∫ 1

0

f(x)dx ≤ 0,

deci

∫ 1

0

(f ′(x) + x)2dx = 0.

Din continuitatea functiei f rezulta ca f ′(x) = −x.Asadar,

f(x) = −x2

2 + a,∨x ∈ [0, 1].

Din conditia f(1) = − 16 , obtinem a = 1

3 , deci f(x) = −x2

2 + 13 .

Subiectul 2.a) Din xk = yl = 0 rezulta (x+ y)k+l = 0 si (x ⋅ y)k+l = 0, decix+ y si x ⋅ y sunt nilpotente.b) Notam cu U(A) multimea elementelor inversabile ale lui A si cu N(A) mul-timea elementelor nilpotente ale lui A . Observam ca daca x ∈ N(A) , atunci1 + x ∈ N(A) , exista k astfel incat x2k+1 = 0, deci(1 + x)(1− x+ ...+ x2k) = 1 + x2k+1 = 1.Mai mult , daca x, y ∈ N(A), atunci(1 + x)(1 + y) = 1 + x + y + xy = 1 + z, unde z = x + y + xy ∈ N(A). Prinurmare ({1 + x∣x ∈ N(A)}, ⋅) este subgrup cu n elemente al grupului (U(A), ⋅).Intr-un inel comutativ finit , orice element neinversabil este divizor al lui zero.Rezulta ca U(A) are q−d elemente , unde q este numarul elementelor lui A.Dinteorema lui Lagrange obtinem ca n divide q − d.Pe de alta parte (N(A),+) este subgrup al grupului (A,+).Folosind din nouteorema lui Lagrange deducem ca n divide q.Prin urmare n divide d.

Subiectul 3.Cum 0,1,2 sunt toate sume de doua patrate , rezulta ca n ≥ 4.Fie k ∈ {3, ..., n − 1} astfel incat k se poate scrie ca suma de doua patrate in

Zn si sa notam cu S = Zn∖{k}.Cum orice element din S se scrie ca suma dedoua patrate , rezulta ca S este inchisa la inmultirea din Zn.Daca k ar fi par , atunci k = 2l .Fiindca l ∕= k , rezulta ca l ∈ S.Cum 2 ∈ S , arrezulta ca k = 2 ⋅ l ∈ S , absurd .Deci k este impar.Daca k /∈ U(Zn), atunci −1 ∈ S.Fiindca k = (−1) ⋅ (−k) /∈ S , rezulta ca

−k /∈ S . Deci k = −k , de unde rezulta ca n = 2k.fiindca k este impar , avem

ca k(k − 1) = 0. De aici rezulta ca k = k2 = k2 + 02 ∈ S , contradictie.

Daca k ∈ U(Zn) , cum U(Zn)∖{k} este inchisa la inmultire , rezulta ca U(Zn)∖{k}este un subgrup al lui U(Zn).Din teorema lui Lagrange ,Φ(n)−1∣Φ(n)⇒ Φ(n) =2. De aici rezulta ca n ∈ {4, 6}. Cum in Z6 orice element se poate scrie ca sumade doua patrate , rezulta ca n poate fi numai 4.Se vede usor ca 3 este singurulelement din Z4 care nu se poate scrie ca suma de doua patrate , deci n = 4 este

Page 157: olimpiade2009

157

singurul numar cu proprietatea din enunt.

Subiectul 4. Functia f este strict monotona . Presupunem mai intai ca feste strict crescatoare.Atunci pentru orice c ∈ [0, 1] sa consideram functia

gc(x) =

{x− c, x ∈ [0, c)

0, x ∈ [c, 1]

Relatia

∫ 1

0

gc(f(x))dx =

∫ 1

0

gc(x)dx este echivalenta cu∫ f−1(c)

0

(f(x)− c)dx =

∫ c

0

(x− c)dx,

adica∫ f−1(c)

0

f(x)dx = cf−1(c)− c2

2.

Notand f−1(c) = t, obtinem∫ t

0

f(x)dx = tf(t)− f2(t)

2⇒∫ t

0

(f(x)− x)dx = − (f(t)− t)2

2. (1)

Notand cu ℎ(x) = f(x)− x pentru x ∈ [0, 1] , relatia de mai sus devine∫ t

0

ℎ(x)dx = −ℎ2(t)

2,∨t ∈ [0, 1]. (2)

Observam ca ℎ este continua si ca ℎ(0) = ℎ(1) = 0.Fie t0 un punct de minimglobal al lui ℎ. Daca ℎ(t0) < 0, atunci t0 < 1. Mai mult , exista � > 0 astfel incat

ℎ(x) < 0 pentru orice x ∈ [t0, t0 + �].Asadar,

∫ t0+�

0

ℎ(x)dx <

∫ t0

0

ℎ(x)dx ⇒

−ℎ(t0 + �)

2< −ℎ

2(t0)

2⇒ ℎ(t0 + �) < ℎ(t0).

Aceasta contrazice alegerea lui t0 ca punct de minim global.Asadar , ℎ(t0) ≥ 0,deci ℎ(x) ≥ 0 pentru orice x ∈ [0, 1].Din (2) rezulta ca ℎ(x) = 0 , deci f(x) = xpentru orice x ∈ [0, 1].Daca f este strict descrescatoare , observam ca functia 1 − f(x) este strictcrescatoare si satisface conditia din enunt.Din cele de mai sus rezulta ca1− f(x) = x⇒ f(x) = 1− x pentru orice x ∈ [0, 1].